You are on page 1of 119

(Ngi gia vi cun sch, trong

bc Trng Athena caRafaeln)






.:: CHUYN N THI VO LP 10 ::.
Bin son: Trn Trung Chnh 1
CAC H MIII OING THONG CMIYIN OI
(O) : ng trn tm O
(O; R) : ng trn tm O, bn knh R
AABC : Tam gic ABC
S
ABC
: Din tch AABC


(ABC) : ng trn ngoi tip AABC
a, b, c : di cc cnh i din vi cc nh A, B, C ca AABC
h
a
, h
b
, h
c
: di cc ng cao xut pht t cc nh A, B, C ca AABC
m
a
, m
b
, m
c
: di cc ng trung tuyn xut pht t cc nh A, B, C ca AABC
l
a
, l
b
, l
c
: di cc ng phn gic xut pht t cc nh A, B, C ca AABC
R, r : Bn knh cc ng trn ngoi tip, ni tip tam gic
r
a
, r
b
, r
c
: Bn knh cc ng trn bng tip i din vi cc nh A, B, C ca AABC
pcm : iu phi chng minh
2p : Chu vi ca tam gic (p =
a b c
2
+ +
l na chu vi)
n
k 1 2 n
k=1
a = a +a +... +a

: Tng ca n s hng t a
1
n a
n
.
n
k 1 2 n
k=1
a = a a ...a
[
: Tch ca n s hng t a
1
n a
n
.

TONG HIT HIIN TMIC
1. ng thng:
nh ngha: Mt ng thng c hiu nh l mt ng di (v tn), mng (v cng) v thng
tuyt i.
Tin 'Clit: Qua hai im bt k ta lun xc nh duy nht mt ng thng v ch mt ng
thng.
K hiu: Ngi ta thng dng cc ch ci in thng a, b, c, ..., m, n, p ... t tn cho cc ng
thng hoc dng hai ch ci in hoa hay hai ch ci in thng t tn cho ng thng.
V d: AB, xy, ...
y x
B A

im khng thuc ng thng: im A khng nm trn ng thng a, im A khng thuc
ng thng a (hay ni cch khc l ng thng a khng i qua im A).
K hiu: A e a.
2. on thng:
nh ngha: on thng AB l hnh gm im A, im B v tt c cc im nm gia A v B.
B A

Hai im A v B gi l hai u mt (hay cn gi l hai mt) ca on thng AB.
Lu :
im M nm gia A v B khi v ch khi AM + MB = AB v A, M, B thng hng.
M
B A

3. Tia:
Tia l hnh gm im O v mt phn ng thng bi chia ra bi im O c gi l mt tia gc O
(c hai tia Ox v Oy nh hnh v).
www.VNMATH.com
.:: CHUYN N THI VO LP 10 ::.
Bin son: Trn Trung Chnh 2
O
y
x

Hai tia c chung mt gc O to thnh ng thng c gi hai tia i nhau (hai tia Ox v Oy trong
hnh v l hai tia i nhau)
4. im:
k hiu im, ngi ta dng cc ch ci in hoa A, B, C, ...
Bt c hnh no cng l mt tp hp cc im.
Trung im ca on thng: Trung im M ca on thng AB l im nm gia hai im A, B v
cch u hai im A v B.
M B A

Trung im M ca on thng AB cn gi l im chnh gia ca on thng AB.
Lu :
im chnh gia hai im khc vi im nm gia hai im.
5. Mt phng:
Na mt phng b a: Hnh gm ng thng a v mt phn mt phng b chia ra bi a c gi l
mt na mt phng b a.
a

Mt phng l hai na mt phng hp li theo mt phng (phng ca vect) nht nh.
u
d
Q
P

6. Gc:

Gc nhn

Gc vung

Gc t


Gc bt

Gc phn

Gc y
Gc khi

B A
A
B

ng phn gic
.:: CHUYN N THI VO LP 10 ::.
Bin son: Trn Trung Chnh 3
R
R

Chia i mt gc
bng compa v thc
k

Gc i nh

Gc ngoi ca tam gic

Gc tm ca ng
trn

(1) Hai gc ph nhau l hai gc c tng s o bng 90
0
.
y
z
x
O

Gc

xOy v gc

yOz l hai gc ph nhau.


(2) Hai gc b nhau l hai gc c tng s o bng 180
0
.
z
y
x O

Gc

xOy v gc

yOz l hai gc b nhau


(3) Hai gc so le trong: Cho hai ng thng a //b v ng thng c ct a, b ln lt ti A, B.
c
b
a
B
A
2
2
1
1

Khi :

A B =
1 1
v

A B =
2 2
.

(4) Hai gc ng v: Cho hai ng thng a //b v ng thng c ct a, b ln lt ti A, B. Khi :

1 1
A = B ,

2 2
A B = ,

3 3
A B = ,

4 4
A B = .
www.VNMATH.com
.:: CHUYN N THI VO LP 10 ::.
Bin son: Trn Trung Chnh 4
4
4
3
3
c
b
a
B
A
2
2
1
1

7. Tam gic:
7.1. K hiu:
Tam gic ABC c k hiu l AABC.
Mt tam gic ABC c ba nh (gc) ln lt l A, B, C v ba cnh l AB, BC, CA.
7.2. Cc ng trong tam gic:
ng cao: L on thng ni mi nh v vung gc vi cnh i din nh . Mt tam gic c
ba ng cao. Giao im ca ba ng cao gi l trc tm ca tam gic.
Trong AABC, c cc ng cao AH, BK, CF.
F
K
H
C
B
A

ng trung tuyn: L ng thng k t nh v i qua trung im ca cnh i din vi nh .
Mt tam gic c ba ng trung tuyn. Giao im ca ba ng trung tuyn gi l trng tm ca
tam gic.
G
N
P
M
C B
A

Trong AABC, c cc ng trung tuyn AP, BN, CM.
di ng trung tuyn:

BG AG CG 2
= = =
BN AP CM 3
GN GP GM 1
= = =
BN AP CM 3
GN GP GM 1
= = =
GB GA GC 2

ng trung trc: L ng thng vung gc vi mt cnh ti trung im ca n. Mt tam gic c
ba ng trung trc. Giao im ca ba ng trung trc gi l tm ca ng trong ngoi tip tam
gic.
.:: CHUYN N THI VO LP 10 ::.
Bin son: Trn Trung Chnh 5
d
B A

ng thng (d) l ng trung trc ca on thng AB.
O
C
B
A

im O l giao im ca ba ng trung trc.
ng phn gic: L ng thng chia mt gc thnh hai gc c s o bng nhau. Mt tam gic c
ba ng phn gic. Giao im ca ba ng phn gic gi l tm ca ng trong ni tip tip tam
gic.
Trong AABC c: OM = ON = ON.
P
N
M
C
B
A

ng trung bnh: L ng thng ni trung im hai cnh ca mt tam gic. Mt tam gic c ba
ng trung bnh. Tam gic to bi ba ng trung bnh th ng dng vi tam gic cho.
N M
A
B
C

MN gi l ng trung bnh ca tam gic. Ta c: MN // BC v
1
MN BC
2
= .
7.3. Phn loi tam gic:
Tam gic nhn: L tam gic c ba gc u nhn (s o ba gc < 90
0
).
www.VNMATH.com
.:: CHUYN N THI VO LP 10 ::.
Bin son: Trn Trung Chnh 6
A
B
C

Tam gic u: L tam gic c ba cnh v ba gc bng nhau.
Trong tam gic u, ng cao cng l ng trung tuyn, ng phn gic, ng trung trc.
C B
A
60
0
60
0
60
0

Tam gic cn: L tam gic c hai cnh bng nhau hoc hai gc mt y bng nhau.
B
A
C

Tam gic vung: L tam gic c mt gc vung (bng 90
0
).
Trong mt tam gic vung, cnh i din vi gc vung gi l cnh huyn v l cnh ln nht.
Cho AABC, c

0
A 90 = th BC
2
= AB
2
+ AC
2
. y l h thc trn l h thc Pitago.
B
A C

nh l PITAGO:
nh l thun:
Trong tam gic vung, bnh phng cnh huyn bng tng bnh phng hai cnh gc vung.
BC
2
= AB
2
+ AC
2

nh l o:
Tam gic c tng bnh phng mt cnh bng tng bnh phng hai cnh cn li l tam gic vung.
Nu tam gic ABC tha mn BC
2
= AB
2
+ AC
2
th AABC l tam gic vung ti A.
.:: CHUYN N THI VO LP 10 ::.
Bin son: Trn Trung Chnh 7
7.4. Tnh cht ca cnh v gc ca tam gic:
Tnh cht 1: Cho tam gic ABC, tng ba gc:

0
A B C 180 . + + =
Tnh cht 2: di mt cnh ln hn hiu di hai cnh kia v nh hn tng di ca chng.
AB + BC > AC > |AB - BC|
Tnh cht 3: Trong hai cnh ca cng mt tam gic, cnh i din vi gc ln hn l cnh ln hn.
Gc i din vi cnh ln hn l gc ln hn.

BC AC AB A B C. > > > >
7.5. Din tch tam gic:
(1) Cng thc tnh din tch tam gic:
1
S b.h
2
=
trong b l di ca cnh v h l di ng cao ng vi cnh b.

(2) Cng thc Heron: ( )( )( ) S p p a p b p c =
trong
( )
1
p a b c
2
= + +

l na chu vi ca tam gic.
8. ng trn:
8.1. Khi nim:
ng trn tm O bn knh R (vi R > 0) l hnh gm cc im cch im O
cho trc mt khong khng i bng R.

K hiu: (O; R), ta cng c k hiu l (O).
Lu :
- Qua ba im khng thng hng ta ch xc nh c mt ng trn.
- Mt ng trn c mt tm i xng l tm ng trn.
- Mt ng trn c v s trc i xng l cc ng knh ca ng
trn.

8.2. ng knh v dy cung:
nh l 1: Trong cc dy ca mt ng trn, dy ln nht l ng knh.
AB l ng knh, CD l dy cung th AB > CD.
nh l 2: Trong mt ng trn, ng knh vung gc vi mt dy th i
qua trung im ca dy y.
Nu OH AB ti H th AH = HB.
nh l 3: Trong mt ng trn, ng knh i qua trung im ca mt
dy khng i qua tm th vung gc vi dy y.
8.3. Lin h gia dy v khong cch t tm n dy:
nh l 1: Trong mt ng trn:
Hai dy bng nhau th cch u tm.
Nu AB = CD th OM = ON.
Hai dy cch u tm th bng nhau.
Nu OM = ON th AB = CD.
nh l 2: Trong hai dy ca mt ng trn:
Dy no ln hn th dy gn tm hn.
Nu AB > CD th OM < ON.
Dy no gn tm hn th dy ln hn.
Nu OM < ON th AB > CD.

N
M
C
D
B
A
O
N
M C
D B
A
O
H
B A
O
h
b b
h
h
b
R
O
D
C
B
A
O
www.VNMATH.com
.:: CHUYN N THI VO LP 10 ::.
Bin son: Trn Trung Chnh 8
8.4. Khong cch gia ng thng v ng trn:
Gi R l bn knh ng trn v d l khong cch t tm O n ng thng a. Ta c:
H
a
O

(d > R)
O
a
H

(d = R)
H
a
O


(d < R)
ng thng v ng trn
khng giao nhau.
ng thng v ng trn tip
xc nhau.
ng thng v ng trn
ct nhau ti hai im (giao
nhau).
nh l 1:
Nu mt ng thng l tip tuyn ca
mt ng trn th n vung gc vi
bn knh i qua tip im.
Nu a l tip tuyn vi (O) ti H th
a OH.
nh l 2:
Tip tuyn vi ng trn: Nu hai
tip tuyn ca mt ng trn ct
nhau ti mt im th im cch
u hai tip im.
AH = BH.
Tia k t im i qua tm l tia phn gic ca gc to bi hai tip tuyn.
HO l tia phn gic ca gc

AHB.
Tia k t tm i qua im l tia phn gic ca gc to bi hai bn knh i qua cc tip im.
OH l tia phn gic ca gc

AOB.

8.5. ng trn ni tip v ng trn bng tip:
ng trn ni tip:
- ng trn tip xc trong vi ba cnh ca tam gic l ng
trn ni tip tam gic.
- Tm ng trn ni tip l giao im ca ba ng phn gic
gc trong ca tam gic.
ng trn ngoi tip:
- ng trn tip xc ngoi vi ba cnh ca tam gic l ng
trn ngoi tip tam gic.
- Tm ng trn ngoi tip l giao im ca ba ng phn gic gc
ngoi ca tam gic.

8.6. V tr tng i ca hai ng trn:
Nu gi bn knh (O) l R v (O') l r th ta c:
- Hai ng trn c hai im chung c gi l hai ng trn ct
nhau.
Hai im chung A, B gi l giao im. on thng AB ni hai
im gi l dy chung.
O
a
H
B
A
O
H
.:: CHUYN N THI VO LP 10 ::.
Bin son: Trn Trung Chnh 9
B
A
O' O

(R - r < OO' < R + r)
A
O' O

(R + r = OO')
A
O' O

(R - r = OO')
Hai ng trong ct nhau. Hai ng trong tip xc nhau.
Hia ng trn trong
nhau,
O' O

(OO' > R + r)

Hai ng trong ngoi nhau.

8.7. Gc vi ng trn:
Gc tm:
nh ngha: Gc c nh trng vi tm ng trn gi l gc tm.

S o cung nh bng s o ca gc tm chn cung .

s AmB AOB =
S o cung ln bng hiu s gia 360
0
v s o cung nh.

( )
AmB AnB =
0
1
s 360 s
2

S o ca na ng trn bng 180
0
.

8.8. Lin h gia cung v dy cung:
nh l 1: Vi hai cung nh trong mt ng trn hay trong hai ng trn bng nhau:
Hai cung bng nhau cng hai dy bng nhau.
Hai dy bng nhau cng hai cung bng nhau.
nh l 2: Vi hai cung nh trong mt ng trn hay trong hai ng
trn bng nhau:
Cung ln hn cng dy ln hn.
Cung nh hn cng dy nh hn.
8.9. Gc ni tip:
nh ngha: Gc ni tip l gc c nh nm trn ng trn v hai cnh
cha hai dy cung ca dng trn .
nh l: Trong mt ng trn, s o ca gc ni tip bng na s o ca
cung b chn.

1
AOB AB
2
= s
H qu: Trong mt ng trn:
n
m

B
A
O
O
B
A
O
www.VNMATH.com
.:: CHUYN N THI VO LP 10 ::.
Bin son: Trn Trung Chnh 10
- Cc gc ni tip bng nhau chn cc cung bng nhau.

1
AOB ACB AB
2
= = s
- Cc gc ni tip cng chn mt cung hoc chn cc cung bng nhau th
bng nhau.
- Gc ni tip (nh hn hoc bng 90
0
) c s o bng na s o ca gc
tm cng chn mt cung.
- Gc ni tip chn na ng trn l gc vung.

8.10. Gc to bi tip tuyn v dy cung:
S o ca gc to bi tip tuyn v dy cung bng na s o ca cung b chn.
a
O
A
B

(s

AB ABa = )

8.11. Gc c nh bn trong ng trn v gc c nh bn ngoi ng trn.
S o ca gc c nh bn trong ng trn bng na tng s o hai cung b chn.
E
n
m
O
D
B
C
A


( )
1
BEC = sBmC+sAnD
2

S o ca gc c nh bn ngoi ng trn bng na hiu s o hai cung b chn.
O
C
A
D
B
M

O
C
B
A
M

O
m
n
B
A
M


( )
1
CMD= sCD- sAB
2
;

( )
1
BMC = sBC- sAB
2
;

( )
1
AMB = sAmB- sAnB
2

8.12. di ng trn, cung trn:
C
O
B
A
O
.:: CHUYN N THI VO LP 10 ::.
Bin son: Trn Trung Chnh 11
- Cng thc tnh di ng trn:
C = 2tR = td.
(R l bn knh, d l ng knh)
- Cng thc tnh di cung trn:
Trn ng trn bn knh R, di l ca mt cung n
0
c tnh nh sau:
Rn
180
t
= l
8.13. Din tch hnh trn, hnh qut trn:
- Din tch hnh trn:
S = tR
2
.
- Din tch hnh qut trn:

2
R n
S
360
t
= hay
R
S
2
=
l


9. Hnh hc khng gian:
Hnh tr - din tch xung quanh ca hnh tr:
- Din tch xung quanh:
S
xq
= 2tRh.
(R l bn knh y v h l chiu cao)
- Din tch ton phn:
S
tp
= 2tRh + 2tr
2
= 2tR(h + R)
- Th tch hnh tr:
V = Sh = tR
2
h.
(S l din tch y, h l chiu cao)
Hnh nn - hnh nn ct:
* Hnh nn:
- Din tch xung quanh ca hnh nn:
S
xq
= tRl.
(vi l l di ng sinh, r l bn knh y)
- Din tch ton phn ca hnh nn (tng din tch xung quanh v din tch y) l
S
tp
= tRl + tR
2
= tR(l + R)
(vi l l di ng sinh, r l bn knh y)
- Th tch hnh nn:
2
1
V R h
3
= t
(vi l l di ng sinh, r l bn knh y)
* Hnh nn ct:
- Cng thc tnh din tch xung quanh ca hnh nn ct:
( )
xq 1 2
S r r = t + l
- Th tch ca hnh nn ct:
( )
2 2
1 2 1 2
1
V h r r r r
3
= t + +
(h l chiu cao)
- Hnh cu:
h
R
l
r
1
r
2
h
l
l
n
0
R
O
l
n
0
R
O
R
h
www.VNMATH.com
.:: CHUYN N THI VO LP 10 ::.
Bin son: Trn Trung Chnh 12

- Cng thc tnh din tch mt cu:
S = 4tR
2
hay S = td
2
.
(Vi R l bn knh mt cu, d l ng knh mt cu)
- Th tch hnh cu:

3
4
V R
3
= t
(Vi R l bn knh mt cu)


.:: CHUYN N THI VO LP 10 ::.
Bin son: Trn Trung Chnh 13
CAC PMIONG PMAP CMING MINM

CMI OI 1
NMAN BIIT VA TM OIII HIIN CIA MOT MNM

1. Kin thc c bn:
1.1. Tam gic cn:
Cc phng php chng minh tam gic cn:
Phng php 1: Tam gic c hai cnh bng nhau l tam gic cn.
Phng php 2: Tam gic c hai gc bng nhau l tam gic cn.
Phng php 3: Tam gic c mt ng cao va l ng trung tuyn, ng trung trc, ng
phn gic ca mt gc v ngc li th tam gic l tam gic cn.
Lu : C th chng minh mt tam gic l tam gic cn da vo cc biu thc hoc cc h thc
c chng minh.
1.2. Tam gic u:
Cc phng php chng minh tam gic u:
Phng php 1: Tam gic c ba cnh bng nhau l tam gic u.
Phng php 2: Tam gic c ba gc bng nhau v bng 60
0
l tam gic u.
Phng php 3: Tam gic cn c s o gc nh cn bng 60
0
l tam gic u.
Phng php 4: Tam gic c cc ng cao va l ng trung tuyn, ng phn gic, ng
trung trc v ngc li l tam gic u.
1.3. Tam gic vung:
Cc phng php chng minh tam gic vung:
Phng php 1: Tam gic c mt gc vung l tam gic vung.
Phng php 2: Tam gic c hai cnh nm trn hai ng thng vung gc l tam gic vung.
Phng php 3: S dng nh l o v ng trung tuyn ca tam gic vung.
nh l: Trong mt tam gic c ng trung tuyn ng vi cnh huyn bng mt na cnh huyn th
tam gic l tam gic vung.
Phng php 4: S dng nh l o ca nh l Pitago.
nh l: Nu mt tam gic tha mn bnh phng mt cnh bng tng bnh phng hai cnh cn li
th tam gic l tam gic vung.
Tc l, nu BC
2
= AB
2
+ AC
2
th tam gic ABC vung ti A.
Phng php 5: Tam gic ni tip ng trn c mt cnh l ng knh th tam gic l tam gic
vung.
1.4. Tam gic vung cn:
Cc phng php chng minh tam gic vung cn:
Phng php 1: Tam gic vung c hai cnh gc vung bng nhau l tam gic vung cn.
Phng php 2: Tam gic vung c mt gc nhn bng 45
0
l tam gic vung cn.
Phng php 3: Tam gic cn c s o mt gc y bng 45
0
l tam gic vung cn.
1.5. Hnh thang, hnh thang cn, hnh thang vung:
Din tch hnh thang:
( )
ABCD
1
S AB CD .AH
2
= +
Tnh cht:
nh l 1: Trong hn thang cn, hai cnh bn bng nhau.
nh l 2: Trong hnh thang cn, hai ng cho bng nhau.
nh l 3: Hnh thang c hai ng cho bng nhau l hnh thang cn.
ng trung bnh ca hnh thang: ng trung bnh ca hnh thang l on thng ni trung im hai
cnh bn ca hnh thang.
www.VNMATH.com
.:: CHUYN N THI VO LP 10 ::.
Bin son: Trn Trung Chnh 14
N
M
D
C
B A

nh l 1:
ng thng i qua trung im mt cnh bn ca hnh thang v song song vi hai y th i qua
trung im cnh bn th hai.
nh l 2:
ng trung bnh ca hnh thang th song song vi hai y v bng na tng hai y.
( )
1
MN AB CD
2
+
Phng php chng minh hnh thang:
Phng php 1: Hnh thang l t gic c hai cnh i song song.
Phng php chng minh hnh thang vung:
Phng php 1: Hnh thang vung l hnh thang c mt gc vung.
Phng php chng minh hnh thang cn:
Phng php 1: Hnh thang cn l hnh thang c hai gc k mt y bng nhau.
Phng php 2: Hnh thang cn l hnh thang c hai gc k mt y bng nhau.
Phng php 3: Hnh thang cn l hnh thang c hai ng cho bng nhau.
1.6. Hnh bnh hnh:
nh ngha: Hnh bnh hnh l t gic c cc cnh i song song.
O
H
A B
D C

Din tch hnh bnh hnh:
ABCD
S AH.CD AH.AB = =
Cc phng php chng minh hnh bnh hnh:
Phng php 1: T gic c cc cnh i song song.
Phng php 2: T gic c cc cnh i bng nhau.
Phng php 3: T gic c cc cnh i song song v bng nhau.
Phng php 4: T gic c cc gc i bng nhau.
Phng php 5: T gic c hai ng cho ct nhau ti trung im ca mi ng.
1.7. Hnh ch nht:
nh ngha: Hnh ch nht l t gic c bn gc vung.
A B
D C

Chu vi hnh ch nht:
( ) ( )
ABCD
C 2 AB BC 2 AD DC = + = +
.:: CHUYN N THI VO LP 10 ::.
Bin son: Trn Trung Chnh 15
Din tch hnh ch nht:
ABCD
S AB.CD =
Cc phng php chng minh hnh ch nht:
Phng php 1: T gic c ba gc vung.
Phng php 2: Hnh thang cn c mt gc vung.
Phng php 3: Hnh bnh hnh c mt gc vung.
Phng php 4: Hnh bnh hnh c hai ng cho bng nhau.
1.8. Hnh thoi:
O
A
B
D
C

nh ngha: Hnh thoi l t gic c bn cnh bng nhau.
Tnh cht:
Trong hnh thoi: Hai ng cho vung gc vi nhau.
Hai ng cho l cc ng phn gic ca cc gc ca hnh thoi.
Chu vi hnh thoi:
ABCD
C 4AB 4BC 4CD 4DA = = = =
Din tch hnh thoi:
ABCD
1
S AC.BD BO.AC OD.AC
2
= = =
Cc phng php chng minh hnh thoi:
Phng php 1: T gic c bn cnh bng nhau.
Phng php 2: Hnh bnh hnh c hai cnh k bng nhau.
Phng php 3: Hnh bnh hnh c hai ng cho vung gc vi nhau.
Phng php 4: Hnh bnh hnh c mt ng cho l ng phn gic ca mt gc.
1.9. Hnh vung:
D C
A B

nh ngha: Hnh vung l t gic c bn gc vung v bn cnh bng nhau.
Tnh cht:
Hnh vung c tt c cc tnh cht ca hnh ch nht v hnh thoi.
Chu vi hnh vung:
ABCD
C 4AB 4BC 4CD 4AD = = = =
Din tch hnh vung:
2 2 2 2
ABCD
S AB BC CD AD = = = =
Phng php chng minh hnh vung:
Phng php 1: Hnh ch nht c hai cnh k bng nhau.
Phng php 2: Hnh ch nht c hai ng cho vung gc vi nhau.
Phng php 3: Hnh ch nht c mt ng cho l ng phn gic ca mt gc.
www.VNMATH.com
.:: CHUYN N THI VO LP 10 ::.
Bin son: Trn Trung Chnh 16
Phng php 4: Hnh thoi c mt gc vung.
Phng php 5: Hnh thoi c hai ng cho bng nhau.
2. Bi tp p dng:
Bi tp 1: Cho AABC c ba gc u nhn, ni tip ng trn tm O. Gi H l trc tm ca AABC.
D l mt im trn cung BC khng cha im A. Xc nh v tr ca im D t gic BHCD l
hnh bnh hnh.
Gii
Gi s tm c im D trn cung BC sao cho t gic BHCD l hnh
bnh hnh.
Khi : BD // HC v CD // HB.
V H l trc tm tam gic ABC nn CH AB v BH AC.
BD AB v CD AC.
Do :

00
ABD 90 = v

ACD 90 = .
Vy AD l ng knh ca ng trn tm O
Ngc li nu D l u ng knh AD ca ng trn tm O th t gic
BHCD l hnh bnh hnh.
Bi tp 2: Cho ng trn (O) ng knh AB = 2R v C l mt im thuc ng trn
(C A; C B) = = . Trn na mt phng b AB c cha im C. K tia Ax tip xc vi ng trn (O),
gi M l im chnh gia ca cung nh AC. Tia BC ct Ax ti Q , tia AM ct BC ti N. Chng minh
cc ABAN v AMCN cn.
Gii
Xt AABM v ANBM, ta c:
AB l ng knh.
Nn

0
NMB 90 AMB= = .
M l im chnh gia ca cung nh AC nn

MBN ABM BAM BNM = = .
ABAN cn ti nh B.
Xt t gic AMCB ni tip:


MCN BAM= (cng b vi

MCB)


MCN MNC = (cng bng

BAM)
AMCN cn ti nh M.
Bi tp 3: Cho AABC cn ti A, (AB > BC). im D di ng trn cnh AB, (D khng trng vi A,
B). Gi (O) l ng trn ngoi tip ABCD. Tip tuyn ca (O) ti C v D ct nhau K.
a) Chng minh t gic ADCK ni tip?
b) T gic ABCK l hnh g? V sao?
c) Xc nh v tr im D sao cho t gic ABCK l hnh bnh hnh?
Gii
c) Theo cu b, t gic ABCK l hnh thang.
Do , t gic ABCK l hnh bnh hnh.
AB // CK


BAC= ACK
M

1
ACK =
2
s

EC =
1
2
s

BD =

DCB
Nn

BCD= BAC
Dng tia Cy sao cho

BCy = BAC.
Khi , D l giao im ca

AB v Cy.
D
H
O
C B
A
N
Q
M
x
C
B
A
O
K
D
C B
A
.:: CHUYN N THI VO LP 10 ::.
Bin son: Trn Trung Chnh 17
Vi gi thit

AB >

BC th

BCA >

BAC >

BDC.
D e AB.
Vy im D xc nh nh trn l im cn tm.

3. Bi tp t luyn:
Bi tp 1:Cho tam gic u ABC ni tip ng trn tm O. D v E ln lt l im chnh gia ca
cc cung AB v AC. DE ct AB I v ct AC L.
a) Chng minh DI = IL = LE.
b) Chng minh t gic BCED l hnh ch nht.
c) Chng minh t gic ADOE l hnh thoi v tnh cc gc ca hnh ny.
Bi tp 2:Cho t gic ABCD ni tip ng trn c cc ng cho vung gc vi nhau ti I.
a) Chng minh rng nu t I ta h ng vung gc xung mt cnh ca t gic th ng vung
gc ny qua trung im ca cnh i din ca cnh .
b) Gi M, N, R, S l trung im ca cc cnh ca t gic cho. Chng minh MNRS l hnh ch
nht.
c) Chng minh ng trn ngoi tip hnh ch nht ny i qua chn cc ng vung gc h t I
xung cc cnh ca t gic.
Bi tp 3:Cho tam gic vung ABC ( ZA = 1v) c AH l ng cao. Hai ng trn ng knh
AB v AC c tm l O
1
v O
2
. Mt ct tuyn bin i i qua A ct ng trn (O
1
) v (O
2
) ln lt
ti M v N.
a) Chng minh tam gic MHN l tam gic vung.
b) T gic MBCN l hnh g?
c) Gi F, E, G ln lt l trung im ca O
1
O
2
, MN, BC. Chng minh F cch u 4 im E, G, A, H.
d) Khi ct tuyn MAN quay xung quanh im A th E vch mt ng nh th no?
Bi tp 4:Cho hnh vung ABCD. Ly B lm tm, bn knh AB, v 1/4 ng trn pha trong hnh
vung.Ly AB lm ng knh , v 1/2 ng trn pha trong hnh vung. Gi P l im tu trn
cung AC ( khng trng vi A v C). H v K ln lt l hnh chiu ca P trn AB v AD, PA v PB
ct na ng trn ln lt I v M.
a) Chng minh I l trung im ca AP.
b) Chng minh PH, BI, AM ng qui.
c) Chng minh PM = PK = AH
d) Chng minh t gic APMH l hnh thang cn.
) Tm v tr im P trn cung AC tam gic APB l u.
Bi tp 5: Cho tam gic u ABC ni tip ng trn (O). Trn cung nh AB ly mt im M.
ng thng qua A song song vi BM ct CM ti N. Chng minh rng tam gic AMN l tam gic
u.
Bi tp 6: T mt im A bn ngoi ng trn (O; R) v hai tip tuyn AB, AC vi ng trn. Gi M
l trung im ca AB. Tia CM ct ng trn ti im N. Tia AN ct ng trn ti im D.
a) Chng minh rng MB
2
= MC. MN
b) Chng minh rng AB// CD
c) Tm iu kin ca im A cho t gic ABDC l hnh thoi. Tnh din tch c hnh thoi .

CMI OI 2
CMING MINM SONG SONG
1. Kin thc c bn:
Cc phng php chng minh:
Phng php 1: Hai ng thng song song vi nhau khi v ch khi chng cng vung gc vi mt
ng thng th ba.
www.VNMATH.com
.:: CHUYN N THI VO LP 10 ::.
Bin son: Trn Trung Chnh 18
Phng php 2: Dng mi quan h gia cc gc: So le bng nhau, ng v bng nhau, trong cng
pha bng nhau,
Phng php 3: S dng nh l o ca nh l Talt.
nh l: Nu mt ng thng ct hai cnh ca mt tam gic v nh ra trn hai cnh ny nhng
on thng t l th hai ng thng song song vi cnh cn li ca tam gic.
Phng php 4: p dng tnh cht ca cc t gic c bit, ng trung bnh ca tam gic.
Phng php 5: p dng tnh cht hai dy chn gia hai cung bng bng nhau ca ng trn.
2. Bi tp p dng:
Bi tp 1: Cho AABC, trung tuyn AM, ng phn gic ca gc

AMB ct cnh AB ti D. ng
phn gic ca gc

AMC ct cnh AC E. Chng minh rng: ED // BC.


Gii
Trong AABM c MD l phn gic ca

AMB nn, ta c:
AD
DB
=
MA
MB
(1) (nh l)
Trong AAMC c ME l phn gic ca

AMC nn, ta c:
AE
EC
=
MA
MC
(2) (nh l)
V MB = MC (gi thit).
Nn t (1) v (2).
Suy ra:
AD
DB
=
AE
EC

Trong AABC c DE nh ra 2 cnh AB, AC nhng on thng t l nn DE // BC
Bi tp 2: Cho t gic ABCD. Gi K, L ln lt l trng tm ca cc tam gic ABC v tam gic
BCD. Chng minh rng KL // AD.
Gii
Gi M l trung im ca BC.
V K l trng tm ca AABC
nn MK=
1
3
MA (tnh cht trng tm ca tam gic)
hay
MK
MA
=
1
3
(1)
V L l trng tm ca ABCD
nn ML =
1
3
MD hay
ML
MD
=
1
3

(2)
T (1) v (2) suy ra
MK ML
=
MA MD
nn KL //AD (nh l Talt o)
Do trong AAMD c KL nh ra trn 2 cnh MA, MD nhng on thng t l nn
KL // AD (nh l Talt o).
Bi tp 3: Cho hnh thang ABCD (AB // CD), M l trung im ca CD. Gi I l giao im ca AM
v BD v K l giao im ca BM v AC. Chng minh rng: IK //AB.
Gii
Ta c:
IM MD
=
IA AB
(do AB // MD hay AAIB AMID)
v (Do AB // MC)
M MD = MC (gi thit)
M
L
K
D
C
B
A
K I
M
D C
B
A
E
D
C
B
A
M
.:: CHUYN N THI VO LP 10 ::.
Bin son: Trn Trung Chnh 19
Nn:
IM KM
=
IA KB
.
Suy ra IK // AB (iu phi chng minh)
V trong AAMB c IK nh ra trn 2 cnh MA, MB nhng on thng t l nn
IK // AB (nh l Talt o).
3. Bi tp t luyn:
Bi tp 1: Cho hnh thang ABCD (AB // CD, AB < CD). K AK // BC, AKBD = E; K BI //AD;
BIAC = F (K, I e CD). Chng minhn rng: EF // AB.
Bi tp 2: Cho t gic ABCD. Qua B, v Bx // CD ct AC ti E. Qua C v Cy // BA ct BD ti F.
Chng minh rng: EF // AD.
Bi tp 3: Cho hnh bnh hnh ABCD ng phn gic ca gc BAD ct BD ti M, ng phn
gic ca gc ADC ct AC ti N. Chng minh rng: MN //AD.
Bi tp 4: Cho AABC. Ly im M ty trn cnh BC. Ly N ty trn cnh AM. ng thng
DE // BC (D e AB, E e AC). Gi P l giao im ca DM v BN v Q l giao im ca CN v EM.
Chng minh rng: PQ // BC.
Bi tp 5: Tam gic cn ABC c BA = BC = a, AC = b. ng phn gic gc A ct BC ti M,
ng phn gic ca gc C ct BA ti N. Chng minh rng: MN // AC.
Bi tp 6: Cho ng trn (O), im A nm bn ngoi ng trn. K cc tip tuyn AM, AN vi
ng trn (M, N l cc tip im). V ng knh NOC. Chng minh rng AO // MN.

CMI OI J
CMING MINM MAI OIONG TMANG VIONG GOC

1. Kin thc c bn:
Phng php chng minh ng thng a v ng thng b vung gc vi nhau:
Phng php 1: Chng minh chng song song vi hai ng vung gc khc.
Phng php 2: ng thng vung gc vi mt trong hai ng thng song song th vung gc
vi ng thng cn li.
Phng php 3: Dng tnh cht ca ba ng cao v cnh i din trong mt tam gic.
Phng php 4: ng knh i qua trung im ca mt dy.
Phng php 5: Phn gic ca hai gc k b nhau.
Phng php 6: S dng gc ni tip na ng trn.
Phng php 7: S dng tnh cht ng trung trc.
Phng php 8: Tnh cht tip tuyn v ng knh ca ng trn.
2. Bi tp p dng:
Bi tp 1: Cho AABC, cc ng cao BD v CE. Gi M, N l chn cc ng vung gc k t B, C
n DE. Gi I l trung im ca DE, K l trung im ca BC. Chng minh rng: KI ED?
Chng minh
Xt ABDC c: DK l ng trung tuyn
1
DK = BC
2
(1)
Xt ABEC c: EK l ng trung tuyn
1
EK = BC
2
(2)
T (1) v (2), suy ra: DK = EK.
Suy ra: AEKD cn ti K.
M I l trung im ca DE.
Do : KI l ng cao ca AEKD KI ED.
Bi tp 2: Cho ng trn tm O, ng knh AB. S l mt im nm bn ngoi ng trn. SA v
SB ln lt ct ng trn ti M, N. Gi H giao im ca BM v AN. Chng minh rng SH AB.
www.VNMATH.com
.:: CHUYN N THI VO LP 10 ::.
Bin son: Trn Trung Chnh 20
Chng minh
Ta c:

0
AMB 90 = (t/c gc ni tip chn na ng trn)

0
ANB 90 = (t/c gc ni tip chn na ng trn)
Xt ASAB c AN, BM l hai ng cao.
M H l giao im ca AN v BM H l trc tm ca ASAB.
Suy ra: SH thuc ng cao th ba ca ASAB.
Vy SH AB.
Bi tp 3: Cho hnh thang vung ABCD,

( )
0
A D 90 = = , c CD = 2AB. Gi H l chn ng
vung gc h t D xung AC v M l trung im ca HC. Chng minh rng ng thng qua DM
vung gc vi ng thng qua BM.
Gii
E
M
H
D
C
B A

K BE CD (E e CD).
V CD = 2AB nn AB = DE = EC.
Hay E l trung im ca CD.
Xt ADHC c EM l ng trung bnh.
EM // DH EM AC (V DH AC).
Xt t gic MADE c

0
ADC 90 = v

0
AME 90 = .
Suy ra: T gic MADE ni tip ng trong ng knh AE. Tc l bn im M, A, D, E nm trn
mt ng trn. (1)
Xt t gic ABED c:

0
ADE 90 = v AB = DE.
T gic ABCD l hnh ch nht.
Bn im A, B, E, D nm trn mt ng trong ng knh AE. (2)
T (1) v (2), suy ra: M thuc ng trn ng knh AE.
Ta c: T gic ABMD ni tip.
M

0
BAD 90 =

0
BMD 90 = .
BM DM.
Bi tp 4: Cho tam gic cn ABC, gi H l trung im ca BC v E l hnh chiu ca H trn AC.
Gi O l trung im ca on thng HE. Chng minh AO vung gc vi BE.
Chng minh
Gi K l trung im ca EC.
Ta c: HK l ng trung bnh ca ABEC nn HK // EB (1)
Trong AEHC, ta c: OK l ng trung bnh nn OK // HC. (2)
M AH HC (gi thit) (3)
T (2) v (3), suy ra: OK AH (*)
Ta li c: HE AC (v E l hnh chiu ca H trn AC) (**)
T (*) v (**), suy ra: O l trc tm ca AAHK
AO HK (4)
T (1) v (4), suy ra: AO BE (iu phi chng minh).
.:: CHUYN N THI VO LP 10 ::.
Bin son: Trn Trung Chnh 21
Bi tp 5: Cho AAHC, c

( )
0
H 90 = . ng cao HE. Gi O, Kln lt l trung im ca EH v
EC. Chng minh AO vung gc vi HK.
Chng minh
T gi thit c OK l ng trung bnh ca tam gic EHC
OK // HC.
Mt khc: HC AH
OK AH
Xt AAHK c: HE AC, OK AH
O l trc tm ca AAHK
AO HK.
Bi tp 6: Cho t gic ABCD ni tip ng trn ng thi ngoi tip ng trn khc c cc tip
im M, N, P, Q ln lt vi cc cnh AB, BC, CD, DA ca t gic cho. Chng minh rng MP
vung gc vi NQ.
Chng minh
k
l
n
m
I
Q
N
P
M
D
C
B
A
O
O'

Gi (O) l ng trn ni tip t gic v (O) l ng trn ngoi tip t gic.
Ta c:


sMQPN-sMnN
B =
2
(gc c nh bn ngoi ng trn)


sPNMQ-sPkQ
D =
2
(gc c nh bn ngoi ng trn)

D + B = 180
0
(v t gic ABCD ni tip (O))

0
sPNMQ - sPkQ sMQPN - sMnN
+ =180
2 2



0
2PIN+2MmQ
=180
2



PlN+MmQ = 180
0

M


0
0
PlN MnQ 180
MIQ PIN 90
2 2
+
+ = = =
s s

MP QN. (iu phi chng minh)

3. Bi tp t luyn:
www.VNMATH.com
.:: CHUYN N THI VO LP 10 ::.
Bin son: Trn Trung Chnh 22
Bi tp 1: Cho AABC u. Gi H l trung im ca BC v E l hnh chiu ca H trn AC. Gi O l
trung im ca on thng HE. Chng minh: AO BE.
Bi tp 2: Cho tam gic vung cn ABC

( )
0
A 90 = . Gi H l trung im ca BC v E l hnh chiu
ca H trn AC. Gi O l trung im ca on thng HE. Chng minh: AO BE.
Bi tp 3: Cho AABC cn ti A, ng cao AH. H HI AC, M l trung im ca HI. Chng minh
BI AM.
Bi tp 4: Cho hnh ch nht ABCD. Gi H l hnh chiu ca B trn AC. I v N ln lt l trung
im ca AD v HC. Chng minh: BN IN.
Bi tp 5: Cho AABC cn ti A, ng cao AH. Dng hnh ch nht AHCK, HI AC. Gi M , N
ln lt l trung im ca IC v AK . Chng minh: MN BI.
Bi tp 6: Cho hnh ch nht ABCD. Gi H l hnh chiu ca B trn AC. Gi E, F, M ln lt l
trung im ca AB, DH, BH. Chng minh: AM EF.
Bi tp 7: Cho hnh ch nht ABCD. Gi H l hnh chiu ca B ln AC. E, F, M, N ln lt l trung
im ca AB, DH, HC, AD. Chng minh: EF MN.
Bi tp 8: Cho AABC

( )
0
A 90 = . H l hnh chiu ca A trn BC. I, K l th t hai im thuc AH
v CK sao cho
HK HI
=
KC IA
. Chng minh: BI AK.
Bi tp 9: Cho hnh thang vung ABCD

( )
0
A B 90 = = v AC = m, BD = n. Gi H l hnh chiu
ca A trn BC. Ly im K e HC, sao cho
KH n
=
HC m
. Chng minh: DK AK.
Bi tp 10: Cho t gic ABCD ni tip ng trn tm O. Gi E l giao im ca hai cnh i AD
v BC. Gi F l giao im ca hai cnh i DC v AB. Chng minh rng cc tia phn gic trong ca
hai gc E v F vung gc vi nhau.
Bi tp 11: Cho hnh ch nht ABCD. Trn tia AD v BC ln lt ly hai im E v F sao cho
DF=CE=DC. Trn tia DC ly im H sao cho CH = CB. Chng minh: AE FH.
Bi tp 12: Cho hnh vung ABCD. T l mt im bt k trn cnh AB (T khc A v B). Tia DT
ct tia CB ti E. ng thng CT ct AE ti M. Chng minh rng ng thng DE vung gc vi
ng thng DM.
Bi tp 13: Cho hnh vung ABCD c nh. Ly im T trn cnh AB (T khc A v B). Tia DT ct
tia CB ti E. ng thng CT ct ng thng AE ti M .ng thng BM ct ng thng DE ti
F. Tm qu tch im F khi T chy trn cnh AB.
Bi tp 14: Cho ATBE

( )
0
B 90 = . V ng phn gic BD v ng cao BF. T D dng DA v
DC theo th t vung gc vi cnh TB v cnh BE (A trn cnh TB, C trn BE). Chng minh rng
cc ng thng TC, AE, BF ct nhau ti mt im.
Bi tp 15: ng trn tm O ni tip trong tam gic ABC. Gi M v N ln lt l hai tip im
ca ng trn vi hai cnh AB v AC. Tia MN ct tia phn gic ca gc B ti P. Chng minh
BP vung gc vi CP.

CMI OI 4
CMING MINM MAI OOAN TMANG BANG NMAI

1. Kin thc c bn:
Phng php 1: Chng minh hai on thng c cng di (theo cng n v o chiu di).
Phng php 2: Chng minh hai on thng cng bng on thng th ba th bng nhau.
.:: CHUYN N THI VO LP 10 ::.
Bin son: Trn Trung Chnh 23
Phng php 3: Chng minh cc on thng bng nhau l cc cnh ca cc tam gic, t gic c
bit (hnh c bit), tam gic bng nhau.
V d: Hai cnh bn ca tam gic cn th bng nhau, cc cnh ca tam gic u th bng nhau, hai
cnh bn ca hnh thang cn, cc cp cnh i ca hnh bnh hnh, hnh ch nht, hnh thoi, hnh
vung th bng nhau.
Phng php 4: Chng minh t s di ca cc cp cnh cn chng minh lun t gi tr bng 1.
Phng php 5: S dng nh ngha, tnh cht ca:
Trung im, trung trc ca on thng.
ng trung tuyn, ng trung bnh, ng trung trc, ... trong tam gic.
ng cho ca hnh bnh hnh, hnh ch nht, hnh thoi, hnh vung, ...
2 im, 2 on thng i xng qua 1 im, 1 trc.
Phng php 6: Chng minh hai tam gic c cng din tch vi cc ng cao, cnh y tng
ng.
Phng php 7: S dng tnh cht ca dy cung v tip tuyn vi ng trn.

2. Bi tp p dng:
Bi tp 1: Cho ng trong (O) ng knh, dy CD khng ct ng knh AB. Gi H v K theo
th t l chn cc ng vung gc k t A v B n CD. Chng minh rng: CH = DK.
Chng minh

Theo gi thit, ta c: AH CD v BK CD nn AH // BK.
Suy ra: AHKB l hnh thang.
K OM CD ti M MC = MD (t/c ng knh v dy
cung) (1)
Xt hnh thang AHKB c OA = OB = R; OM // AH // BK
(cng vung gc vi CD)
OM l ng trung bnh ca hnh thang
MH = MK (2)
T (1) v (2), ta c: CH = DK.
Bi tp 2: Trong hnh vung ABCD v na ng trn ng knh AD v v cung AC m tm l
D. Ni D vi im P bt k trn cung AC, DP ct na ng trn ng knh AD K. Chng minh
PK bng khong cch t P n AB.
Chng minh
K PI AB.
Xt AAPK v AAPI:
AAPK vung ti K
(V

AKD = 90
0
gc ni tip chn na ng trn ng knh AD)
AADP cn ti D
AD = DP


2
P DAP =
Mt khc:

1
P DAP = (So le trong v AD // PI)
Do :

1 2
P P =
AAPK = AAPI (c chung cnh huyn v mt cp gc nhn bng nhau)
PK = PI.
Bi tp 3: Cho hnh thang ABCD (AB// CD) c ACD = BDC. Chng minh rng: AD = BC.
Chng minh
Gi E l giao im ca AC va BD
O
K
H
M
D
C
B A
I
2
1
K
P
D
C
B A
www.VNMATH.com
.:: CHUYN N THI VO LP 10 ::.
Bin son: Trn Trung Chnh 24
Xt AECD c:

1 1
D C = (do

ACD BCD = )
AECD l tam gic cn.
Suy ra ED = EC (1)
Do

1 1
B D = v

1 1
A C = (so le trong)
M

1 1
D C =
AEAB l tam gic cn.
Suy ra: EA = EB (2)
T (1) v (2), suy ra: AC = BD.
Hnh thang ABCD c hai ng cho bng nhau nn l hnh thang cn.
Suy ra: AD = BC.
Bi tp 4: cho hnh bnh hnh ABCD. Gi E, F ln lt l trung im ca AD, BC. Chng minh
rng: BE = DF.
Chng minh
Ta c: DE =
1
2
AD; BF =
2
1
BC
M AD = BC (hai cnh i ca hnh bnh hnh ABCD)
DE = BF.
Mt khc: DE // BF.
EBFD l hnh bnh hnh.
Vy BE = DF.
Bi tp 5: Cho hnh bnh hnh ABCD. Gi I, K ln lt l trung im ca CD, AB. ng cho BD
ct AI, CK theo th t ti M, N. Chng minh rng: DM = NB.
Chng minh
T gic AICK c: AK // IC v AK = IC
T gic AICK l hnh bnh hnh.
AI // CK.
ADCN c IC = ID v IM // CN.
Suy ra: DM = MN (1)
ABAM c: BK = KA v KN // AM.
Suy ra: MN = NB (2)
T (1) v (2), suy ra: DM = NB.
Bi tp 6: Cho tam gic ABC cn ti A.Trn tia i ca tia BC ly im M, trn tia i ca tia CB
ly im N sao cho BM= CN.
a) Chng minh: AM = AN.
b) K BH AM (H eAM), CKAN (K eAN). Chng minh: BH = CK.
c) Chng minh: AH = AK.
Chng minh

a) AAMB cn


ABC ACB =



( )
0
ABM ACN 180 ABC = = +

AABM v A ACN c:
AB = AC (gi thit)

ABM ACN = (chng minh trn)

BM = CN
(gi thit)

AABM = AACN (c.g.c)
D
C
B A
D
E
F
C
B
A
I
K
D
M
N
C
B
A
H
M
O
N
K
C B
A
.:: CHUYN N THI VO LP 10 ::.
Bin son: Trn Trung Chnh 25


M N =

AAMN cn ti A
AM = AN
b) Xt
AHBM v AKNC c:

M N = (theo cu a)

MB = CN
AHMB = AKNC (ch gn)
NK = CK.
c) Theo cu a) ta c AM = AN (1)
Theo chng minh trn: HM = KN (2)
T (1), (2) HA = AK.

3. Bi tp t luyn:
Bi tp 1: Cho hnh vung ABCD. K AC ct BD ti H. Ly hai im E, F ln lt thuc AD, BC
sao cho AE = CF, AF ct HB ti I. Gi M l trung im ca IB. Chng minh: AE= IM.
Bi tp 2: Cho tam gic ABC c AP l phn gic. Trn na mt phng b BC cha nh A, v tia
Px sao cho gc CPx bng gc BAC. Tia ny ct AC E. Chng minh rng: PB = PE.
Bi tp 3: Gi P l im nm trn ng trn (O) ngoi tip tam gic ABC. H cc ng vung
gc PA
1
, PB
1
, PC
1
xung cc cnh BC, CA, AB.
a) Chng minh rng A
1
, B
1
, C
1
thng hng.
b) Gi H l trc tm ca tam gic ABC. ng thng A
1
B
1
C
1
ct PH ti I. Chng minh IP = IH.
Bi tp 4: Dng pha ngoi tam gic ABC cc tam gic u ABD v ACE. V hnh bnh hnh
EADF. Chng minh BCF l mt tam gic u.
Bi tp 5: Cho 3 im A, B, C thng hng theo th t . Ly AB v BC l cnh dng hai tam gic
u ABE v BCF nm v cng mt pha b AC. Gi I v J l trung im ca AF v CE. Chng
minh rng:
EF
IJ =
2
.
Bi tp 6: Cho tam gic ABC v (I) l ng trn ni tip tam gic ABC. Cc tip im trn cc
cnh BC, CA, AB ln lt l A
1
, B
1
, C
1
. Gi E l im i xng ca B qua CI, F l im i xng
ca B qua AI. Chng minh rng B
1
E = B
1
F.
Bi tp 7: Cho ng trn (O) v ng thng d khng ct ng trn (O). Gi A l hnh chiu ca
(O) trn d. Qua A k mt ct tuyn ct (O) B v C. Hai tip tuyn ca (O) ti B v C ct d E v
F. Chng minh: AE = AF.
Bi tp 8: Cho ng trong (O) ng knh AB, dy CD ct ng knh AB ti G. Gi H v K ln
lt l hnh chiu ca A v B trn CD. Chng minh rng: CH = DK.
Bi tp 9: Cho t gic ACBD ni tip ng trn ng knh AB. Chng minh rng hnh chiu
vung ca cc cnh i din ca t gic trn ng cho CD bng nhau.

CMI OI 5
CAC GOC BANG NMAI

1. Kin thc c bn:
Cc phng php chng minh hai gc bng nhau:
Phng php 1: Hai gc c cng mt s o th bng nhau.
Phng php 2: Hai gc ca hai tam gic bng nhau hoc hai tam gic ng dng, hai gc ca tam
gic cn, u; hai gc ca cng mt y trong hnh thang cn, hai gc i ca hnh bnh hnh, th
bng nhau.
Phng php 3: Hai gc cng bng mt gc th 3.
Phng php 4: Tia phn gic chia mt gc thnh hai phn bng nhau.
Phng php 5: Cc gc so le trong, ng v, i nh, ...
Phng php 6: Cc gc ni tip cng chn mt cung trong mt ng trn th bng nhau.
www.VNMATH.com
.:: CHUYN N THI VO LP 10 ::.
Bin son: Trn Trung Chnh 26
Phng php 7: T gic ni tip c gc ngoi bng gc i trong.
Phng php 8: S dng hm s lng gic: sin, cos, tan v cot.
Phng php 9: S dng tnh cht ca php tnh tin, i xng, quay.

2. Bi tp p dng:
Bi tp 1: Cho hai ng trn (O) v (O) ct nhau ti A v B. V ng knh AC v AD ca (O)
v (O). Tia CA ct ng trn (O) ti F, tia DA ct ng trn (O) ti E. Chng minh:

EFC EDC =
Chng minh
Ta c:

0
CED 90 = (gc ni tip chn na ng trn (O))

0
CFD 90 = (gc ni tip chn na ng trn (O))


( )
0
CED CFD 90 = =
Hai nh E, F cng nhn cnh CD mt gc bng 90
0
.
T gic CEFD ni tip.


EFC EDC = (cng chn

EC).
Bi tp 2: Cho hnh vung ABCD c nh. E l im di ng
trn cnh CD (khc C v D). Tia AE ct ng thng BC ti F. Tia Ax vung gc vi AE ti A ct
ng thng DC ti K. BD ct KF ti I.
a) Chng minh:

CAF CKF =
b) Chng minh:

IDF IEF =
c) Chng minh: AKAF vung cn.
Chng minh
a) Ta c:

0
KAF 90 = (AK AF) v

0
KCF 90 = (ABCD
l hnh vung)
Suy ra:

( )
0
KAF KCF 90 = =
Hai nh A, C cng nhn on KF mt gc bng 90
0
.
T gic ACFK ni tip.


CAF CKF =
b) T gic ACKF ni tip nn ta c:

AFK ACK = m

0 0
ACK 45 , BDC 45 = = (ABCD l
hnh vung)
Suy ra:

( )
0
AFK BDC 45 = =
Do : T gic IDEF ni tip (V gc ngoi bng gc trong ca nh i din)


IDF IEF =
c) AAKF vung ti A (gi thit), ta c:

0 0
AFK 45 AKF 45 = = AKAF vung cn ti A.
Bi tp 3: Cho AABC c ba gc nhn ni tip ng trn (O; R). Hai ng cao BE v CF ct nhau
ti H.
a) Chng minh t gic BFEC ni tip ni tip c ng trn.
b) Hai tia BE v CF ct ng trn (O) ln lt ti M v N. Ax l tip tuyn ti A. Chng minh

xAN ANM = .
c) Chng minh:

MNC EFC = .
E
F
D
C
B
A
O'
O
M
I
K
F
E
D
C
B A
.:: CHUYN N THI VO LP 10 ::.
Bin son: Trn Trung Chnh 27
Chng minh
a) T gic BFEC c:

0
BFC 90 = (CF l ng cao)

0
BEC 90 = (BE l ng cao)
Hai nh F, E cng nhn cnh BC mt gc bng 90
0
.
T gic BFEC ni tip.
b) V Ax l tia tip tuyn ca (O).
Suy ra: AO Ax.
V

xAN ACN = (1) (gc to bi tip tuyn v dy
cung vi gc ni tip cng chn mt cung).
Ta c:

ANM ABM = (cng chn

AM)
V

ABM ACN = (cng chn

EF )
Suy ra:

ANM ACN = (2)
T (1) v (2), suy ra:

xAN ANM = . (iu phi chng minh)
c) Ta c:

MNC MBC = (gc ni tip chn na ng trn)
V

EFC MBC = (t gic BFEC ni tip)
Suy ra:

MNC EFC = . (iu phi chng minh).
Bi tp 4: Cho AABC c 3 gc nhn ni tip ng trn (O). M l im thuc cung nh AC. V
MH BC ti H, MI AC ti I.
Chng minh:

IHM ICM = .
Chng minh
Xt t gic MIHC, c:

0
MIC 90 = (MI AC)

0
MHC 90 = (MH BC)
Hai nh I, H cng nhn on MC mt gc bng 90
0
.
T gic MIHC ni tip.


IHM ICM = (cng chn

MI )
(iu phi chng minh)

Bi tp 5: ( thi HSG 12 tnh ng Nai 2013 - 2014)
Cho AABC ni tip ng trn (O) c AB < BC < AC v

ABC l gc nhn. ng trn (I) ni tip


tam gic v tip xc vi BC ti D. M, N ln lt l giao im ca hai ng thng AO, AI vi (O).
Bit A khng trng vi M v N. Chng minh:

IND IMO = .
Chng minh
Gi T l giao ca ON v BC.
D chng minh c:
IN = BN =

a
BT a
2
A A
cosNBC
cos 2cos
2 2
= =
IN a a sin A A ID
sin
A A a A
AM 2 IA
2.2R.cos 2cos . 2cos
2 2 sin A 2
= = = = =
Mt khc, ta c:

( )
DIN IAM IMO = =
T
N
D
O
I
C
B
M
A
x
O
N
M
H
F
E
C
B
M
A
I
O
H
C
B
M
A
www.VNMATH.com
.:: CHUYN N THI VO LP 10 ::.
Bin son: Trn Trung Chnh 28
Suy ra: ADIN AIAM


IND IMO = . (iu phi chng minh)

3. Bi tp t luyn:
Bi tp 1: Cho AABC, trn cnh AB v AC ln lt ly hai im D v E sao cho BD = CE. Gi M
v N ln lt l trung im ca BC v DE. ng thng qua M v N ln lt ct AB v AC ti P v
Q. Chng minh rng:

MPB MQC = .
Bi tp 2: Cho D l trung im ca on thng AM. Trn cng na mt phng b AM ta v na
ng trn ng knh AM v na ng trn ng knh AD. Tip tuyn ti D ca ng trn nh
ct na ng trn ln ti C v cc tip tuyn ti C v A ca ng trn ln ct nhau ti B. Ni P
bt k trn cung nh AC vi im D ct na ng trn nh ti K. Chng minh rng: AP l phn
gic ca

BAK.
Bi tp 3: Cho hnh vung ABCD cnh a. E l im nm gia A v B, ng thng CE ct dng
thng AD ti K. Qua C k ng thng vung gc vi CE, ct AB ti I.
a) Chng minh rng: Trung im ca IK di ng trn mt ng thng c nh khi E di ng trn
on AB.
b) Cho BE = x. Tnh BK, CK, IK v din tch t gic ACKI theo a v x.
Bi tp 4: Cho tam gic ABC vi

A < 90
o
, c AB < AC ni tip trong ng trn tm O. V ng
cao AH v bn knh OA. Chng minh rng

OAH= B- C.
Bi tp 5: Cho hai ng trn (O
1
) v (O
2
) ct nhau A v B (O
1
v O
2
thuc hai na mt phng b
AB). Qua A k ct tuyn ct ng trn (O
1
) C, ct ng trn (O
2
) D. Cc tip tuyn ca hai
ng trn k t C v D ct nhau I. Chng minh rng khi ct tuyn CAD thay i th:
a)

CBD khng i
b)

CID khng i
Bi tp 6: Cho hnh bnh hnh ABCD, P trong hnh bnh hnh sao cho

PAB= PCB. Chng minh
rng:

PBA= PDA.
Bi tp 7: Cho hnh bnh hnh ABCD, trn BC v CD ly 2 im tng ng l M v N sao cho
BN=DM. Gi I l giao im ca BN v DM. Chng minh:

AID= AIB.
Bi tp 8: Cho (O
1
) v (O
2
) tip xc trong vi nhau ti A. im C thuc (O
1
). K tip tuyn ca
(O
1
) ti C ct (O
2
) ti B v D. Chng minh:

BAC= CAD.
Bi tp 9: Cho hnh bnh hnh ABCD v im P nm ngoi hnh bnh sao cho

PAB= PCB ng
thi A v C khc pha vi ng thng PB. Qua A v ng thng Ax //DP, qua P v ng thng
Py // AD hai ng thng ny ct nhau Q.
a) chng minh t gic ABPQ ni tip.
b) Chng minh:

APB= DPC
.

Bi tp 10: (NK 2006 2007 CD) cho AABC nhn, c trc tm H. Cc ng thng BH v CH ln
lt ct AC, AB ti M, N. Bit:

0
NHM=120
.
a) Chng minh:

AMN= ABC
. Tnh:
MN
BC
.
b) Tnh:
AH
BC
.


CMI OI 6
CMING MINM MAI TAM GIAC BANG NMAI
.:: CHUYN N THI VO LP 10 ::.
Bin son: Trn Trung Chnh 29

1. Kin thc c bn:
Hai tam gic bng nhau khi v ch khi tha mn mt trong ba trng hp sau:
Trng hp1: Hai tam gic c ba cp cnh tng ng bng nhau th bng nhau (cnh-cnh-cnh).
A'
B'
C'
C
B
A

AB A' B'
AC A' C' ABC A' B' C'
BC B' C'
=

= A = A
`

=
)
(cnh-cnh-cnh)
Trng hp 2: Hai tam gic c hai cp cnh tng ng bng nhau v cp gc xen gia cc cnh
bng nhau th bng nhau (cnh-gc-cnh).
A'
B'
C'
C
B
A


AC A' C'
C C' ABC A' B' C'
BC B' C'
=

= A = A
`

=
)
(cnh-gc-cnh)
Trng hp 3: Hai tam gic c mt cp cnh bng nhau v hai cp gc k vi cp cnh y bng
nhau th bng nhau (gc-cnh-gc).
A'
B'
C'
C
B
A



B B'
BC B' C' ABC A' B' C'
C C'

= A = A
`

=
)
(gc-cnh-gc)
Lu trng hp bng nhau ca tam gic vung:
Trng hp 1: Nu hai cnh gc vung ca tam gic vung ny bng hai cnh gc vung ca tam
gic vung kia th hai tam gic vung bng nhau.
Trng hp 2: Nu mt cnh gc vung v mt gc nhn k cnh y ca tam gic vung ny bng
mt cnh gc vung v mt gc nhn k cnh y ca tam gic vung kia th hai tam gic bng
nhau.
www.VNMATH.com
.:: CHUYN N THI VO LP 10 ::.
Bin son: Trn Trung Chnh 30
Trng hp 3: Nu cnh huyn v gc nhn ca tam gic vung ny bng cnh huyn v gc nhn
ca tam gic vung kia th hai tam gic vung bng nhau.
Trng hp 4: Nu cnh huyn v cnh gc vung ca tam gic vung ny bng cnh huyn v
cnh gc vung ca tam gic vung kia th hai tam gic vung bng nhau.
2. Bi tp p dng:
Bi tp 1: Cho AABC c

0
A 90 = . Trn tia i ca AB, ly im D sao cho AB = AD. Chng minh:
AABC = AADC.
Chng minh
Xt AABC v AADC c:
AB = AC (gi thit)


0
CAD CAB 90 = =
AC cnh chung.
AABC = AADC (cnh - gc - cnh)

Bi tp 2: Cho AABC c

0
A 90 = . ng thng AH BC ti H. Trn ng vung gc vi BC ti
B ly im D khng cng na mt phng b BC vi im A sao cho AH = BD.
a) Chng minh: AAHB = ADBH.
b) Chng minh: AB // HD.
Chng minh
a) Xt AAHB v ADBH, ta c:
AH = BD (gi thit)


0
A B 90 = = (gi thit)
BH l cnh chung.
AAHB = ADBH (c g c)
b) V AAHB = ADBH

ABH BHD = (gc tng ng)
M

ABH v

BHD v tr so le
AB // HD.
Bi tp 3: Cho AABC vung ti A. V BD l tia phn gic ca gc B. V AE BC ti E. Chng
minh: AABD = AEBD.
Chng minh
Xt AABD = AEBD, ta c:


0
BAD BED 90 = = (gi thit)
BD cnh chung.


1 2
B B = (gi thit)
AABD = AEBD (cnh huyn gc nhn).

3. Bi tp t luyn:
Bi tp 1: Cho AABC c AB =AC. Gi M l trung im ca
cnh BC.
a) Chng minh: AABM = AACM.
b) Chng minh: AM BC.
Bi tp 2: Cho AABC. Qua A k ng thng song song vi BC, qua C k ng thng song song
vi AB hai ng thng ny ct nhau ti D
a) Chng minh: AABC = AADC.
b) Chng minh: AADB = ACBD.
c) Gi O l giao im ca AC v BD. Chng minh: AABO = ACOD.
2
1
E
D
C
B
A
D B
C
A
D
H
C
B
A
.:: CHUYN N THI VO LP 10 ::.
Bin son: Trn Trung Chnh 31
Bi tp 3: Cho gc vung xAy. Trn tia Ax ly 2 im B v D, trn tia Ay ly 2 im C v E sao
cho AB = AC v AD = AE.
a) Chng minh: AACD = AABE.
b) Chng minh: ABOD = ACOE.
Bi tp 4: Cho gc xOy khc gc bt. Trn tia Ox ly 2 im A v D, trn tia Oy ly 2 im C v E
sao cho OD = OE v OA = OB.
a) Chng minh: AODC = AOBE.
b) Gi A l giao im ca BE v CD. Chng minh: AAOB = AAOC.
Bi tp 5: Cho AABC, c AB = AC. Tia phn gic ca gc A ct BC ti M.
a) Chng minh: AMB = AMC.
b) Chng minh M l trung im ca cnh BC.
c) K l mt im bt k trn on thng AM, ng thng CK ct cnh AB ti I. V IH vung gc
vi BC ti H. Chng minh gc

BAC 2BIH = .
Bi tp 6: Cho gc xOy khc gc bt. Ly cc im A, B thuc tia Ox sao cho OA < OB. Ly cc
im C, D thuc tia Oy sao cho OC = OA, OB = OD. Gi M l giao im ca AD v BC. Chng
minh rng:
a) AD = BC.
b) AMAB = AMCD.
c) OM l tia phn gic ca gc xOy.
Bi tp 7: Cho AABC, (AB < AC) c AM l phn gic ca gc A (M thuc BC). Trn AC ly D sao
cho AD = AB.
a) Chng minh: BM = MD
b) Gi K l giao im ca AB v DM. Chng minh: ADAK = ABAC.
Bi tp 8: Cho tam gic ABC vung ti A. K AH BC. K HP AB v ko di c PE = PH.
K HQ AC v ko di c QF = QH.
a) Chng minh: AAPE = AAPH, AAQH = AAQF.
b) Chng minh: E, A, F thng hng v A l trung im ca EF.
Bi tp 9: Cho AABC vung C, c

0
A 60 = . Tia phn gic ca gc BAC ct BC E, k EK AB
(K e AB), k BD AE (D e AE).
Chng minh:
a) AK = KB
b) AD = BC
Bi tp 10: Cho AABC, AB = AC v M l trung im ca AC v N l trung im ca AB. Gi K l
giao im ca BM v CN. Chng minh:
a) ABNC = ACMB
b) ABKC cn ti K.
Bi tp 11: Cho on thng BC. Gi I l trung im ca BC. Trn ng trung trc ca BC ly
im A (A = I)
a) Chng minh: AAIB = AAIC.
b) K IH AB, k IK AC. Chng minh: AAHK c 2 cnh bng nhau
c) Chng minh: HK // BC.
Bi tp 12: Cho AABC vung ti A, c BD l phn gic. K DE BC (E e BC). Gi F l giao
im ca AB v DE. Chng minh rng:
a) BD l ng trung trc ca AE
b) DF = DC
c) AD < DC
d) AE // FC
www.VNMATH.com
.:: CHUYN N THI VO LP 10 ::.
Bin son: Trn Trung Chnh 32
Bi tp 13: Cho bit

0
AOB 120 = . K tia phn gic OC ca

AOB. Trn tia OC ly im M v


OAHM, OB MK.
a) Tnh s o cc

HMO v

KMO.
b) Chng minh: AMHO = AMKO.

CMI OI 7
CMING MINM MAI TAM GIAC OONG OANG

1. Kin thc c bn:
Phng php 1: Hai tam gic c gi l ng dng vi nhau nu chng c cc cp cnh tng
ng t l v cc gc tng ng t l.
Xt AABC v AA'B'C', ta c:
Nu
AB AC BC
A' B' A' C' B' C'
= = v

A A'; B B'; C C' = = = th AABC AA'B'C'.
Phng php 2: nh l Talet: Nu mt ng thng song song vi mt cnh ca tam gic v ct
hai cnh cn li th n nh ra trn cnh nhng on thng tng ng t l.
N M
C
B
A

(MN // BC)
Ta c:
AM AN
=
AB AC
;
AM AN
=
MB NC

Phng php 3: Chng minh cc iu kin cn v hai tam gic ng dng:
Hai tam gic c cc cp cnh tng ng t l th ng dng.
Hai tam gic c hai cp gc tng ng bng nhau th ng dng.
Hai tam gic c hai cp cnh tng ng t l, hai gc xen gia hai cp cnh y bng nhau.
Phng php 4: Chng minh trng hp th nht (cnh-cnh-cnh): Nu 3 cnh ca tam gic ny
t l vi 3 cnh ca tam gic kia th 2 tam gic ng dng.
ABC A

AB AC BC
A' B' C'
A' B' A' C' B' C'
A = =

Phng php 5: Chng minh trng hp th 2 (cnh-gc-cnh): Nu 2 cnh ca tam gic ny t l
vi 2 cnh ca tam gic kia v 2 gc to bi to cc cp cnh bng nhau th hai tam gic ng
dng.
AABC AABC

AB AC
A' B' A' C'
A A'

.

Phng php 6: Chng minh trng hp th 3 (gc-gc): Nu 2 gc ca tam gic ny ln lt
bng 2 gc ca tam gic kia th hai tam gic ng dng.
AABC AABC


A A'
B B'

.

Phng php 7: S dng chng minh cho tam gic vung:
.:: CHUYN N THI VO LP 10 ::.
Bin son: Trn Trung Chnh 33
- Tam gic vung ny c mt gc nhn bng gc nhn ca tam gic vung kia th hai tam gic
ng dng.
- Tam gic vung ny c hai cnh gc vung t l vi hai cnh gc vung ca tam gic vung kia
th hai tam gic ng dng.
- Nu cnh huyn v mt cnh ca tam gic vung ny t l vi cnh huyn v cnh gc vung ca
tam gic vung kia th hai tam gic ng dng.
Phng php 8:
Chng minh cc tnh cht ca t s ng dng suy ra hai tam gic ng dng:
- T s hai ng phn gic, hai ng cao, hai ng trung tuyn, hai bn knh ni tip v ngoi
tip, hai chu vi tng ng ca hai tam gic ng dng bng t s ng dng.
- T s hai ng cao ca hai tam gic ng dng:
Ta c: ABC

A'B'C' A , BH v BH l hai ng cao.


Nu a l t s ng dng ca hai tam gic ABC v

A'B'C' A th
BH
= a
B'H'
.
- T s hai ng phn gic ca hai tam gic ng dng:
Ta c:

ABC

A'B'C' A , BD v BD l hai ng phn gic ln lt ca

B v

B' .
Nu a l t s ng dng ca hai tam gic ABC v

A'B'C' A th
BD
= a
B'D'
.
- T s hai ng trung tuyn ca hai tam gic ng dng:
Ta c:

ABC

A'B'C' A , BM v BM l hai ng trung tuyn.


Nu a l t s ng dng ca hai tam gic ABC v

A'B'C' A th
BM
= a
B'M'
.
- T s chu vi ca hai tam gic ng dng:
Ta c:

ABC

A'B'C' A .
Nu a l t s ng dng ca hai tam gic ABC v

A'B'C' A th

ABC
A'B'C'
C AB+BC+CA
= = a
C A'B' +B'C' +C'A'
.
- T s bn knh ng trn ngoi ca hai tam gic ng dng:
Ta c: ABC

A'B'C' A v OM, ON, OP l bn knh ng trn ngoi tip ABC, OM, ON,
OP l bn knh ng trn ngoi tip A'B'C' A
N'u a l t s ng dng ca hai tam gic ABC v

A'B'C' A th

OM ON OP
= = = a
O'M' O'N' O'P'
.
- T s bn knh ng trn ngoi ca hai tam gic ng dng:
Ta c: ABC A'B'C' A v OM, ON, OP l bn knh ng trn ngoi tip ABC, OM, ON,
OP l bn knh ng trn ngoi tip A'B'C' A
N'u a l t s ng dng ca hai tam gic ABC v

A'B'C' A th

OM ON OP
= = = a
O'M' O'N' O'P'
.
- T s din tch ca hai tam gic ng dng th bng bnh phng t s ng dng.
Nu ABC A'B'C' A v l t s ng dng ca hai tam gic th

2 ABC
A'B'C'
S
S
= a
2. Bi tp p dng:
Bi tp 1: Cho AABC cn ti A; BC = 2a. Gi M l trung im ca BC. Ly cc im D v E trn
AB; AC sao cho

DME =

B.
www.VNMATH.com
.:: CHUYN N THI VO LP 10 ::.
Bin son: Trn Trung Chnh 34
a) Chng minh rng: ABDM ACME
b) Chng minh: AMDE ADBM
c) Chng minh: BD.CE khng i?
Chng minh
a) Ta c:

DBM ECM = (1)
v

DBM DME = (gi thit)
M


0
0
DBM BMD MDB 180
DME BMD CME 180
+ + =
+ + =



MDB CME = (2)
T (1) v (2), suy ra: ABDM ACME (g - g).
b) V ABDM ACME nn
BD DM
CM ME
= v BM = CM (gi thit)

BD DM
BM ME
=
AMDE ADBM.
c) V ABDM ACME

BD BM
=
CM CE

BD.CE = CM . BM
M CM = BM =
BC
2
= a BD . CE =
2
a
4
(khng i)
Bi tp 2: Cho AABC, BD v CE l 2 ng cao ca AABC. DF v EG l 2 ng cao ca AADE.
Chng minh rng: AADE AABC ng dng.
Chng minh
Xt AADB v AAEC, ta c:

A l gc chung.

0
AEC ADB 90 = =
AADB AAEC (g - g)
Suy ra:
AD AB
AE AC
=

AD AE
AB AC
=
V

A = 90
0

AADE AABC (g - c - g)
Bi tp 3: Ly im M trn ng cho AC ca t gic ABCD c

0
B D 90 = = . K MN BC (N e BC)
v MP AD (P eAD). Chng minh:
MN MP
1
AB CD
+ = .
Chng minh
V AB BC (gi thit)
MN BC (gi thit)
Nn MN // AB
E
D
M
C
B
A
G
F
E
D
C
A
B
.:: CHUYN N THI VO LP 10 ::.
Bin son: Trn Trung Chnh 35
ACNM ACBA
MN MC
AB AC
= (1)
Ta c: MP // CD nn AAMP AACD

MP AM
CD AC
= (2)
Cng v vi v ca (1) v (2) ta c:

MN MP MC AM AC
1
AB CD AC AC
+
+ = = =
Vy
MN MP
1
AB CD
+ = .

Bi tp 4: Cho on thng AB. Gi O l trung im ca AB. V v 1 pha AB cc tia Ax v By
vung gc vi AB. Ly C trn Ax, D trn By sao cho gc COD = 90
0
.
a) Chng minh rng: AACO ABDO.
b) Chng minh rng: CD = AC + BD.
c) K OM CD ti M, gi N l giao im ca AD vi BC. Chng minh rng: MN // AC.
Chng minh
a) Ta c:


AOC BOE = (1)



0
0
BOE BOD 90
BDO BOD 90
+ =
+ =



BOE BDO = (2)
Xt AACO v ABDO, c:


0
OAC DBO 90 = = (gi thit)


BOE BDO = (theo (2))
AACO ABDO (g - g)
b) K CO ct DB ti E.
Ta c: AAOC = ABOE (g - c - g)
OC = OE.
Xt ACOD v AEOD, c:
OC = OE (chng minh trn)


0
COD EOD 90 = =
OD l cnh chung.
ACOD = AEOD (c - g - c).
CD = ED (cnh tng ng).
Ta c: AC = BE AC + BD = BE + BD = ED (V CD = ED)
Vy: AC + BD = CD.
c) Ta c: AANC ADNB.

AN AC
ND BD
=

AN BE
ND BD
= (V AC = BE)
V CD = ED nn ACDE cn ti D.
OD l ng cao h t nh D.
Theo chng minh cu b, ta c:
OB = OM (2 ng cao tng ng)
P
M
N
D
B
C
A
N
M
E
D
C
O
B
A
www.VNMATH.com
.:: CHUYN N THI VO LP 10 ::.
Bin son: Trn Trung Chnh 36
CM = BE (hnh chiu ng vi cc cnh bng nhau)
MD = BD (hnh chiu ng vi cc cnh bng nhau)

AN BE CM
ND BD MD
= =

AN CM
ND MD
=
Theo nh l Talet, ta c: MN // AC.
3. Bi tp t luyn:
Bi tp 1: Cho hnh bnh hnh ABCD vi ng cho AC > BD. Gi E v F l chn ng vung
gc k t C n cc ng thng AB v AD. Gi G l chn ng vung gc k t B n AC.
a) Chng minh rng: ACBG AACF.
b) Chng minh rng: AB.AE + AD.AF = AC
2
.
Bi tp 2: Cho AABC, M l trung im ca cnh BC. T mt im E trn cnh BC, ta k Ex // AM.
Ex ct tia CA F v tia BA G. Chng minh rng: FE + EG = 2AM.
Bi tp 3: Cho hnh bnh hnh ABCD, trn ng cho AC ly I. Tia DI ct ng thng AB ti M,
ct ng thng BC ti N.
a) Chng minh rng:
AM DM CB
AB DN CN
= =
b) Chng minh rng: ID
2
= IM.IN.
Bi tp 4: Cho AABC, BD v CE l 2 ng cao ca AABC. DF v EG l 2 ng cao ca AADE.
Chng minh rng:
a) AADE AABC.
b) FG // BC.
Bi tp 5: Cho AABC (AB < AC). Hai ng cao BD v CE ct nhau ti H.
a) So snh

BAH CAH = .
b) So snh 2 on thng BD v CE.
c) Chng minh: AADE AABC.
Bi tp 6: Cho 4 im A, E, F, B theo th t y trn 1 ng thng. Trn cng 1 na mt phng b
AB, v cc hnh vung ABCD; FGHE.
a) Gi O l giao im ca AG v BH. Chng minh: AOHE AOBC.
b) Chng minh rng: Cc ng thng CE v FD cng i qua O.
Bi tp 7: Cho AABC c cc trung im ca BC, CA, AB theo th t l D, E, F. Trn cnh BC ly
im M v N sao cho BM = MN = NC. Gi P l giao im ca AM v BE; Q l giao im ca CF
v AN. Chng minh rng:
a) F, P, D thng hng v D, Q, E thng hng.
b) AABC ADQP.
Bi tp 8: Cho AABC; H, G, O ln lt l trc tm, trng tm, giao im 3 ng trung trc ca A.
Gi E, D theo th t l trung im ca AB v AC.
Chng minh :
a) AOED AHCB
b) AGOD AGBH
c) Ba im O, G, H thng hng v GH = 2OG.
Bi tp 9: Cho AABC, AD l phn gic

A ; AB < AC. Trn tia i ca DA ly im I sao cho



ACI = BDA. Chng minh rng:
a) AADB AACI; AADB ACDI
b) AD
2
= AB.AC - BD.DC.
.:: CHUYN N THI VO LP 10 ::.
Bin son: Trn Trung Chnh 37
Bi tp 10: Cho tam gic ABC c cc gc u nhn. Cc ng cao AD, BE, CF ct nhau H.
Chng minh rng:
a) AE.AC = AF.AB
b) AAFE AACB
c) AFHE ABHC
d) BF.BA + CE.CA = BC
2
.

CMI OI
MI TMIC IIONG THONG TAM GIAC VIONG

1. Kin thc c bn:
(1) AB
2
= BH.BC;
AC
2
= CH.BC
(2) AB.AC = AH.BC
(3) AH
2
= BH.HC
(4)
2 2 2
1 1 1
AH AB AC
= +
Kt qu:
Vi tam gic u cnh l a, ta c:
2
a 3 a 3
h = ; S =
2 4

T s lng gic p dng trong tam gic vung:
t

ACB ; ABC = o =|, khi :

AB AH
sin ;
BC HC
AC HC
cos ;
BC AC
o = =
o = =


AB AH
tan
AC HC
AC HC
cot
AB AH
o = =
o = =


b a sin B a cosC c tan B ccot C
c a cos B a sinC bcot B btanC
= = = =
= = = =

Kt qu suy ra:
(1) sin cos ; cos sin ; tan cot ; cot tan o = | o = | o = | o = |

sin cos
(2) 0 sin 1; 0 cos 1; tan ; cot
cos sin
o o
< o < < o < o = o =
o o


2 2 2
2 2
1 1
(3) sin cos 1; tan .cot 1; 1 tan ; 1 cot
cos sin
o+ o = o o = + o = + o =
o o

4) Cho AABC nhn, BC = a; AC = b; AB = c. khi :
2 2 2
ABC
1
a b c 2bc.cos A; S bc.sin A
2
A
= + =

2. Bi tp p dng:
Bi tp 1: Cho AABC vung ti A, ng cao AH. Bit AH = 9cm, CH = 16cm.
a) Tnh di cc cnh AB, AC.
b) Tnh chiu cao AH.
H
C B
A

H
C B
A
www.VNMATH.com
.:: CHUYN N THI VO LP 10 ::.
Bin son: Trn Trung Chnh 38
Gii
a) Ta c: BC = BH + HC = 9 + 16 = 25 (cm)
AABC vung ti A, AH BC (gi thit)
S dng h thc v gc vung v hnh chiu ca n ln cnh huyn, ta c:
AB
2
= BH.HC = 9.25 = 225.
( ) AB 225 15 cm = =

AC
2
= CH.CB = 16.25 = 400
Suy ra:
( ) AC 400 20 cm = =

b) Theo h thc lin h gia ng cao thuc cnh huyn v hnh chiu ca hai gc vung trn cnh
huyn, ta c:
AH
2
= BH.HC = 9.16 = 144 AH = 12 (cm)
Bi tp 2: Cho AABC vung ti A, AB = 30 (cm), tanB =
8
15

a) Tnh AC, BC.
b) Tnh sinB, cosB, cotB.
Gii
a) Trong AABC vung ti A, ta c:
tanB =
AC 8
AB 15
=

m AB = 30 (cm) nn ta c:
AC 8 30.8
AC 16
AB 15 15
= = =

(cm)
Theo nh l Pitago, ta c:
BC
2
= AB
2
+ AC
2
= 30
2
+ 16
2
= 1156
Suy ra: BC = 34 (cm)
b) Theo nh ngha, ta c cc t s lng gic ca cc gc l:
AC 16
sin B 0, 4706
BC 34
AB 30
cosB 0, 8824
BC 34
AB 30
tan B 1, 875
AC 16
= = ~
= = ~
= = =

Bi tp 3: Cho AABC, ng cao AH (H e BC),

0
B 42 = AB = 12cm, BC = 22cm. Tnh cc cnh v
gc cn li ca tam gic.
Gii
Trong AAHB vung ti H,

0
B 42 = nn

0 0 0
HAB 90 42 48 = =

p dng h thc lng lin h gia cc cnh v gc trong ca tam gic vung AHB, ta c:
AH = AB.sinB = 12.sin 42
0
~ 12.0,669 ~ 8,028 (cm)
BH = AB.cosB = 12.cos 42
0
~ 12.0,743 ~ 8,916 (cm)
Trong tam gic vung AHB, ta c:
16 9
H
C B
A
30
C
B
A
.:: CHUYN N THI VO LP 10 ::.
Bin son: Trn Trung Chnh 39

0
0 0 0
AH 0, 028
tan C 0, 614
HC 13, 084
C 31 30'
HAC 90 31 30' 58 30'
= ~ =
~
~ =

Do :

0 0 0
BAC 48 58 30' 106 30'
AH AC.sinC
= + =
=
Suy ra:
( )
0
AH 8, 028
AC 15, 35 cm
sinC sin31 30'
= ~ ~


3. Bi tp t luyn:
Bi tp 1: Cho AABC vung ti A, bit:
a) a = 72cm,

0
B 58 =
b) b = 20cm,

0
B 48 =
c) b = 15cm,

0
C 30 =
d) b = 21cm, c = 18cm.
Bi tp 2: Cho AABC vung ti A, ng cao AH. Bit HB = 25cm, HC = 64cm. Tnh

B, C.
Bi tp 3: Chng minh rng: Nu mt tam gic c hai cnh bng a v b, gc nhn to bi hai ng
thng bng c th din tch ca tam gic bng:
1
S ab.sinc
2
=
Bi tp 4: Cho AABC c, AB = 16cm v

0
B 60 = .
a) Tnh BC.
b) Tnh S
ABCD
.
Bi tp 5: Cho AABC vung ti A. Gi H l chn ng cao h t A. Bit rng AB = 7cm, AC =
9cm. Tnh BH, CH, AH.
Bi tp 6: Cho AABC vung cn ti A, ng cao AH. Bit BC = a, AH = h. Tnh di cnh bn
theo a, h.
Bi tp 7: Cho AABC vung ti A, ng cao AH, k HM AB ti M. Chng minh:
3
2
AB
BM
BC
=
Bi tp 8: Cho AABC c AB = 48cm, AC = 14cm, BC = 50cm. Tnh di ng phn gic ca
gc C.
Bi tp 9: Cho AABC vung ti A c AB = 3cm; BC = 5cm. AH l ng cao. Tnh BH; CH; AC
v AH.
Bi tp 10: Cho ABC cn ti A c BC = 16cm; AH = 6cm. Mt im D e BH sao cho BD = 3,5
cm. Chng minh: ADAC vung.
Bi tp 11: Cho AABC vung ti A c AC = 10cm; AB = 8cm. Tnh:
a) BC.
b) Hnh chiu ca AB v AC ln BC.
c) ng cao AH.
Bi tp 12: Cho ng trn tmO bn knh R = 10cm.Dy cung AB bt k c trung im I.
a) Tnh AB nu OI = 7cm.
b) Tnh OI nu AB = 14cm.
H
C B
A
www.VNMATH.com
.:: CHUYN N THI VO LP 10 ::.
Bin son: Trn Trung Chnh 40
Bi tp 13: Cho ng trn tm O c ng knh AB = 26,5 cm. V dy cung AC = 22,5cm. H l
hnh chiu ca C trn AB, ni BC. Tnh BC; BH; CH v OH.
Bi tp 14: Hnh thang ABCD cn; y ln AB = 30cm, y nh CD = 10cm v gc A l 60
0
.
a) Tnh cnh BC.
b) Gi M; N ln lt l trung im AB v CD. Tnh MN.
Bi tp 15: Cho a gic li ABCD c AB = AC = AD = 10cm, gc B bng 60
0
v gc A l 90
0
.
a) Tnh ng cho BD.
b) Tnh khong cch BH v iu kin t B v D n AC.
c) Tnh HK.
d) V BE DC ko di. Tnh BE; CE v DC.
Bi tp 16: Cho on thng AB=2a. T trung im O ca AB v Ox AB ti O. Trn Ox ly D:
OD =
a
2
. T B k BC AD ko di.
a) Tnh AD; AC v BC theo a.
b) Ko di DO mt on OE = a. Chng minh: Bn im A, C, B v E cng nm trn mt ng
trn.
b) Xc nh tnh cht CE vi gc ACB.
c) V ng vung gc vi BC ti B ct CE ti F. Tnh BF.
d) Gi P l giao im ca AB v CE. Tnh AP v BP.
Bi tp 17: Cho AABC nhn, ni tip (O; R) c:

0
AOB 90 = v

0
AOC 120 =
a) Chng minh: O trong tam gic ABC.
b) Tnh cc gc tam gic ABC.
c) Tnh ng cao AH v BC theo R.
Bi tp 18: Cho AABC c ba gc nhn. AB = c, AC = b, BC = a. Chng minh rng:
a b c
sin A sin B sinC
= =
Bi tp 19: Cho AABC c ba gc nhn. AB = c, AC = b, BC = a. Chng minh rng:
b
2
= a
2
+ c
2
- 2ac.cosB.
Bi tp 20: Cho AABC vung ti A,

( )
0
C a, a 45 = < trung tuyn AM. ng cao AH. Bit BC = a,
AC = b, AH = h.
a) Tnh: sina, sin2a theo a, b, h.
b) Chng minh rng: sin2a = 2sina.cosa.
Bi tp 21: Cho AABC cn ti A. ng cao thuc cnh bn bng h, gc y bng a. Chng minh
rng:
2
ABC
h
S
4sina.cos a
A
= .
Bi tp 22: Cho hnh thang ABCD, y ln AB = 20cm, cnh bn AD = 8cm v to vi y ln AB
mt gc 65
0
.
a) Tnh di ng cao AH v y nh CD.
b) Tnh s gc

ABD v ng cho BD.


Bi tp 23: Cho hnh thang ABCD c

0
A D 90 = = , AD = 30cm, CD = 18cm v BC = 20cm.
a) Tnh cch gc

ABC, BCD.
b) Tnh cc gc

DAC, ADB v d di cc ng cho AC, BD.
Bi tp 24: Cho AABC. Bit AB = 10cm, AC = 24cm, BC = 26cm.
a) Chng minh rng: AABC vung ti A.
b) Tnh: sinB, sinC.
c) Tnh chiu cao AH v on thng m chiu cao n chia ra trn BC.

.:: CHUYN N THI VO LP 10 ::.
Bin son: Trn Trung Chnh 41
CMI OI 9
CMING MINM CAC MI TMIC MNM MOC

1. Kin thc c bn:
- Dng nh l Talet, tnh cht ng phn gic, tam gic ng dng, cc h thc lng trong tam
gic vung,
Gi s cn chng minh: MA.MB = MC.MD
Lp s : MA.MB = MC.MD
MA MD
MAD
MC MB
: = :A AMCB hoc AMAC AMDB
Ngoi ra cn ch n vic s dng cc h thc trong tam gic vung; phng tch ca mt im
vi ng trn.
2. Bi tp p dng:
Bi tp 1: Cho ng trn (O; R), tip tuyn Ax. Trn tip tuyn Ax, ly im F sao cho BF ct
ng trn ti C. Tia phn gic ca gc ABF ct Ax ti E v ct ng trn ti D.
a) Chng minh OD // BC.
b) Chng minh h thc: BD.BE = BC.BF
Chng minh
a) BODcn O (v OD = OB = R)

OBD ODB =
M

OBD= CBD (ga thit) nn

ODB= CBD.
Do : OD // BC.
b) Ta c:

0
ADB= 90 (gc ni tip chn na ng trn (O)
AD BE .

0
ACB 90 = (gc ni tip chn na ng trn (O)
AC BF .
EAB A vung A (do Ax l tip tuyn ), c AD BE nn AB
2
= BD.BE (1)
FAB vung A (do Ax l tip tuyn), c AC BF nn AB
2
= BC.BF (2)
T (1) v (2) suy ra: BD.BE = BC.BF.
Bi tp 2: Cho tam gic ABC nhn, cc ng cao BD v CE ct nhau ti H.
a) Chng minh t gic BCDE ni tip.
b) Chng minh: AD.AC = AE.AB.
Chng minh
a) Xt t gic BCDE, c:

0
0
BDC 90
BEC 90
=
=

Ta c hai nh D, E cng nhn cnh BC vi mt gc bng 90
0
.
T gic BCDE ni tip.
b) Xt AADB v AAEC, ta c:

0
ADB AEC 90 = = (V BD, CE l hai ng cao)

A l gc chung.
AADB AAEC (g - g).

AD AB
AE AC
=
AD.AC = AE.AB
H
C
D
E
B
A
D
E
C
B
F
x
A
O
www.VNMATH.com
.:: CHUYN N THI VO LP 10 ::.
Bin son: Trn Trung Chnh 42
Bi tp 3: T im A ngoi ng trn (O), k hai tip tuyn AB, AC ti ng trn (B, C l cc
tip im). ng thng qua A ct ng trn (O) ti D v E (D nm gia A v E, dy DE khng
qua tm O). Gi H l trung im ca DE, AE ct BC ti K .
a) Chng minh t gic ABOC ni tip ng trn .
b) Chng minh: HA l tia phn gic ca

BHC
c) Chng minh:
2 1 1
= +
AK AD AE
.
Chng minh
a) Ta c:

0
ABO= ACO= 90 (tnh cht tip tuyn)
Trong t gic ABOC c

0
ABO+ACO=180 nn ni tip c trong mt ng trn.
b) Ta c: AB = AC (tnh cht hai tip tuyn ct nhau).
Suy ra:

AB= AC.
Do :

AHB= AHC.
Vy HA l tia phn gic ca

BHC.
c) Chng minh:
2 1 1
= +
AK AD AE
:
Xt AABD v AAEB, c:

BAE l gc chung

ABD= AEB(=
1
2
s

BD)
Suy ra: AABD AAEB
Do :
2
AB AD
AB AD.AE
AE AB
= = (1)
Xt AABK v AAHB, c:

BAH l gc chung

ABK = AHB (do

AB= AC)
AABK AAHB.
Suy ra:
2
AK AB
AB AK.AH
AB AH
= = (2)
T (1) v (2) suy ra: AE.AD = AK. AH
1 AH
AK AE.AD
=
2 2AH
AK AE.AD
= =
( ) 2 AD DH
AE.AD
+
=
2AD 2DH
AE.AD
+
=
AD AD ED
AE.AD
+ +
=
AE+AD
AE.AD
=
1 1
+
AD AE

(do AD + DE = AE v DE = 2DH).
Vy
2 1 1
= +
AK AD AE
(pcm).

3. Bi tp t luyn:
Bi tp 1: Cho (O) c ng knh AB. Qua A k tip tuyn xy. Ly im M e Ax; ni BM ct (O)
ti C. Chng minh: MA
2
= MB.MC.
Bi tp 2: Cho AABC u, ni tip ng trn (O). D l mt im trn cung BC (BC l cung nh).
CD v AB ko di ct nhau M; BD v AC ko di ct nhau N. Chng minh: AB
2
= BM.CN.
Bi tp 3: Cho AABC c AB < AC. T M e AB v MEF //BC ct AC ti E v ng thng song
song AB v t C ti F. AC ct BF ti I. Chng minh: IC
2
= IE.IA.
H
K
D
E
C
B
A
O
.:: CHUYN N THI VO LP 10 ::.
Bin son: Trn Trung Chnh 43
Bi tp 4: Cho hnh ch nht ABCD c AB = 36mm; AD = 24mm. T D ni n trung im M ca
AB ct AC ti I v CB ko di ti K. Chng minh: ID
2
= IM.IK.
Bi tp 5: Cho AABC vung ti A. V phn gic trong AD ca gc A (D e BC). Gi khong cch
t D n AB l d. Bit AB = c, AC = b, BC = a. Chng minh:
1 1 1
= +
d b c
.
Bi tp 6: Cho (O; R) v hai dy cung song song nhau AD v BE v hai pha ca dy AB v cng
hp vi AB mt gc 45
0
. Ni DE ct AB ti M. Chng minh: MA
2
+ MB
2
+ MD
2
+ ME
2
= 4R
2
.
Bi tp 7: Cho ba im A, B, C cng nm trn mt ng thng xy theo th t trn. V ng trn
(O) i qua B v C. T A v hai tip tuyn AM v AN. Chng minh AM
2
= AN
2
= AB.AC.
Bi tp 8: Trn cung nh BC ca ng trn ngoi tip tam gic u ABC, ly mt im P tu .
Gi l giao im ca AP v BC.
a) Chng minh: BC
2
= AP . AQ .
b) Trn AP ly im M sao cho PM = PB. Chng minh: BP + PC = AP.
c) Chng minh:
1 1 1
= +
PQ PB PC
.
Bi tp 9: Cho hnh bnh hnh ABCD (AC > BD). V CE AB v FC AD. Chng minh rng:
AB.AE + AD.AF = AC
2
.
Bi tp 10: Cho tam gic ABC, M l trung im ca cnh BC. T 1 im E trn cnh BC ta k
Ex // AM. Ex ct tia CA F v tia BA G. Chng minh rng: FE + EG = 2AM.
Bi tp 11: Cho Cho hnh bnh hnh ABCD, trn ng cho AC ly I. Tia DI ct ng thng AB
ti M, ct ng thng BC ti N.
a) Chng minh rng:
AM DM CB
AB DN CN
= =
b) Chng minh rng: ID
2
= IM.IN.
Bi tp 12: Ly 1 im O trong tam gic ABC. Cc tia AO, BO, CO ct BC, AC, AB ln lt ti P,
Q, R. Chng minh rng:
OA OB OC
2
AP BQ CR
+ + = .
Bi tp 13: Cho tam gic ABC (AB = AC) c gc nh bng 20
0
; cnh y l a; cnh bn l b.

Chng minh rng: a
3
+ b
3
= 3ab
2
.
Bi tp 14: Cho AABC c = 30
0
. Dng bn ngoi ABCD u. Chng minh: AD
2
= AB
2
+ AC
2
.
Bi tp 15: Cho AABC cn ti A (

0
A 90 = ). T B k BM AC. Chng minh rng:
2
AM AB
2 1
AC BC
| |
=
|
\ .
.

CMI OI 10
TI GIAC NOI TIIP OIONG THON

1. Kin thc c bn:
Cc phng php chng minh t gic ni tip:
- Chng minh bn nh ca t gic cng cch u mt im.
- Chng minh t gic c tng hai gc i din bng 180
0
(b nhau).
- Chng minh hai nh cng nhn mt on thng di mt gc bng nhau.
- Chng minh tng ca gc ngoi ti mt nh vi gc trong i din b nhau.
Hay din t l: Gc ngoi bng gc i trong.
- Nu MA.MB = MC.MD hoc NA.ND = NC.NB th t gic ABCD ni tip.
(Trong : M = AB CD; N = AD BC)
www.VNMATH.com
.:: CHUYN N THI VO LP 10 ::.
Bin son: Trn Trung Chnh 44
- Nu PA.PC = PB.PD th t gic ABCD ni tip. (Trong P= AC BD)
- Chng minh t gic l hnh thang cn; hnh ch nht; hnh vung;
Nu cn chng minh cho nhiu im cng thuc mt ng trn ta c th chng minh ln lt 4
im mt lc. Song cn ch tnh cht Qua 3 im khng thng hng xc nh duy nht mt ng
trn
2. Bi tp p dng:
Bi tp 1: Cho AABC, BD, CE l hai ng cao. Chng minh: T gic BCDE v AEHD ni tip.
Chng minh
Xt t gic BCDE, c:

0
BDC BEC 90 = = (V BD, CE l hai ng cao)

A l gc chung.
Hai nh E, D cng nhn cnh BC vi mt gc bng 90
0
.
T gic BCDE ni tip.
Xt t gic AEHD, c:

0
AEH 90 = (EC l ng cao)

0
ADH 90 = (BD l ng cao)


0
AEH ADH 180 + =
T gic AEHD ni tip.
Bi tp 2: Cho hnh thang cn ABCD (AB > CD, AB // CD) ni tip trong ng trn (O). K cc
tip tuyn vi ng trn (O) ti A v D chng ct nhau E. Gi M l giao im ca hai ng
cho AC v BD. Chng minh t gic AEDM ni tip c trong mt ng trn.
Chng minh
Ta c:

1
EAC =
2
s

AC (gc to bi tia tip tuyn AE v dy AC ca


ng trn (O))
Tng t:

1
xDB=
2
s

DB (Dx l tia i ca tia tip tuyn DE)


M AC = BD (do ABCD l hnh thang cn) nn

AC= BD.
Do

EAC= xDB.
Vy t gic AEDM ni tip c trong mt ng trn.
Bi tp 3: Cho na ng trn (O) ng knh AB = 2R, dy cung AC. Gi M l im chnh gia
cung AC. ng thng k t C song song vi BM ct tia AM K v ct tia OM D. OD ct AC ti
H. Chng minh t gic CKMH ni tip.
Chng minh
Ta c:

0
AMB= 90 (gc ni tip chn na ng trn ng knh AB)
AM MB .
M CD // BM (gi thit) nn AM CD .
Vy

0
MKC= 90 .
Ta c:

AM= CM(gi thit)
OM AC

0
MHC 90 = .
H
C
D
E
B
A
E
M
O
A B
C D
H
K
D
M
C
B A
O
.:: CHUYN N THI VO LP 10 ::.
Bin son: Trn Trung Chnh 45
T gic CKMH c

0
MKC+MHC=180 nn ni tip c ng trn.
Bi tp 4: Cho na ng trn tm O ng knh AB. T im M trn tip tuyn Ax ca na
ng trn v tip tuyn th hai MC (C l tip im). ng thng MB ct na ng trn (O) ti
Q. Gi giao im ca MO v AC l I. Chng minh rng: T gic AMQI ni tip.
Chng minh
Ta c:
MA = MC (tnh cht hai tp tuyn ct nhau)
OA = OC (bn knh ng trn (O))
Do : MO AC

0
MIA 90 = .

0
AQB 90 = (gc ni tip chn na ng trn (O))

0
MQA 90 = .
Hai nh I, Q cng nhn cnh AM vi mt gc bng
90
0

T gic AMQI ni tip c trong ng trn.
Bi tp 5: Cho ng trn (O) ng knh AB. Trn tia
AB ly im D nm ngoi on AB v k tip tuyn DC vi ng trn (O) (C l tip im). Gi E
l chn ng vung gc h t A xung ng thng CD v F l chn ng vung gc h t D
xung ng thng AC.
Chng minh: T gic EFDA ni tip.
Chng minh
Ta c:

0
AED= 90 (V AE CD ti E)

0
AFD= 90 (V DF AC ti F)
Hai nh E, F cng nhn cnh AD vi mt gc bng 90
0
.
T gic EFDA ni tip.
Bi tp 6: S dng tnh cht ca nh l Pltm :
nh l Ptolemy hay ng thc Ptolemy l ng thc
trong hnh hc Euclid miu t quan h gia di bn cnh v
hai ng cho ca mt t gic ni tip ng trn. nh l ny mang tn nh ton hc v thin vn
hc ngi Hy Lp c i Ptolemy (Claudius Ptolemaeus).
AC
AD
BD
CD
BC
AB
D
C
B
A

Nu A, B, C, v D l 4 nh ca t gic ni tip ng trn th:
AC. BD = AB. CD + BC + AD
(vi du gch ngang k hiu di ca cc cnh)
nh l ny cng c th pht biu thnh nh l thun v o:
nh l thun: Nu mt t gic ni tip trong mt ng trn th tch ca hai ng cho bng tng
cc tch ca cc cp cnh i din.
nh l o: Nu mt t gic tha mn iu kin tng cc tch ca cc cp cnh i din bng tch
ca hai ng cho th t gic ni tip mt ng trn.
Chng minh
I
Q C
M
x
B A
O
F
E
C
O D B
A
www.VNMATH.com
.:: CHUYN N THI VO LP 10 ::.
Bin son: Trn Trung Chnh 46
A
B
C
D
K
A
B
C
D
K
K
D
C
B
A

Gi ABCD l t gic ni tip ng trn.
Trn cung nh BC, ta c cc gc ni tip:

BAC BDC = , v trn cung AB,

ADB ACB = .
Ly 1 im K trn AC sao cho

ABK CBD = ;
T

ABK CBK ABC CBD ABD + = = + .
Suy ra:

CBK ABD = .
Do vy tam gic ABK DBC, v tng t c ABD KBC.
Suy ra:
AK CD
=
AB BD
, v
CK DA
=
BC BD
;
T AK.BD = AB.CD, v CK.BD = BC.DA;
Cng cc v ca 2 ng thc trn: AK.BD + CK.BD = AB.CD + BC.DA;
Hay: (AK+CK).BD = AB.CD + BC.DA;
M AK+CK = AC, nn ACBD = AB.CD + BC.DA. (iu phi chng minh)

3. Bi tp t luyn:
Bi tp 1: Cho ng trn (O) ng knh AB. M l mt im trn tip tuyn xBy. AM ct (O) ti
C; ly D e BM; ni AD ct (O) ti I. Chng minh: T gic CIDM ni tip.
Bi tp 2: Cho AABC vung ti A c AB = 5cm v AC = 5 3 cm. ng cao AH (H e BC).
ng trn (H; HA) ct AB ti D v AC ti E. Chng minh: T gic CEBD ni tip.
Bi tp 3: Cho ng trn (O) ng knh AB. T A v B v Ax AB v By BA. V tip tuyn
xMy (tip im M) ct Ax ti C v By ti D. OC ct AM ti I v OD ct BM ti K. Chng minh:
T gic CIKD ni tip.
Bi tp 4: Cho ng trn (O) ng knh AB, v bn knh OC AB. T B v tip tuyn Bx. Gi
M l trung im OC, AM ko di ct ng trn ti E v Bx ti I. Tip tuyn t E ct Bx ti D.
Chng minh: T gic MODE ni tip.
Bi tp 5: Cho tam gic ABC (

0
BAC< 45 ) ni tip trong na ng trn tm O ng knh AB.
Dng tip tuyn vi ng trn (O) ti C v gi H l chn ng vung gc k t A n tip tuyn
. AH ct ng trn (O) ti M (M = A). ng vung gc vi AC k t M ct AC ti K. Chng
minh t gic MKCH ni tip.
Bi tp 6: Cho tam gic ABC vung A, ng cao AH. ng trn tm O ng knh AH ct cc
cnh AB, AC ln lt ti M v N ( A = M v N). Chng minh:
a)

AHN= ACB
b) T gic BMNC ni tip.
Bi tp 7: Cho ng trn (O;R) ng knh AB. Gi C l im bt k thuc ng trn (C = A
v B). Gi M, N ln lt l im chnh gia ca cc cung nh AC v BC. Cc ng thng BN v
AC ct nhau ti I, cc dy cung AN v BC ct nhau P. Chng minh t gic ICPN ni tip. Xc
nh tm K ca ng trn ngoi tip t gic .
.:: CHUYN N THI VO LP 10 ::.
Bin son: Trn Trung Chnh 47
Bi tp 8: Cho ng trn (O; R), ng knh AB . Trn tip tuyn k t A ca ng trn ny ly
im C sao cho AC = AB. T C k tip tuyn th hai CD ca ng trn (O; R), vi D l tip im.
Chng minh rng t gic ACDO ni tip.
Bi tp 9: Cho ng trn (O) ng knh AB bng 6cm . Gi H lim nm gia A v B. Qua H
v ng thng vung gc vi AB, ng thng ny ct ng trn (O) ti C v D. Hai ng thng
BC v DA ct nhau ti M. T M h ng vung gc MN vi ng thng AB ( N thuc thng
AB). Chng minh MNAC l t gic ni tip.

CMI OI 11
CAC OIONG TMANG OONG QIY

1. Kin thc c bn:
Phng php 1: p dng tnh cht cc ng ng quy trong tam gic.
Phng php 2: Chng minh cc ng thng cng i qua mt im: Ta ch ra hai ng thng ct
nhau ti mt im v chng minh ng thng cng i qua im .
Phng php 3: Dng nh l o ca nh l Talet.
Phng php 4:
nh l Lyness m rng(B Sawayama): Cho tam gic ABC ni tip ng trn (O). M e BC.
Mt ng trn (O') tip xc vi hai cnh MA v MC ti E v F ng thi tip xc vi c ng
trn (O) ti K. Khi ta c tm ng trn ni tip ca tam gic ABC nm trn ng thng EF.
nh l Pascal: Cho 6 im A, B, C, D, E, F cng thuc mt ng trn. Khi cc giao im ca
cc cp cnh AB v DE, BC v EF, CD v FA thng hng.
Phng php 6:
nh l CEVA: Cho tam gic ABC. Ly cc im D, E v F ln lt nm trn cc cnh BC, AC, AB.
O
F
E
D
C
B
A

nh l pht biu rng cc ng thng AD, BE v CF l nhng ng thng ng quy khi v ch
khi:
AF BD CE
. . =1
FB DC EA


2. Bi tp p dng:
Bi tp 1: Cho tam gic ABC dng tam gic u MAB, NBC, PAC thuc min ngoi tam gic
ABC. Chng minh rng:
a) MC = NA = PB
b)
( ) ( ) ( )
0
AM.MC = MC, BP = BP, NA = 60
c) MC, NA, PB ng quy
Chng minh
a) Xt AABN v AMBC, c:
AB = MB;
BC = BN (cc cnh ca tam gic u)
www.VNMATH.com
.:: CHUYN N THI VO LP 10 ::.
Bin son: Trn Trung Chnh 48

ABN= MBC (cng bng

0
60 +ABC )
AABN = AMBC (c.g.c)
AN = MC (*)
Tng t: AABP = AAMC (c.g.c)
AB = AM;
BC = BN (cc cnh ca tam gic u)

BAP = MAC (cng bng

0
60 +BAC )
BP = MC (**)
T (*) v (**) ta c: AN = MC = BP (pcm).
b)
Trong AAPC, c:

0
1 1 2 2 A +C +P +P =180 m

1 1 P = C
Trong APCK, c:

0
2 2 1 2 C +C +P K =180 +


( )

0 0 0
2 2 2 1 60 C P K 180 K 60 + + + = = (1)
Tng t: AABN = AMBC


1 3
N = C m

0
1 2
N + N = 60




0
2 3
N +C = 60 m

0
4
C = 60
ANKC c

0
2 3 4 3
N +C +C +K =180

0
3
K = 60 (2)
Tng t: AACN = APCB


2 2
P = A m

0
2
1
P +P = 60


0
1 2
P +A = 60 m

0
1
A = 60
Trong AAKP, c:

0
1
K = 60 (3)
T (1), (2), (3) ta c iu phi chng minh
c) Gi s MC BP = K, ta chng minh cho A, K, N thng hng.
Theo chng minh trn ta c:

0 0 0 0
2 3 1 1 2 3
K 60 , K 60 , K 60 K K K 180 = = = + + =
A,K,N thng hng
Vy AN, MC, BP ng quy (pcm)
Bi tp 2: Cho hnh bnh hnh ABCD. Trn AB v CD ly 2 im E v F sao cho AE = CF. Trn
AD v BC ly H v G sao cho DH = BG.
a) Chng minh: T gic EGFH l hnh bnh hnh
b) Chng minh: AC, BD, EF, GH ct nhau ti 1 im.
Chng minh
a) Xt ADHF v ABGE, ta c:
DH = BG

HDF GBE = (V ABCD l hnh bnh hnh)
DF = BE (V AE = CF)
ADHF = ABGE
HF = EG. (1)
Mt khc, ta c:

DHG BGH = v

DHF BGE FCG EGH = = (2)
T (1), (2) suy ra: T gic EGFH l hnh bnh hnh.
b) (Theo cu a)
T gic EGFH l hnh bnh hnh.
2
1
4
3
2 1
3
2
1
2
1
K
P
N
M
C B
A
I G
H
F
E
D
C
B
A
.:: CHUYN N THI VO LP 10 ::.
Bin son: Trn Trung Chnh 49
Gi I l giao im ca 2 ng cho HG v EF (ca hnh bnh hnh EGFH)
Ta li c: T gic AGCH l hnh bnh hnh (AH // CG v AH = CG)
Giao ca 2 ng cho HG v AC l I (I trung im HG)
Tng t, ta c: Hnh bnh hnh HBGD c giao im ca 2 ng cho l HG v BD ti I (I l trung
im HG)
Suy ra: HG, EF, AC, BD ct nhau ti im I (cng l im duy nht).
Bi tp 3: Cho hai ng thng d
1
v d
2
ct nhau ti O. Trn d
1
ln lt ly ba im phn bit A, B,
C khc O sao cho OA = AB = BC. Trn d
2
ln lt ly ba im E, M, N khc O sao cho OE = OM =
MN. Chng minh rng ba ng thng AE, BN v CM ng quy.
Chng minh
Gi D l giao im ca BN v CM.
Qua M k ng thng song song vi OC ct BC ti F.
Qua O k ng thng song song vi BN ct MF ti G.
Xt AFBO v AOGF, ta c:


BOF GFO = (so le trong)
OF l cnh chung


BFO GOF = (so le trong)
AFBO = AOGF (g-c-g).
FG = BO. (1)
Xt ANFM v AOGM, ta c:


GOM FNM =
MO=MN


OMG NMF = (i nh)
ANFM = AOGM.
MF = MG. (2)
T (1) v (2), suy ra: MF = OA = AB = BC.
S dng kt qu va tm c ny kt hp:


DCB DMF = (so le trong) v

DBC DFM = (so le trong)
Suy ra: ADBC = ADFM (g-c-g).
Do : DC = DM
hay D l trung im ca CM. (3)
Xt ACEM, ta c:
CO l trung tuyn ng vi cnh ME (do OE=OM)
CA =
2
3
CO
A l trng tm ca ACEM.
Suy ra: AE i qua trung im ca cnh CM. (4)
T (3) v (4), ta suy ra: AE i qua D.
Vy BN,CM v AE ng quy ti D.
Bi tp 4: Cho AABC, cc ng cao AD, BE, CF ca tam gic ng quy ti H. Gi I l trung im
ca HC.
a) Chng minh BCEF l t gic ni tip.
b) Chng minh rng H l tm ng trn ni tip ADEF v DIEF l t gic ni tip 1 ng trn.
c) V pha ngoi AABC dng cc AABM v ACAN sao cho chng l cc tam gic vung cn ti cc
nh B v C tng ng. Chng minh rng cc ng thng AD, BN, CM ng quy.
Chng minh
a) HS t lm.
b) Ta d dng chng minh c cc t gic AEHF, AEDB ni tip trong ng trn.
E
G
N
M
F
D
O
A
B
C
www.VNMATH.com
.:: CHUYN N THI VO LP 10 ::.
Bin son: Trn Trung Chnh 50
Khi , ta c:

FAH FEH = (cng chn

FH)
v

FAH BAD =

BAD BED = (cng chn

BD)
v

BED HED =


FEH HED =
HE l tia phn gic ca

FED.
Tng t, ta c:
HF l tia phn gic ca

EFD.
HD l tia phn gic ca

EDF.
H l giao im ca 3 ng phn gic
trong ca ADEF.
Vy H l tm ca ng trong ni tip ADEF.
* Theo chng minh trn, ta c:

FED 2FAD = v

FAD FCD = (HS t chng minh t gic ACDF ni tip)

HID 2FCD = (gc ngoi bng tng ca 2 gc trong khng k)


FED 2FAD 2FCD HID FID = = = =
Hay

FED FID =
T gic EIDF ni tip.
c) Trn tia i tia AD, ly T sao cho AT = BC.


0
MBC 90 ABC TAB = + =
AMBC = ABAT (c - g - c)


BTD BCM =
CM TB
Tng t, ta c: BN TC.
M TD BC
Vy TD, CM, BN ng quy (3 ng cao ca ATBC)
Bi tp 5: Cho t gic ABCD, AD, BC khng song song, ni tip ng trn (O). P l giao im
ca AC v BD. ng trn (O
1
) tip xc vi cc on PA, PB v tip xc trong vi (O) ti E.
ng trn (O
2
) tip xc vi cc on PC, PD v tip xc trong vi (O) ti F. Chng minh rng AD,
BC, EF ng quy.
Chng minh
Gi s (O
2
) tip xc PB, PC ti X, Y v tip
xc (O) ti F.
Theo b Sawayama (nh l Lyness m rng) ta
c XY i qua H, K (vi H,K l tm ni tip cc AADC,
ABDC.
Gi Z, T l giao im ca HK trn AD, BC. Gi M, N,
P, Q l trung im cc cung AD, BD, AC, BC ca (O).
V (O
2
) tip xc AC, BD nn F, X, N v F, Y, P thng
hng.
Ta s chng minh: M, Z, F thng hng.
Tht vy: Gi Z l giao ca FM v AD.
AN giao BM ti S. Gi R l trung im cung CD.
Theo nh l Pascan cho lc gic MFNADB ta c S, Z,
X thng hng.
T
N
M
A
H
I
F
E
D C B
K
R
Z
H
S
Q
M
T
P
N
Y
E
X
O
1
F
O
2
P
D C
B
A
O
.:: CHUYN N THI VO LP 10 ::.
Bin son: Trn Trung Chnh 51
Tip tc vi lc gic NARBMC ta c H, K, S thng hng.
M H, K, X thng hng, nn ta c Z, X, H, K thng hng hay Z trng Z.
Tng t, ta c: F, T, Q thng hng.
Gi (O
3
) l ng trn tip xc AD, BC v tip xc (O) ti cung nh DC.
Ta s chng minh (O
3
) l (ZFT).
Tht vy, gi Z", T" l tip im trn AD, BC ca (O
3
) th theo b Sawayama, ta cng c Z", T",
H, K thng hng hay Z", T" trng Z, T.
M MZ v NT ct nhau ti F nn ta c ngay ZFT chnh l (O
3
).
T , ta quy bi ton v pht biu n gin hn nh sau: (O
3
) tip xc AD, BC v tip xc cung
nh CD ti F.
Tng t c E.
Khi AD, BC, EF ng quy.
Bi tp 6: Chng minh da vo nh l CEVA.
nh l CEVA: Cho tam gic ABC. Ly cc im D, E v F ln lt nm trn cc cnh BC, AC,
AB.
nh l pht biu rng cc ng thng AD, BE v CF l nhng
ng thng ng quy khi v ch khi:

AF BD CE
. . =1
FB DC EA

Chng minh
Gi s AD, BE v CF ng qui ti mt im O no (trong
hay ngoi tam gic). Do ABOD v ACOD c chung chiu cao
( di ca ng cao), ta c:
BOD BD
=
COD DC

Tng t,
BAD BD
=
CAD DC

Ta suy ra
BAD BOD ABO
BD
DC CAD COD CAO

= =


Tng t,
BCO
CE
,
EA ABO
=
v
CAO
AF
.
FB BCO
=
Nhn ba ng thc trn cho ta:
AF BD CE
. . 1.
FB DC EA
=
(iu phi chng minh).
Ngc li, gi s rng ta c nhng im D, E v F tha mn ng thc. Gi giao im
ca AD v BE l O, v gi giao im ca CO v AB l F'. Theo chng minh trn,
AF' BD CE
. . 1.
F'B DC EA
=
Kt hp vi ng thc trn, ta nhn c:
O
F
E
D
C
B
A
www.VNMATH.com
.:: CHUYN N THI VO LP 10 ::.
Bin son: Trn Trung Chnh 52
AF' AF
.
F'B FB
=
Thm 1 vo mi v v ch rng AF'' + F''B = AF + FB = AB, ta c:
AB AB
.
F'B FB
=
Do F''B = FB, vy F v F'' trng nhau . V vy AD, BE v CF = CF'' ng qui ti O, v nh l
c chng minh (l ng theo c hai chiu).
3. Bi tp t luyn:
Bi tp 1: Cho tam gic ABC dng cc tam gic u MAB, NBC, PAC thuc min ngoi tam gic
ABC. Chng minh MC, NA, PB ng quy.
Bi tp 2: Cho tam gic ABC dng cc tam gic u MAB, NBC, PAC v c tm ln lt l O
1
, O
2
,
O
3
. Chng minh cc ng trn ngoi tip 3 tam gic u trn u ng quy ti mt im.
Bi tp 3: Gi A', B', C' l tip im ca ng trn ni tip AABC vi cc cnh BC, CA, AB.
Chng minh rng: AA', BB', CC' ng quy.
Hng dn
Chng minh
A'B B'C C'A
. . =1
A'C B'A C'B
AA', BB', CC' ng quy
Bi tp 4: Cho hnh thang ABCD (AB > CD). Gi E l giao im hai cnh bn AD v BC; F l
trung im ca AB. Chng minh rng: AC, BD, CF ng quy.
Bi tp 5: Cho tam gic nhn ABC. Cc ng cao AH, BK, CL ct nhau ti I. Gi D, E, F ln lt
l trung im ca BC, CA, AB. Gi P, Q, R ln lt l trung im ca IA, IB, IC. Chng minh PD,
QE, RF ng quy. Gi J l im ng quy, chng minh I l trung im ca mi ng.
Hng dn
Chng minh PEDQ, PRDF l hnh ch nht
PD, QE, RF l ng cho ca 2 hnh ch nht
iu phi chng minh.
Bi tp 6: Cho AABC ni tip ng trn (O) v c H l trc tm. Gi A', B', C' l im i xng
ca H qua BC, CA, AB. Qua H, v ng thng d bt k. Chng minh rng: Cc ng thng i
xng ca d qua cc cnh ca AABC ng quy ti mt im trn (O).
Hng dn
Gi d
1
, d
2
, d
3
l cc ng thng i xng ca d qua cc cnh ca AABC.
Gi I l giao ca d
1
v d
2

Chng minh t gic A'B'C'I l t gic ni tip. Suy ra A'B'C'I l ni tip (O).
Chng minh I thuc d
3
.

CMI OI 12
BA OIIM TMANG MANG

1. Kin thc c bn:
Phng php 1:
Tin clit: Qua mt im A nm ngoi ng thng a k c duy nht mt ng thng song
song vi a.
H qu: Qua mt im A nm ngoi ng thng a k c duy nht mt ng thng vung gc
vi a.
Phng php 2: Chng minh qua mt im c hai ng thng vung gc vi 1 ng thng cho
trc ti im .
Phng php 3: Chng minh tng hai gc bng 180 (s dng t gic ni tip, cc gc bng
nhau...).
.:: CHUYN N THI VO LP 10 ::.
Bin son: Trn Trung Chnh 53
Phng php 4: S dng tnh cht ng quy ca ba ng cao, phn gic, trung trc, trung tuyn...
Phng php 5: Chng minh im AM + MB = AB th A thuc on thng BC. Suy ra A, B, C
thng hng.
Phng php 7: Dng tnh cht ng trung trc: Chng minh cc im cch u hai u mt ca
mt on thng cho trc th u nm trn mt ng thng.
Phng php 8: Dng tnh cht tia phn gic: Chng minh 3 im cng cch u hai cnh ca
mt gc.
Phng php 9: S dng tnh cht ng cho ca cc t gic c bit.
Phng php 10: S dng tnh cht ng knh v dy cung ca ng trn.
Phng php 11: S dng tnh cht hai ng trn tip xc nhau.
on thng ni hai tm ca hai ng trong v tip tuyn chung th vung gc vi nhau.
2. Bi tp p dng:
Bi tp 1: Cho AABC vi hai trung tuyn BD v CE. Gi M v N theo th t thuc cc tia i ca
cc tia EC v DB sao cho EC = EM v DB = DN. Chng minh rng A, M, N thng hng.
Gii
T gic AMBC c:
EA = EB,
EM = EC (gt)
Nn l hnh bnh hnh.
Suy ra: AM // BC. (1)
Chng minh tng t, ta c:
AN // BC. (2)
T (1) v (2) suy ra ba im A, M, N
thng hng (tin clit).
Bi tp 2: Cho hnh ch nht ABCD (AB < CD), c O l giao im ca hai ng cho. Trn tia
i ca tia CD ly im E sao cho CE = CD. Gi F l hnh chiu ca ca D trn BE; I l giao im
ca AB v CF; K l giao im ca AF v BC. Chng minh rng ba im O, K, I thng hng.
Gii
ABCD l hnh ch nht nn:
AB = CD, AC = BD v OA = OB = OC = OD.
Ta c CB AI (v ABCD l hnh ch nht)
CB l ng cao ca ACAI. (1)
AFBD vung ti F (v F l hnh chiu ca D ln BE) c:
FO l trung tuyn ng vi cnh huyn BD nn OF =
1
2
BD
OF =
1
2
AC.
AFAC c FO l ng trung tuyn ng vi cnh AC.
M FO =
1
2
AC nn AFAC vung ti F.
Suy ra AF CI hay AF l ng cao ca ACAI. (2)
K l giao im ca AF v CB nn t (1) v (2), suy ra K l trc tm ca ACAI.
Do IK AC. (3)
Mt khc, t gic ABEC c:
AB = CE (cng bng CD) v AB // CE (v AB // CD)
nn l hnh bnh hnh
BE // AC BF //AC ABFC l hnh thang.
Li c AFDE vung ti F, FC l trung tuyn ng vi cnh DE (v CD = CE) nn
CF = CD CF = AB (v AB = CD).
N
M
E
D
C
A
B
K
I
F
E
O
D
C
B
A
www.VNMATH.com
.:: CHUYN N THI VO LP 10 ::.
Bin son: Trn Trung Chnh 54
Suy ra ABAC = AFCA (cnh huyn cnh gc vung)
AF = BC.
Hnh thang ABFC c hai ng cho AF v BC bng nhau nn l hnh thang cn.
Suy ra:

IAC ICA = AIAC cn ti I
IO l trung tuyn ng thi l ng cao.
Hay IO AC. (4)
T (3) v (4) suy ra I, K, O thng hng (pcm).
Bi tp 3: Cho t gic ABCD. Gi M, I v N theo th t l trung im ca AB, AC v CD. Chng
minh rng nu
AD BC
MN
2
+
= th M, I, N thng hng v ABCD tr thnh hnh thang.
Gii
Gi s:
AD BC
MN
2
+
= (1)
V MA = MB, IA = IC nn MI l ng trung bnh ca tam
gic ABC.
Suy ra: MI // BC v MI =
1
2
BC.
Chng minh tng t ta c: IN // AD v IN =
1
2
AD.
M
AD BC 1 1
MN BC AD
2 2 2
+
= = + hay MN = MI + IN.
T suy ra I nm gia M v N, hay M, I, N thng hng.
Lc , ta c: BC // AD v cng song song vi MN.
Do ABCD tr thnh hnh thang.
Vy nu
AD BC
MN
2
+
= th M, I, N thng hng v ABCD tr thnh
hnh thang.
Bi tp 4: ng trn tm O v ng trn tm O ct nhau ti A v B.
Gi C, D ln lt i xng vi B qua O v O. Chng minh rng C, A,
D thng hng.
Gii
V C i xng vi B qua O nn O l trung im ca BC. Suy ra BC l
ng knh ca (O).
Ta c OA = OB = OC =
1
2
BC nn tam gic ABC vung ti A

0
BAC 90 = .
Chng minh tng t ta c:

0
BAD 90 = .
Do :

0
CAD BAC BAD 180 = + =
C, A, D thng hng.
Bi tp 5: Cho hnh bnh hnh ABCD. Gi O l giao im ca hai ng cho; E l im i xng
ca A qua B; F l giao im ca BC v ED; G l giao im ca BC v OE; H l giao im ca EC
v OF. Chng minh rng A, G, H thng hng.
Gii
N
I
M
D
C
B
A
D
C
B
A
O'
O
.:: CHUYN N THI VO LP 10 ::.
Bin son: Trn Trung Chnh 55
H
G
F
E
O
D
C
B A

V O l giao im ca hai ng cho AC v BD nn OA = OC
suy ra EO l trung tuyn ca AEAC.
E i xng vi A qua B nn B l trung im ca EA.
Suy ra CB l trung tuyn ca AEAC.
G l giao im ca CB v EO nn G l trng tm ca AEAC. (1)
Mt khc, ABCD l hnh bnh hnh nn CD // AB, CD = AB,
M B l trung im ca AE nn suy ra CD // BE, CD = BE.
Do t gic BECD l hnh bnh hnh.
T F l trung im ca hai ng cho ED v BC ca hnh bnh hnh BECD.
Ta c OF l ng trung bnh ca ACAB nn OF // AB OH // AE HE = HC.
Do AH l trung tuyn ca AEAC. (2)
T (1) v (2) suy ra A, G, H thng hng (pcm).
Bi tp 6: Cho hnh bnh hnh ABCD. Trn ng cho BD ly hai im E v F sao cho BE = DF.
K EH AB, FK CD (H e AB, K e CD). Gi O l trung im ca EF. Chng minh rng ba im
H, O, K thng hng.
Gii
V EH AB, FK CD v AB // CD
nn EH // FK (1)
Xt AHBE v AKDF c BE = DF,

0
KDF HBE, DKF BHE 90 = = =
AHBE = AKDF (cnh huyn gc nhn)
HE = KF (2)
T (1) v (2) suy ra HEKF l hnh bnh hnh
Trung im ca EF cng l trung im ca HK.
Vy E, H, K thng hng (pcm).
Bi tp 7: Cho t gic ABCD. Cc ng thng AB v CD ct nhau ti M, cc ng thng AD v
BC ct nhau ti N. Gi I, J, K theo th t l trung im ca BD, AC, MN. Chng minh rng I, J, K
thng hng.
Gii
Gi K l giao im ca IJ vi MN.
Gi E, F ln lt l chn ng vung gc k t N, M ti ng
thng IJ.
D thy M, N nm v hai pha ca IJ.
Ta c :
NIJ NDC NDI NJC NCI CID
S S S S S S =
NIJ NDC NBD NAC AIC CBD
1 1 1 1
S S S S S S
2 2 2 2
=
( )
NIJ NDC NAB ABD ABC ADC ADIC CBD
1 1 1 1
S S S S S S S S
2 2 2 2
=

( ) ( )
NIJ ABCD ABD BCD ABCD ABC ADC ABCD
1 1 1 1
S S S S S S S S
2 4 2 4
= + + =
O
K
H
F
E
D
C
B A
K
J
I
N
M
D C
A
B
www.VNMATH.com
.:: CHUYN N THI VO LP 10 ::.
Bin son: Trn Trung Chnh 56

Chng minh tng t ta c:
MIJ ABCD
1
S S
4
= .
Do S
NIJ
= S
MIJ
hay
NKJ MKJ
1 1
NF.IJ= ME.IJ ME NF S S
2 2
= = .
Hai ANKJ v AMKJ c chung chiu cao h t J.
Suy ra: NK = MK.
M MK = NK (gt) nn K K.
Vy ba im I, J, K thng hng.
Bi tp 8: Ba im A, B, C cng thuc ng thng a, im O khng thuc a. Chng minh rng nu
ba im M, N, P tha mn h thc
OM ON OP
OA OB OC
= = th M, N, P thng hng.
Gii
Tht vy, theo nh l Talet o th t
OM ON
OA OB
= .
Suy ra: MN // AB.
Tng t MP // AC.
Nhng A, B, C thng hng nn M, N, P thng hng (tin
'clit).
Bi tp 9 (B hnh thang): Trong hnh thang c hai y khng bng nhau. Chng minh rng
giao im ca hai ng thng cha hai cnh bn, giao im ca hai ng cho v trung im ca
hai y nm trn cng mt ng thng.
Gii
Gi s hnh thang cho l ABCD (AB // CD, AB < CD) c I, J
tng ng l giao im ca hai ng thng cha hai cnh v ca
hai ng cho.
Gi M v N ln lt l giao im ca IJ vi AB v CD.
Do AB // CD nn p dng h qu ca nh l Talet ta c:
AM BM IM
DN CN IN
| |
= =
|
\ .
v
AM BM JM
CN DN JN
| |
= =
|
\ .
hay
AM BM IM
DN CN IN
| |
= =
|
\ .
.
Bi tp 10: Trn mt phng cho n im (n > 3) v bt k ng thng no i qua hai trong nhng
im u cha mt im cho. Chng minh rng tt c cc im cho cng nm trn mt
ng thng.
Gii
Gi s tt c cc im khng cng nm trn mt ng thng.
Qua mi cp im cho v mt ng thng (c mt s hu
hn ng ny) v chn khong cch khc 0 t cc im cho
n cc ng thng ny.
Gi s khong cch t im A n ng thng BC, trong
A, B, C l cc im cho l khong cch nh nht.
Trn ng thng BC cn c mt im D no .
T A k AQ vung gc vi BC ti Q.
Hai trong cc im B, C, D nm cng mt pha i vi im Q,
chng hn C v D nh hnh v, khi ta c CQ < DQ.
H CH vung gc vi AD ti H.
D thy CH < AQ. iu ny mu thun vi vic chn im A v ng thng BC.
a
P
N
M
O
C B A
N
M
J
I
D C
B A
H
Q D
A
C B
.:: CHUYN N THI VO LP 10 ::.
Bin son: Trn Trung Chnh 57
T ta c iu phi chng minh.
Bi tp 11 (nh l MENELAUS): L mt nh l v cc tam gic trong hnh hc phng.
H
F
G
C
B
A

Cho tam gic ABC. Cc im H, F, G ln lt n m trn AB, BC, CA.
Khi o

: G, H, F th ng ha

ng khi va

chi

khi :
AH BF CG
. . = -1.
HB FC GA

Chng minh
Ph n thu n:
S

du ng nh l sin trong ca

c tam gia

c AGH, BFH, CGF, ta c:

AH sinAGH BF sinBHF CG sinGFC


= ; = ; =
GA HB FC
sinAHG sinHFB sinCGF
.
(v

i lu y

r ng

sinAGH= sinCGF; sinAHG = sinBHF; sinHFB= sinGFC)
Nhn t

ng v ta c iu phi chng minh .


Ph n a

o:
Gi F' = GH BC. Hon ton tng t ta c c:
( )
AH BF' CG AH BF CG
. . = . . = -1 .
HB F'C GA HB FC GA

Hay
BF' BF
=
F'C FC
, suy ra F F'.
3. Bi tp t luyn:
Bi tp 1: Cho ABC, ng cao AH. Trn na mt phng b AB khng cha im C dng hnh
vung ABDE ; trn na mt phng b AC khng cha im B dng hnh vung ACMN. Dng hnh
bnh hnh AEIG. Gi K l giao im ca CD v BM. Chng minh rng bn im I, A, K, H thng
hng.
Bi tp 2: Trn cc cnh AB, BC, CD, DA ca hnh vung ABCD ta ly ln lt cc im M, N, P,
Q sao cho AM = BN = CP = DQ. Gi O l giao im ca hai ng cho. Chng minh rng M, O, P
thng hng.
Bi tp 3: Cho gc vung xAy. Mt im B c nh trn Ax, cn mt im C chuyn ng trn Ay.
ng trn ni tip tam gic ABC tip xc vi cc cnh AB v AC ln lt M v N. Chng minh
rng MN lun i qua mt im c nh khi im C chuyn ng trn Ay.
Bi tp 4: Trong hnh vung ABCD ly im E sao cho

0
EBC ECB 15 = = . Trn na mt phng b
CD khng cha im E v tam gic u CDF. Chng minh rng B, E, F thng hng.
Bi tp 5: Cho hnh thang ABCD, y ln AB. ng thng k t C song song vi AD ct BD v
AB ln lt ti E v F. ng thng k t D song song vi BC ct AC v AB ln lt ti P v Q.
Chng minh rng bn im M, N, P, Q thng hng.
Bi tp 6:Trn mt ng thng ly bn im theo th t l A, E, F, B. Dng cc hnh vung
ABCD, EFGH sao cho chng nm cng mt na mt phng b l ng thng cho. Gi O l
giao im ca AG v BH. Chng minh rng :
a) C, O, E thng hng.
2. D, O, F thng hng.
www.VNMATH.com
.:: CHUYN N THI VO LP 10 ::.
Bin son: Trn Trung Chnh 58
Bi tp 7: Cho hnh bnh hnh ABCD. Trn cnh BC ly im E. Ly im F im i xng vi C
qua E. T im F k Fx v Fy ln lt song song vi AD v AB. Gi I l giao im ca Fx v AB ;
K l giao im ca FI v AD. Chng minh rng I, K, E thng hng.
Bi tp 8: Cho ABC vung ti A, cnh huyn BC = 2AB. Trn cnh AC ly im D sao cho

1
ABD ABC
3
= ; trn cnh AB ly im E sao cho

1
ACE ACB
3
= . Gi F l giao im ca BD v
CE; G v H theo th t l cc im i xng ca F qua cc cnh BC v AC. Chng minh rng:
a) Ba im H, D, G thng hng.
b) Tam gic EDF cn.
Bi tp 9: Cho gc vung xOy tam gic. M thuc Ox; A, B thuc Oy. ng thng i qua A v
vung gc vi AM ct ng thng i qua B v vung gc vi BM ti P. Gi H l giao im ca AP
vi MB; K l giao im ca AM vi BP; I, K, E ln lt l trung im ca MP, AB v KH. Chng
minh rng I, E, N thng hng.
Bi tp 10: Cho hnh vung EFGH. Mt gc vung xEy quay quanh nh E c cnh Ex ct FG v
GH theo th t ti M v N, cn cnh Ey ct cc ng FG v GH theo th t t P v Q. Gi I v K
theo th t l trung im ca PN v QM. Chng minh rng bn im F, H, K, I thng hng.
Bi tp 11: Cho t gic ABCD v mt im O nm bn trong t gic sao cho cc tam gic ABO,
BCO, CDO, DAO c din tch bng nhau. Chng minh rng hoc ba im A, O, C thng hng, hoc
ba im B, O, D thng hng.
Bi tp 12: Cho AABC c ba gc nhn, cc ng cao BD v CE. Gi I l im thuc on BC; H
l giao im ca BD v CE; N thuc on AH ; M thuc on DE. Chng minh rng M, I, N thng
hng.
Bi tp 13: Cho hnh vung EFGH. Mt gc vung Exy quay quanh nh E. Cnh Ex ct cc ng
thng FG v GH theo th t ti M v N; cnh Ey ct cc ng thng FG v GH theo th t P v
Q. Gi I v K theo th t l trung im ca PN v QM. Chng minh rng 4 im F, H, K, I thng
hng.
Bi tp 14: Cho

0
xOy 90 = . Ly im M thuc Ox, A v B cng thuc Oy. ng thng i qua A
v vung gc vi AM ct ng thng i qua B v vung gc vi BM ti P. Gi H l giao im ca
AP v MB; K l giao im ca AM v BP; I, E, N ln lt l trung im ca MP, AB v KH. Chng
minh rng I, E, N thng hng.
CMI OI 1J
OANG TMIC VA BAT OANG TMIC MNM MOC
1. Kin thc c bn:
Trong hnh hc THCS, chng ta hay gp cc bi ton chng minh cc ng thc v cc bt ng
thc lin quan n ba cnh ca, chu vi ca tam gic, bn knh ca ng trn ni tip r, bn knh
ng trn ngoi tip R, v mt s yu t trong ng trn.
Phng php:
- V tr ca hnh H trn min D sao cho biu thc f c gi tr ln nht ta phi chng t c:
Vi mi v tr ca hnh H trn min D th f m (m l hng s)
Xc nh v tr ca hnh H trn min D sao cho f = m
- V tr ca hnh H trn min D sao cho biu thc f c gi tr nh nht ta phi chng t c:
Vi mi v tr ca hnh H trn min D th f m (m l hng s)
Xc nh v tr ca hnh H trn min D f = m
Cc h thc thng gp:
(1) Din tch tam gic:
Vi a, b, c l di 3 cnh ca tam gic. p =
a b c
2
+ +
l na chu vi. h
a
, h
b
, h
c
l di 3 ng cao.
.:: CHUYN N THI VO LP 10 ::.
Bin son: Trn Trung Chnh 59
( ) ( )( )( )
a b c
a
1 1 1 1
S = a.h = b.h = c.h = bcsinA
2 2 2 2
1 1 abc
= acsinB= absinC = = p- a r = p p- a p- b p- c
2 2 4R

(2) di ng trung tuyn:
2 2 2
2
2b +2c - a
m =
4
a

(3) di ng phn gic:
A
2bc.cos
2
=
b +c
a
l


( )
( )
2
4bc
= p p- a
b +c
a
l

(4) di ng cao:
a b c
1 1 1 1
+ + =
h h h r

(5) Tnh din tch t gic:
(a) Tnh din tch hnh bnh hnh:
S = a.h
a

(a l di ng cho, h
a
l ng cao ng vi ng cho a)
(b) Tnh din tch hnh ch nht:
S = a.b
(Vi a v b l di hai cnh)
(c) Tnh din tch hnh thoi :
S = a.b
(vi a v b l di hai ng cho)
(d) Tnh din tch hnh vung:
S = a
2
.
(vi a l di cnh)
(6) nh l hm s sin:
a b c
= = = 2R
sinA sinB sinC

(7) Bn knh ng trn ni tip, bng tip:
( )
a
A A
r = p- a tan ; r = ptan
2 2

(8) Cng thc tnh di trung tuyn:

( )
2 2 2
2
a
2 b +c - a
m =
4

Mt s nh l h tr:
nh l 1: Gi R v r ln lt l bn knh ca ng trn ngoi tip v ng trn ngoi tip AABC,
d l khong cch gia tm ng trn ngoi tip v tm ng trn ngoi tip ca AABC.
Khi , ta lun c:
2Rr = R
2
d
2

nh l 2: Cho AABC. Nu

ABC> ACB th AC > AB v ngc li
nh l 3: Cho trc AABC v AABC c 2 cp cnh AB = AB v AC = AC. Ta c bt ng
thc

BAC> B'A'C' khi v ch khi BC > BC.
www.VNMATH.com
.:: CHUYN N THI VO LP 10 ::.
Bin son: Trn Trung Chnh 60
nh l 4: Trong nhng ng xin ni mt im M cho trc vi im N trn mt dng thng d
cho trc, ng xin no c hnh chiu di hn th di hn.
nh l 5: Trong nhng ng xin ni mt im M cho trc vi im N trn mt mt phng (P)
cho trc, ng xin no c hnh chiu di hn th di hn.
nh l 6: 2 AABC v AABC vung, c

0
A= A' = 90 v AB = AB.
Nu

ABC A' B' C' > th AC AC
nh l 7: Bn knh ca hai ng trn l R, r (R r), cn khong cch gia tm ca chng l d.
iu kin cn v hai ng trn ct nhau l:
R r s d s R + r
nh l 8: Cc s dng a, b, c l di 3 cnh ca mt tam gic khi v ch khi:
a + b > c, b + c > a v c + a > b
nh l 9: Cho tam gic ABC v mt im M bt k thuc min trong ca tam gic. Khi ta lun
c:
MB + MC < AB + AC
nh l 10: Trong tam gic ABC ng vi cnh di hn l ng cao, ng trung tuyn,ng phn
gic ngn hn
nh l 11: Trong tam gic ABC k hiu h
a
l di ng cao, l
a
l di ng phn gic, m
a
l
di ng trung tuyn xut pht t nh A th ta c bt ng thc:
m
a
l
a
h
a

nh l 12: ng trung tuyn AM ca tam gic ABC nh hn na tng cc cnh AB v AC cng
xut pht t mt nh A
nh l 13: Hnh trn ni tip l hnh trn ln nht c th cha trong nt tam gic.
nh l 14: Mt t gic li b cha trong mt t gic khc ( khng nht thit l li ) th chu vi ca t
gic b cha s nh hn chu vi ca t gic cha n bn trong
nh l 15: Trong na mt phng b chia ra bi ng thng i qua 2 im A v B c 2 ng gp
khc AC
1
C
2
C
k
B v AD
1
D
2
D
p
B sao cho 2 a gic AC
1
C
2
C
k
B v AD
1
D
2
D
p
B l 2 a gic
li. Nu a gic AC
1
C
2
C
k
B cha a gic AD
1
D
2
D
p
B bn trong n th ng gp khc
AC
1
C
2
C
k
B di hn ng gp khc AD
1
D
2
D
p
B
nh l 16: Mt a gic bt k c chu vi khng nh hn chu vi ca a gic to bi bao li ca n
nh l 17: Nu mt a gic li cha a gic li khc th chu vi ca a gic ngoi ln hn chu vi ca
a gic nm trong n
nh l 18: di on thng nm trong mt a gic li khng ln hn khong cch ln nht ni 2
nh ca n
nh l 19: Cho (O; r) v 1 im M bt k trong n.
Khi ta c:
R d MN R + d
Vi N l im bt k trn ng trn v d l khong cch t M ti tm ng trn
nh l 20: Cho (O; r) v im M bt k ngoi ng trn. Khi ta c:
d R MN d + R
nh l 21: Cho trc im M trong hnh trn tm O. Trong cc dy cung qua M, dy cung vung
gc vi MO c di nh nht
nh l 22: Gi P l giao im ca 2 ng trn (O
1
) v (O
2
).
Khi , ta c bt ng thc MN 2O
1
O
2
cho mi dy cung qua P. Du "=" xy ra
MN // O
1
O
2

nh l 23: Din tch tam gic ABC khng ln hn
AB.BC
2

nh l 24: Din tch ca t gic ABCD khng vt qu
AB.BC AD.DC
2
+

nh l 25: Trong cc tam gic c cng chu vi th tam gic u c din tch ln nht.
.:: CHUYN N THI VO LP 10 ::.
Bin son: Trn Trung Chnh 61
Nguyn l on thng: on thng AB l con ng ngn nht ni hai im A v B cho trc trn
mt phng.
Ta c cc h qu sau:
(1) Tng hai cnh ca mt tam gic lun ln hn cnh th ba ca n.
(2) ng gp khc ni hai im A v B cho trc lun c di ln hn di on thng
AB.
(3) di ca cung AB trn mt ng trn cho trc i qua A v B ln hn di on
thng AB.
Nguyn l ng vung gc ngn hn ng xin: on vung gc bao gi cng ngn hn ng
xin.
nh l cnh v gc trong tam gic: Trong mt tam gic ng vi gc ln hn l cnh ln hn v
ngc li.
Mt s bt ng thc cn dng:
(1) Bt ng thc Cauchy:
Vi n 2 s dng ty x
1
, x
2
, , x
n
ta c trung bnh cng ca chng khng nh hn trung bnh
nhn ca chng.
n
1 2 n 1 2 n
x x ... x x x ...x + + + >
(2) Bt ng thc BCS:
Cho trc 2 b n 1 s thc ty x
1
, x
2
, , x
n
v y
1
, y
2
, , y
n
ta c bt ng thc:
( ) ( )( )
2
2 2 2 2 2 2
1 1 2 2 n n 1 2 n 1 2 n
x y +x y +... +x y x +x +... +x y + y +... + y s
(3) Bt ng thc Minkowski:
( ) ( ) ( )
2 2 2
2 2 2 2 2 2 2 2 2
1 1 1 2 2 2 n n n 1 2 n 1 2 n 1 2 n
a b c a b c ... a b c a a ... a b b ... b c c ... c + + + + + + + + + > + + + + + + + + + + +
(4) Bt ng thc Ploteme:
Cho 4 im A, B, C, D trn mt phng ta lun c: AB.CD + AD.BC AC.BD
Du = xy ra T gic ABCD ni tip.
2. Bi tp p dng:
Bi tp 1: Cho ng trn (O) v im P nm trong ng trn( P khng trng vi O).Xc nh v
tr ca dy i qua im P sao cho dy c di nh nht.
Gii
Xt dy AB bt k i qua P. K OH AB
Theo lin h gia dy v khong cch n tm:
AB nh nht OH ln nht
Ta li c OH OP
OH = OP H P
Do maxOH = OP
Khi dy AB vung gc vi OP ti P.
Bi tp 2: Trong cc hnh bnh hnh c hai ng cho bng 6 cm v 8
cm. Hnh no c din tch ln nht? Tnh din tch ln nht .
Gii
Xt hnh bnh hnh ABCD c: AC = 8 cm; BD = 6 cm.
Gi O l giao im hai ng cho.
K BH AC .
Ta c: S
ABCD
= 2S
ABC
= AC.BH
Ta c AC = 8cm, BH BO = 3cm.
Do :
S
ABCD
8.3 = 24 (cm
2
)
S
ABCD
= 24 cm
2

BH BO
H
B
A
P
O
H
O
C
D A
B
www.VNMATH.com
.:: CHUYN N THI VO LP 10 ::.
Bin son: Trn Trung Chnh 62
H O
BD AC
Vy max S
ABCD
= 24 cm
2
.
Khi hnh bnh hnh ABCD l hnh thoi c din tch 24cm
2
.
Bi tp 3: Cho hnh vung ABCD. Trn cc cnh AB, BC, CD, DA ta ly theo th t cc im E, F,
G, H sao cho AE = BF = CG = DH. Xc nh v tr ca cc im E, F, G, H sao cho t gic EFGH
c chu vi nh nht .
Gii
AHAE = AEBF = AFCG = AGHD
HE = EF = FG = GH
EFGH l hnh thoi.
Ta c:

AHE BEF =


0
AHE AEH 90 + =


0
BEF AEH 90 + =

0
HEF 90 =
EFGH l hnh vung.
Gi O l giao im ca AC v EG.
T gic AECG c AE = CG, AE // CG nn l hnh bnh hnh suy ra O l trung im ca AC v
EG, do O l tm ca c hai hnh vung ABCD v EFGH.
AHOE vung cn: HE
2
= 2OE
2
HE = OE 2
Chu vi EFGH = 4HE = 4 2 OE.
Do chu vi EFGH nh nht OE nh nht
K OK AB OE OK (OK khng i)
OE = OK E K
Do : minOE = OK
Nh vy, chu vi t gic EFGH nh nht khi v ch khi E, F, G, H l trung im ca AB, BC, CD,
DA.
Bi tp 4: Cho on thng AB c di 2a. V v mt pha ca AB cc tia Ax v By vung gc vi
AB. Qua trung im ca M ca AB c hai ng thng thay i lun vung gc vi nhau v ct Ax,
By theo th t ti C v D. Xc nh v tr ca cc im C, D sao cho AMCD c din tch nh nht.
Tnh din tch tam gic ?
Gii
Gi K l giao im ca CM v DB
MA = MB;

0
A B 90 = = ,

AMC BMK =
AMAC = AMBK
MC = MK
Mt khc DM CK
ADCK cn


1 2 D D =
K MH CD.
AMHD = AMBD MH = MB = a
S
MCD
=
1
2
CD.MH
1
2
AB.MH =
1
2
2a.a= a
2
S
MCD
= a
2
CD Ax.
Khi :

AMC = 45
0
;

BMD =45
0
.
Vy min S
MCD
= a
2
. Cc im C, D c xc nh trn Ax; By sao cho AC = BC = a.
H
G
F
K E
O
C
D A
B
K
H
y
x
M
D
C
B A
.:: CHUYN N THI VO LP 10 ::.
Bin son: Trn Trung Chnh 63
Bi tp 5: Cho tam gic ABC c

B l gc t. im D di chuyn trn cnh BC. Xc nh v tr ca


im D sao cho tng cc khong cch t B v C n ng thng AD c gi tr ln nht .
Gii
Gi S l din tch AABC.
Khi D di chuyn trn cnh BC ta c:
S
ABD
+ S
ACD
= S
K BE AD, CF AD

1
2
AD.BE +
1
2
AD.CF = S
BE +CF =
2S
AD

Do BE + CF ln nht AD nh nht hnh chiu HD nh nht.
Do HD HB (do

ABD > 90
0
) v HD = HB D B.
Vy Khi D B th tng cc khong cch t B v C n AD c gi
tr ln nht.
Bi tp 6: Cho gc

xOy v im A nm trong gc . Xc nh
im B e Ox, im C e Oy sao cho OB = OC v tng AB + AC l
nh nht.
Gii
K tia Om nm ngoi gc xOy sao cho

yOm xOA = . Trn tia Om
ly im D sao cho OD = OA. Cc im D v A c nh.
OD = OA, OC = OB ,

COD BOA =
ADOC = AAOB CD = AB
Do : AC +AB = AC + CD
M AC +CD AD
AC + AB AD
Xy ra ng thc khi v ch khi C e AD.
Vy min(AC+AB) =AD . Khi C l giao im ca AD v Oy, B thuc tia Ox sao cho OB = OC.
Bi tp 7: Cho hnh ch nht ABCD v im E thuc cnh AD. Xc nh v tr cc im F thuc
cnh AB, G e BC, H e CD sao cho t gic EFGH c chu vi nh nht.
Gii
M
K
I
H
G
F
E
D
C
B
A

A
B
C
D
E
F
G
H
I
K
M

Gi I, K, L theo th t l trung im ca EF, EG, EH.
AAEF vung ti A c AI l trung tuyn AI =
1
2
EF
ACGH vung ti C c CM l trung tuyn CM =
1
2
GH
IK l ng trung bnh ca AEFG IK =
1
2
FG
H
E
F
D
C
A
B
m
C
D
B
A
x
O
y
www.VNMATH.com
.:: CHUYN N THI VO LP 10 ::.
Bin son: Trn Trung Chnh 64
KM l ng trung bnh ca AEGH KM =
1
2
EH
Do : chu vi EFGH = EF +FG +GH +EH = 2(AI + IK + KM + MC)
Ta li c: AI + IK + KM + MC AC
Suy ra: Chu vi EFGH 2AC ( di AC khng i)
Chu vi EFGH nh nht bng 2AC A, I, K, M, C thng hng.
Khi , ta c: EH // AC, FG // AC,

AEI EAI ADB = = nn EF // DB.
Tng t, ta c: GH // DB.
Suy ra: T gic EFGH l hnh bnh hnh c cc cnh song song vi cc ng cho ca hnh ch
nht ABCD.
Bi tp 8: Cho hai ng trn (O) v (O) ct nhau A v B. Mt ct tuyn chung bt k CBD (B
nm gia C v D) ct cc ng trn (O) v (O) ti C v D. Xc nh v tr ca ct tuyn CBD
AACD c chu vi ln nht.
Gii
s

C =
1
2
s

AmB;
s

D =
1
2
s

AnB
S o cc gc AACD khng i
AACD c chu vi ln nht khi mt cnh ca n ln nht,
chng hn AC l ln nht.
AC l dy ca ng trn (O), do AC ln nht khi AC l ng knh ca ng trn (O).
Khi : AD l ng knh ca ng trn (O).
Ct tuyn CBD v tr CBD vung gc vi dy chung AB.
Bi tp 9: Cho ng trn (O) v mt im P nm trong ng trn. Xc nh dy AB i qua P sao
cho

OAB c gi tr ln nht.
Gii
Xt AOAB cn, gc y

OAB ln nht nu gc nh

AOB nh
nht.

1
AOB
2
= s

AB
Gc

AOB nh nht Cung

AB nh nht dy AB nh nht
Khong cch n tm OH ln nht.
Ta c OH OP
OH = OP H P nn maxOH = OP AB OP
Suy ra: Dy AB phi xc nh l dy AB OP ti P.

Bi tp 10: Cho hnh vung ABCD c cnh bng 4cm. Trn cc cnh AB, BC, CD, DA, ly theo
th t cc im E, F, G, H sao cho AE = BF = CG = D. Tnh di AE sao cho t gic EFGH c
chu vi nh nht.
Gii
AAHE = ABEF = ACFG = ADGH
HE = EF = FG = GH, HEF = 90
0

HEFG l hnh vung nn chu vi EFGH nh nht khi HE nh nht.
t AE = x th HA = EB = 4 - x.
AHAE vung ti A nn :
D
C
C'
D'
m
n
B
A
O' O
B'
A'
H B
A
O
P
.:: CHUYN N THI VO LP 10 ::.
Bin son: Trn Trung Chnh 65
HE
2
= AE
2
+ AE
2
= x
2
+ (4 x)
2
= 2x
2
8x +16 = 2(x 2)
2
+8 8
HE = 8 =2 2 x = 2
Chu vi t gic EFGH nh nht bng 8 2 cm , khi AE = 2 cm .
Bi tp 11: Cho tam gic vung ABC c di cc cnh gc vung AB = 6cm, AC = 8cm. M l
im di chuyn trn cnh huyn BC. Gi D v E l chn cc ng vung gc k t M n AB v
AC. Tnh din tch ln nht ca t gic ADME.
Gii
ADME l hnh ch nht .
t AD = x th ME = x
ME //AB
EM CE x CE 4
CE x
AB CA 6 8 3
= = =
AE = 8
4
3
x
Ta c:
S
ADME
= AD.AE = x
4
8 x
3
| |

|
\ .
= 8x
4
3
x
2
=
4
3
(x 3)
2
+12 12
S
ADME
= 12cm
2
x = 3 cm.
Din tch ln nht ca t gic ADME bng 12 cm
2
. Khi D l trung im ca AB, M l trung
im ca BC v E l trung im ca AC.
Bi tp 12: Cho on thng AB, im M di chuyn trn on thng y. V cc ng trn c ng
knh MA v MB. Xc nh v tr ca im M tng din tch ca hai hnh trn c gi tr nh nht.
Gii
t MA = x , MB = y
Ta c: x + y = AB, (0 < x, y < AB)
Gi S v S theo th t l din tch ca hai hnh trn c ng knh l MA v MB.
Ta c:
S + S =
2 2
x y
2 2
| | | |
t + t
| |
\ . \ .
= t.
2 2
x y
4
+

Ta c bt ng thc:

( )
2
2 2
x y
x y
2
+
+ >
S + S
( )
2
x y
.
8
+
> t =
2
AB
.
8
t
Du ng thc xy ra khi v ch khi x = y
Do : min (S + S) =
2
AB
.
8
t .
Khi M l trung im ca AB.
Bi tp 13: Cho im M nm trn on thng AB. V v mt pha
ca AB cc tia Ax v By vung gc vi AB. Qua M c hai ng
thng thay i lun vung gc vi nhau v ct Ax, By theo th t
ti C v D. Xc nh v tr ca cc im C, D sao cho AMCD c din
tch nh nht.
Gii
Ta c: S
MCD
=
1
2
MC.MD
x 8 -
4
3
x
E
D
M
C B
A
B
M
A
O' O

b
a
y
x
M
D
C
B A
www.VNMATH.com
.:: CHUYN N THI VO LP 10 ::.
Bin son: Trn Trung Chnh 66
t MA = a, MB = b,

AMC BDM = =o
MC =
a
coso
, MD =
b
sin o

S
MCD
=
1
2
ab
cos .sin o o

Do a, b l hng s nn S
MCD
nh nht 2sino.coso ln nht.
Theo bt ng thc 2xy s x
2
+y
2
ta c :
2sino.coso s sin
2
o +cos
2
o = 1 nn S
MCD
ab
S
MCD
= ab sino = coso sino = sin(90
0
o) o = 90
0
o o = 45
0

AAMC v ABMD vung cn.
Vy min S
MCD
= ab.
Khi cc im C, D c xc nh trn tia Ax; By sao cho AC = AM, BD = BM.
Bi tp 14: Cho AABC, im M di ng trn cnh BC. Qua M k cc ng thng song song vi
AC v vi AB, chng ct AB v AC theo th t D v E. Xc nh v tr ca im M sao cho hnh
bnh hnh ADME c din tch ln nht.
Gii
S
ADME
ln nht
ADME
ABC
S
S
ln nht
K BK AC ct MD H.
S
ADME
= MD . HK
S
ABC
=
1
2
AC . BK
ADME
ABC
S MD HK
2. .
S AC BK
=
t MB = x , MC = y ,
MD//AC ta c:
MD BM x
AC BC x y
= =
+
;
HK MC y
BK BC x y
= =
+

Theo bt ng thc:
( )
2
xy 1
4
x y
s
+

( )
ADME
2
ABC
S 2xy 1
S 2
x y
= s
+
.
Du ng thc xy ra khi x = y.
Vy maxS
ADME
=
1
2
S
ABC
.
Khi M l trung im ca BC.
Bi tp 15: Cho A ABC vung cn c cnh huyn BC = a. Gi D l trung im ca AB. im E di
chuyn trn cnh AC. Gi H, K theo th t l chn cc ng vung gc k t D, E n BC. Tnh
din tch ln nht ca hnh thang DEKH. Khi hnh thang tr thnh hnh g?
Gii
Ta c:
2S
DEKH
= (DH + EK).HK = (BH + KC).HK
M (BH + KC) + HK = BC = a khng i.
y
x
K
H
E
M
D
C B
A
.:: CHUYN N THI VO LP 10 ::.
Bin son: Trn Trung Chnh 67
Nn (BH + KC).HK ln nht BH + KC) = HK =
a
2

Do :
max S
DEKH
=
2
1 a a a
. .
2 2 2 8
=
Khi : ng cao HK =
a
2
.
Suy ra: KC = BC BH HK = a
a
2

a
2
=
a
4

Do : DH = HB =
a
4
, EK = KC =
a
4
.
Hnh thang DEKH l hnh ch nht, E l trung im ca AC.
Bi tp 16: Cho hnh vung ABCD. Hy xc nh ng thng d i qua tm hnh vung sao cho
tng cc khong cch t bn nh ca hnh vung n ng thng l:
a) Ln nht
b) Nh nht
Gii
Xt trng hp d ct hai cnh i BC v AD
Gi m l tng cc khong cch t bn nh hnh vung n D.
m = 2(AA + BB)
Gi M, N ln lt l trung im ca AB v AB
Suy ra: m = 4MN. Do :
m ln nht MN ln nht
m nh nht MN nh nht
a) MN s MO m ln nht M O d // AB
b) K MH OB.
Chng minh: MN MH
MN nh nht N H d BD hoc d AC.
Bi tp 17: Cho AABC vung cn ti A cc im D, E theo th t
di chuyn trn cc cnh AB, AC sao cho BD = AE. Xc nh v tr cc im D, E sao cho:
a) DE c di nh nht.
b) T gic BDEC c din tch ln nht.
Gii
a) Gi M l trung im ca BC.
ABDM = AAEM

BMD = AME


0
DME = DMA+AME = DMA+BMD= BMA= 90
Gi I l trung im ca DE.
DE = DI+IE =AI + IM AM
Min DE = AM I l trung im ca AM
D l trung im ca AB v E l trung im ca AC
b)t AE = x, AB =AC =a th AD = a x, S
ADE
=
x(a - x)
2

S
BDEC
nh nht S
ADE
ln nht x(a x) ln nht
Do x + (a x) = a khng i nn x(a x) ln nht x = a x x =
a
2

Khi D l trung im ca AB v E l trung im ca AC.
E
K
H
D
C
A
B
d
N
H
D'
C'
M
A'
F
B'
O
C
D A
B
I
E
M
D
C
A
B
www.VNMATH.com
.:: CHUYN N THI VO LP 10 ::.
Bin son: Trn Trung Chnh 68
Bi tp 18: Cho AABC vung ti A, c BC = a, din tch l S. Gi m l trung im ca BC. Hai
ng thng thay i qua M v vung gc vi nhau ct cc cnh AB, AC D, E. Tm:
a) Gi tr nh nht ca on thng DE.
b) Gi tr nh nht ca din tch AMDE.
Gii
a) Gi O l trung im ca DE
Ta c OA = OD = OE = OM
DE = OA + OM AM =
a
2

minDE =
a
2
O l trung im ca AM
D l trung im ca AB v E l trung im ca AC
b) K MH AB, MK AC
ME MK, MD MH.
2S
MDE
= MD.ME MH.MK =
AC
2
.
AB
2
=
S
2

minS
MDE
=
S
4
D H v E K
Bi tp 19: Cho im m di chuyn trn on thng AB. V cc tam gic u AMC v BMD v mt
pha ca AB. Xc nh v tr ca M tng din tch hai tam gic u trn l nh nht.
Gii
Gi K l giao im ca AC v BD.
Cc tam gic AMC, BMD ng dng vi AAKB
t AM = x , BM = y, AB = a, ta c:
2
1
S x
=
S a
| |
|
\ .
;
2
2
S y
=
S a
| |
|
\ .


( )
2
2 2 2
1 2
2 2 2
x y S S x y a 1
S a 2a 2a 2
+ + +
= > = =
Du ng thc xy ra khi v ch khi x = y
Do : min (S
1
+S
2
) =
1
2
M l trung im ca AB.
Bi tp 20: Cho tam gic nhn ABC c cc cnh a, b, c tng ng ng cao AH = h. Hy dng
hnh ch nht MNPQ ni tip trong tam gic ABC sao cho n c din tch ln nht. Bit M eAB; N
e AC; P, Q e BC.
Gii
Gi I l giao im ca AH v MN
t NP = x ; MN = y; AI = h x
AAMN AABC

MN AI y h x h x
y a.
BC AH a h h

= = =
S
MNPQ
= xy =
a
h
.x(h x)
S
MNPQ
ln nht x(h x) ln nht x + (h x) = h khng i nn x(h x) ln nht
x = h x x = h/2
O
E
D
M
C B
A
E
D
K
H
M
C B
A
y
x
2
1
D
C
K
M
B A
h-x
y
I
Q P
N
H
M
C B
A
.:: CHUYN N THI VO LP 10 ::.
Bin son: Trn Trung Chnh 69
Khi MN l ng trung bnh ca AABC.
Bi tp 21: Cho AABC vung ti A. T mt im I nm trong tam gic, k IM BC, IN AC,
IKAB. Tm v tr ca I sao cho tng IM
2
+ IN
2
+ IK
2
nh nht.
Gii
K AH BC, IE AH
ANIK, IMHE l cc hnh ch nht.
IK
2
+ IN
2
= IK
2
+ AK
2
= AI
2
AE
2

IM = EH
nn IK
2
+ IN
2
+ IM
2
= AI
2
+ EH
2
AE
2
+ EH
2

t AE = x, EH = y ta c:
( )
2
2
2 2
x y
AH
x y
2 2
+
+ > =
IK
2
+ IN
2
+ IM
2

2
AH
2
.
Du = xy ra khi I l trung im ca ng cao AH.
Bi tp 22: Cho AABC nhn. T mt im I nm trong tam gic ta k IM BC, IN AC, IKAB.
t AK = x; BM = y; CN = z. Tm v tr ca I sao cho tng x
2
+ y
2
+ z
2
nh nht.
Gii
t:
BK = k, CM = m, AN = n, BC = a, AC = b, AB = c.
x
2
+ y
2
+ z
2
= (IA
2
IK
2
) + (IB
2
IM
2
) + (IC
2
IN
2
)
= (IA
2
IN
2
) + (IB
2
IK
2
) + (IC
2
IM
2
)
= n
2
+ k
2
+ m
2

2(x
2
+ y
2
+ z
2
) = x
2
+ y
2
+ z
2
+ n
2
+ k
2
+ m
2
= ( x
2
+ k
2
) + (y
2
+ m
2
) + ( z
2
+ n
2
)
x
2
+ k
2

( )
2
2 2
x +k
AB c
= =
2 2 2

y
2
+ m
2

( )
2
2 2
y +m
BC a
= =
2 2 2

z
2
+ n
2

( )
2
2 2
z +n
AC b
= =
2 2 2

x
2
+ y
2
+ z
2

2 2 2
a +b +c
4
.
min(x
2
+ y
2
+ z
2
) =
2 2 2
a +b +c
4
x = k, y = m, z = n.
I l giao im ca cc ng trung trc ca AABC.
Bi tp 23: Cho na ng trn c ng knh AB = 10 cm. Mt dy CD c di 6cm c hai u
di chuyn trn na ng trn. Gi E v F theo th t l hnh chiu ca A v B trn CD. Tnh din
tch ln nht ca t gic ABFE.
Gii
K OH CD, ta tnh c: OH = 4cm.
S
ABFE
=
1
2
(AE + BF).EF
= OH.EF s OH.AB = 4.10 = 40
max S
ABEF
= 40cm
2
EF // AB.
Khi : OH AB
E
H
K
N
M
I
C
B
A
n
m
k
z
y
x
N
M
K
I
C B
A
F
H
E
D
C
B A O
www.VNMATH.com
.:: CHUYN N THI VO LP 10 ::.
Bin son: Trn Trung Chnh 70

Bi tp 24: Cho hnh vung ABCD cnh a. V cung BD tm A bn knh a (nm trong hnh vung).
Mt tip tuyn bt k vi cung ct BC, CD theo th t M v N. Tnh di nh nht ca MN.
Gii
t:
CM = m, CN = n, MN = x
m + n + x = 2CD = 2a v m
2
+ n
2
= x
2

Do : x
2
= m
2
+n
2

( )
2
m+n
2

2x
2
(2a x)
2
x 2 2a x
x
2a
2a( 2 1)
2 1
=
+

min MN = 2a
( )
2 -1 m = n.
Khi : Tip tuyn MN // BD, AM l tia phn gic ca

BAC, AN l phn gic ca

DAC
Bi tp 25: Cho hai ng trn (O) v (O) tip xc ngoi ti A. Qua A v hai tia vung gc vi
nhau, chng ct cc ng trn (O), (O) ln lt ti B v C. Xc nh v tr ca cc tia A
ABC c din tch ln nht.
Gii
K OD AB; OE AC ta c:
S
ABC
=
1
2
AB.AC =
1
2
.2AD.2AE = 2.AD.AE
t: OA = R; OA = r;

AOD= O'AE =
AD = R sino; AE = r coso
S
ABC
= Rr.2sino.coso
2sino.coso s sin
2
o + cos
2
o = 1
S
ABC
s Rr
Do :
max S
ABC
= Rr sino = coso sino = sin(90
0
o) o = 90
0
o o = 45
0
.
Vy nu ta v cc tia AB, AC ln lt to vi cc tia AO, AO thnh cc gc

0
OAB= O'AC= 45 th
AABC c din tch ln nht.
Bi tp 26: Cho ng trn (O; R) ng knh BC, A l mt im di ng trn ng trn. V tam
gic u ABM c A v M nm cng pha i vi BC. Gi H l chn ng vung gc k t C
xung MB. Gi D, E, F, G theo th t l trung im ca OC, CM, MH, OH. Xc nh v tr ca im
A din tch t gic DEFG t gi tr ln nht.
Gii
DEFG l hnh bnh hnh.
K OI FH, ta c OI l ng trung bnh ca A BHC nn
OI =
1
2
HC = GD
MO l ng trung trc ca AB nn

0
IMO= 30
OI =
1
2
OM GD =
1
2
OM
M ED =
1
2
OM EG = GD
DEFG l hnh thoi
H
n
m
N
M
A
D
C B

r
R
E
D
C
B
O O'
I
G
F
E
D
H
M
A
C
B
O
.:: CHUYN N THI VO LP 10 ::.
Bin son: Trn Trung Chnh 71

0
HFG = HMO= 30

0
EFG = 60 AEFG u
S
DEFG
= 2S
EFG
= 2.
2 2
EF 3 EF 3
=
4 2
=
2
HC
3
2
2
| |
|
\ .
s
2
BC
3
2
2
| |
|
\ .
=
2
R 3
2

maxS =
2
R 3
2
H B

0
MBC= 90

0
ABC= 30 AC = R.
Bi tp 27: Cho AABC ni tip ng trn (O). D l im bt k thuc cung BC khng cha A v
khng trng vi B, C. Gi H, I, K theo th t l chn cc ng vung gc k t D n cc ng
thng BC, AC, AB. t BC = a, AC = b, AB = c, DH = x, DI = y, DK = z.
a) Chng minh rng:
b c a
y z x
+ =
b) Tm v tr ca im D tng
a b c
x y z
+ + nh nht.
Gii
a) Ly E trn BC sao cho

CDE = ADB
ACDE ng dng vi A ADB

DH CE x CE c CE
DK AB z c z x
= = =
Tng t ABDE ng dng vi A ADC

DH BE x BE b BE
DI AC y b y x
= = =

b c BE CE a
y z x x
+
+ = =
b)
a b c
x y z
+ + =
a a
+
x x
=
2a
x
.
Do S nh nht
a
x
nh nht x ln nht D M (M l im chnh gia ca cung BC
khng cha A).
Bi tp 28: Cho AABC nhn, im M di chuyn trn cnh BC. Gi P, Q l hnh chiu ca M trn
AB, AC. Xc nh v tr ca im M PQ c di nh nht.
Gii
T gic APMQ l t gic ni tip. Gi O l tm ng trn
ngoi tip t gic APMQ.
K OH PQ. t

BAC = o th

POH = o
PQ = 2PH = 2.OP.sino = AM.sino
Do o khng di nn
PQ nh nht AM nh nht AM BC.
Bi tp 29: Cho on thng AB v mt im C trn AB.
V trn cng mt na mt phng b AB cc na ng
trn c ng knh AB, AC, BC. Xc nh v tr ca im
C trn on AB din tch phn gii hn bi ba na
ng trn dt gi tr ln nht.
Gii
M
c
b
z
y
x
E
K
I
H
D
A
C
B
O
H
O
O
P
M
A
C
B
www.VNMATH.com
.:: CHUYN N THI VO LP 10 ::.
Bin son: Trn Trung Chnh 72
Gi (O
1;
r
1
); (O
2;
r
2
); (O
3;
r
3
) l cc ng trn c ng
knh l AB, AC, BC
t: AB = 2a, AC = 2x th r
1
= a, r
2
= x.
Suy ra: BC = 2a 2x v r
3
= a x
Gi S l din tch gii hn bi ba ng trn
Ta c :
2 2 2
3 1 2
r r r
S = - +
2 2 2
| |
|
\ .
=
( )
( )
2
2 2
a - x
a x
- - = x a - x
2 2 2

S ln nht x(a x) ln nht
Mt khc x + (a x) = a khng i nn
x(a x) ln nht x = a x x =
a
2
C O
1

Lc ta c S =
2
a
4
.
Bi tp 30: Cho ng trn (O; R). Trong ng trn (O) v
hai ng trn (O
1
) v (O
2
) tip xc ngoi nhau v tip xc
trong vi (O) trong bn knh ng trn (O
2
) gp i bn
knh ng trn (O
1
). Tm gi tr nh nht ca din tch phn
hnh trn (O) nm ngoi cc hnh trn (O
1
) v(O
2
).
Gii
Gi x l bn knh ng trn (O
1
).
Khi 2x l bn knh ng trn (O
2
)
Xt AOO
1
O
2
, ta c: O
1
O
2
s O O
1
+ OO
2

3x s (R x) + (R 2x) 6x s 2R x s
R
3

Gi S l phn din tch hnh trn (O) nm ngoi cc ng trn
(O
1
) v (O
2
), ta c:
S =
( )
2 2 2 2 2
R - x - 4x = R - 5x
Do x s
R
3
nn x
2
s
2
R
9
S
2
4R
9
;
minS =
2
4R
9
x =
R
3
.
Khi : O
1
, O, O
2
thng hng v bn knh cc ng trn (O
1
)
v (O
2
) l
R
3
v
2R
3
.
Bi tp 31: Cho hnh vung ABCD c cnh bng 1, im M nm trn ng cho BD.
a) Nu cch dng ng trn (I) i qua M v tip xc vi hai cnh AD v CD. Nu cch dng
ng trn (K) i qua M v tip xc vi hai cnh AB, BC.
b) Chng minh rng khi im M di chuyn trn ng cho BD th tng chu vi hai ng trn
khng i.
c) Xc nh v tr ca im M trn BD tng din tch ca hai hnh trn t gi tr nh nht.
Gii
a) Qua M k ng vung gc vi BD ct AB, BC, CD, DA ti P, Q, F, E.
Do AB, BC tip xc vi (K) nn K e MB
PQ KM nn PQ l tip tuyn ca (K)
Vy (K) l ng trn ni tip APBQ
O
3
O
2
C O
1
B
A
O
2
O
1
O
O
2
O
1
O
.:: CHUYN N THI VO LP 10 ::.
Bin son: Trn Trung Chnh 73
Tng t (I) l ng trn ni tip AEDF.
b) Tng chu vi hai ng trn (I) v (K) bng:
2t.IM + 2t.MK = 2t .IK
MD = ID +IM = 2.IJ +IM= 2.IM+IM= ( 2 +1).IM
MB = KB +MK =
2.KH+KM= 2.KM+KM= ( 2 +1).KM


BD = MD + MB =
( )
( ) 2 +1 IM+MK =
( )
2 +1 IK
IK =
( )
BD
= BD 2 -1
2 +1
.
Do BD = AB 2 = 2
IK = 2 ( 2 1) = 2 2
Vy tng chu vi hai ng trn bng 2t(2 2 )
c) Gi x v y l bn knh cc ng trn (I) v (K)
Ta c: x + y = 2 2
Gi S
1
, S
2
l din tch cc hnh trn trn
S
1
+ S
2
= tx
2
+ty
2
= t(x
2
+ y
2
)
( )
( )
2
2
2- 2
x + y
=
2 2

S
1
+ S
2
nh nht x = y M l trung im ca BD.

Bi tp 32: Cho ng trn (O; R), A v B l hai im c nh nm ngoi ng trn. M l im c
nh trn ng trn (O). Xc nh v tr ca im M din tch tam gic MAB c gi tr:
a) Ln nht b) nh nht
Gii
V ng thng d qua O v vung gc AB ti K
d ct ng trn (O) ti C v D.
H AH AB
S
MAB
=
MH.AB
2

a) Ta c: MH MK
Xt 3 im M, O, K, ta c: MK OM + OK
MK OC + OK
MH CK
S
MAB

CK.AB
2
(khng i)
Du = xy ra H

K M

C.
b) Xt 3 im M, O, H, ta c: MH OH OM
M OK OH v OK - OM = OK - OD = DK MH DK
S
MAB

DK.AB
2
(khng i). Du "=" xy ra M
e
[OH]
V M

K M

D
Bi tp 33: Cho ng trn (O; R); A l im c nh trong ng trn (A = O). Xc nh v tr ca
im B trn ng trn O sao cho gc OBA ln nht.
Gii
Gi s c B e (O).
F
E
Q
P
H
J
I
K
M
D C
B A
H
M
D
K
C
B
A
O
www.VNMATH.com
.:: CHUYN N THI VO LP 10 ::.
Bin son: Trn Trung Chnh 74
V dy BC ca ng trn (O) qua A ta c OB = OC = R.
AOBC cn ti O

OBC =
0
180 COB
2


Nn

OBAmax

COBmin.
Trong ACOB, c: CO = OB = R khng i

COBmin BCmin = OHmax


M OH s OA nn OHmax H A BC OA ti A
Vy

OBAmax B e (O) sao cho BC OA ti A.


Bi tp 34: Cho t gic li ABCD. Tm im M trong t gic sao cho AM + MB + MC + MD
t cc tr nh nht.
Gii
Vi 3 im M, A, C, ta c: MA + MC > AC
Ta c: MB + MD > BD.
AM + MB + MC + MD AC + BD (khng i).
Du "=" xy ra
M AC M O
M BD
e


Vy min(AM + MB + MC + MD) = AC + BD M O
Bi tp 35: Cho AABC (

0
A 90 = ) M l im chuyn
ng trn cnh BC. V MD AB; ME AC (D e AB,
E e AC). Xc nh v tr ca M DE c di nh
nht.
Gii
V AH BC (H e BC), H c nh v AH khng i, t
gic AEMD c

0
A E D 90 = = =
AEMD l hnh ch nht.
DE = AM m AM > AH (khng i)
(Theo tnh cht ng xin v ng vung gc).
Du "=" xy ra M H.
Vy khi M H th DE nh nht.
Bi tp 36: Cho ng thng d v ng trn (O; R) c
khong cch t tm n d l OH > R. Ly hai im bt k
A e d; B e (O; R). Hy ch ra v tr ca A v B sao cho
di ca AB ngn nht? Chng minh iu .
Gii
T tm (O) k OH d, OH ct ng trn (O) ti K. Xt
ba im A, B, O, ta c:
AB + OB s OA m OA > OH (quan h ng xin v
ng vung gc).
AB > OH - OB = HK khng i
Vy min AB = KH
A H
B K


Bi tp 37: Cho ng trn (O) v mt im M nm trong ng
trn (M = O). Xc nh v tr ca dy cung AB ca ng trn
(O) qua M sao cho di AB ngn nht.
Gii
Ta c dy AB OM ti M l dy cung c di nh nht.
A
H
B
C
O
M
O
D
C
B
A
H
D
E
M
C
B
A
d
B
A
K
H
O
A
A' B'
B
O
M
.:: CHUYN N THI VO LP 10 ::.
Bin son: Trn Trung Chnh 75
Tht vy:
Qua M v dy A'B' bt k ca (O), A'B' khng vung gc vi OM. V OM' A'B'.
M' e A'B'; M' = M
OM' MM'
OM > OM'
AB < A'B' (theo nh l khong cch t tm n dy).
Bi tp 38: Cho tam gic u ABC ni tip trong ng trn (O; R). M l im di ng trn ng
trn (O). Xc nh v tr ca M MA + MB + MC t gi tr ln nht.
Gii
Ta xt M e cung BC. Trn MA ly D sao cho MB = MD.
Ta chng minh c: ABMD l tam gic u.


2 3 B B + = 60
0
.
M

1 2 B B + = 60
0


1 3 B B + = 60
0
.
Chng minh cho ABAD = ABCM (g-c-g)
AD = MC
MA + MB + MC = MA + MD + DA = 2MA
M MA l dy cung ca ng trn (O; R) MA = 2R
max (MA + MB + MC) = 2.2R = 4R
MA l ng knh ca ng trn (O) M l im chnh
gia ca cung BC.
Tng t ta xt M thuc cung AB v M thuc cung AC
M l im chnh gia cung AB hoc cung AC th MA + MB + MC t gi tr ln nht.
Bi tp 39: Cho ng trn (0; R), ng knh AB, M l im chuyn ng trn ng trn. Xc
nh v tr ca M trn ng trn, MA + 3 MB t gi tr ln nht
Gii
Ta c:

0
AMB 90 = (gc ni tip chn na ng trn)
A MAB, c:

0
M 90 = . Theo nh l Pitago ta c:
MA
2
+ MB
2
= AB
2
= 4R
2

p dng BT Bunhiacopski, ta c:
MA + 3 MB
2 2 2
(1 3)(MA MB ) 4.4R + + = = 4R
MA + 3 MB 4R
Du "=" xy ra
1 3 MB
3
MA MB MA
= =
tanA =
MB
3
MA
= = tg60
0

0
MAB 60 = nn max(MA + 3 .MB) = 4R

0
MAB 60 =
Bi tp 40: Cho on thng AB, im M di chuyn trn
on y. V cc ng trn ng knh MA, MB. Xc
nh v tr ca M tng din tch ca hai hnh trn c
gi tr nh nht.
Gii
t MA = x, MB = y.
Ta c: x + y = AB ( 0 < x < y < AB )
Gi S v Sth t l din tch ca 2 hnh trn c ng
knh l MA v MB


D
O
M
C
B
A
M
O
A B
O
2
O
1
M
B A
www.VNMATH.com
.:: CHUYN N THI VO LP 10 ::.
Bin son: Trn Trung Chnh 76
Ta c: S + S =
2 2
2 2
x y x y
.
2 2 4
+ | | | |
t + t = t
| |
\ . \ .

p dng BT: x
2
+ y
2

( )
2
x y
2
+
S + S t.
( )
2
x y
8
+
=
2
AB
.
8
t
Du "=" xy ra x = y.
Vy Min(S + S ) =
2
AB
.
8
t M l trung im ca AB.
Bi tp 41: Cho AABC c BC = a, AC = b, AB = c. Tm im M nm bn trong tam gic ABC sao
cho
a b c
x y z
+ + c gi tr nh nht. Trong x, y, z l khong cch t M n BC, AC, AB.
Gii
Gi din tch A ABC l S.
Ta c ax + by + cz = 2S (khng i)
p dng BT Bunhiacopski ta c:
(ax + by + cz)
a b c
x y z
| |
+ +
|
\ .

2
a b c
ax by cz
x y z
| |
+ +
|
|
\ .

(ax + by + cz)
a b c
x y z
| |
+ +
|
\ .
(a + b + c)
2


a b c
x y z
| |
+ +
|
\ .

( )
2
a b c
2S
+ +

Vy
a b c
x y z
+ + t gi tr nh nht.

a b c
x y z
| |
+ +
|
\ .
=
( )
2
a b c
2S
+ +


by ax cz
a b c
x y z
= = x = y = z AABC l tam gic u.
Bi tp 42: Cho ng trn (O; R), dy BC c nh. Tm v tr ca A trn cung ln BC tam gic
ABC c chu vi ln nht.
Gii
BC c nh nn

CAB khng i, di BC khng i


Chu vi AABC ch cn ph thuc vo AB + AC.
Trn tia i ca tia AB ly D sao cho AC = AD.
Vy chu vi ca AABC ph thuc vo di ca BD.
Ta c:

CDB cng khng i hay BD l dy ca cung cha gc

1
A
2

dng trn BC.
Vy BD ln nht bng ng knh ca cung cha gc

1
A
2
.
Dng trn BC A l im chnh gia ca cung ln BC.
Bi tp 43: Cho ng trn (O; R) vi dy AB c nh sao
y
z
x
M
c
b
a B C
A
D
O
B C
A
.:: CHUYN N THI VO LP 10 ::.
Bin son: Trn Trung Chnh 77
cho khong cch t O ti AB bng
2
R
. Gi H l trung im ca AB, tia HO ct ng trn (O; R)
ti C. Trn cung nh AB ly M ty (khc A, B). ng thng qua A v song song vi MB ct CM
ti I. Dy CM ct dy AB ti K.
a) So snh gc AIM vi gc ACB.
b) Chng minh: .
MK
1
MB
1
MA
1
= +
c) Gi R
1
, R
2
ln lt l bn knh ng trn ngoi tip AMAK v AMBK. Hy xc nh v tr ca
im M trn cung nh AB tch R
1
.R
2
t gi tr ln nht.
Gii
a) Xt AAOH c cosO =

0
OH 1
= AOH = 60
OA 2


0 0 0
AOB 120 sAB 120 ACB 60 = = =
AABC c ng cao CH ng thi l trung tuyn.
Vy tam gic ABC u

0
ACB 60 =
AI // MB

0
AIM CMB CAB 60 = = =
Vy

AIM ACB = .
b) AAIM u (c hai gc bng 60
0
) AM = MI.
MB CI c) - g - (c AMB AIC = A = A
AMKA AMBC nn
MC
MB
MA
MK
=
AMKB AMAC nn
MK MA
MB MC
=

Vy
MK MK MB MA MB MA
1
MA MB MC MC MC
+
+ = + = =
hay .
MK
1
MB
1
MA
1
= +
c)
Trong AAKM:
3
AK
60 sin 2
AK
M sin 2
AK
R
0 1
= = =
Trong ABKM:
3
BK
60 sin 2
BK
M sin 2
BK
R
0
2
= = =
p dng bt ng thc Csi cho 2 s khng m R
1
, R
2
c:
1 2
1 2
R R AK BK 3R R
R R const
2 2 2 3 2 3
+ +
s = = = =
du "=" xay ra khi R
1
= R
2
AK = BK M l im chnh gia ca cung AB.
Vy R
1
R
2 max
=
2
R
4
khi M l im chnh gia ca cung AB.
Bi tp 44: Cho na ng trn tm O ng knh AB = 2R. Ly im C l trung im ca AO. K
hai tia Ax v By vung gc vi AB v cng mt pha vi na ng trn. im M di ng trn
na ng trn (M A, B). Mt ng thng vung gc vi CM ti M ct Ax P, ct By Q. Tm
v tr ca im M trn na ng trn t gic APQB c din tch nh nht. Tm gi tr din tch
nh nht .
Gii
K
I
M
C
H
O
A
B
www.VNMATH.com
.:: CHUYN N THI VO LP 10 ::.
Bin son: Trn Trung Chnh 78
T gic APMC ni tip

PCA PMA =
C

0
AMB 90 =

0
PMA BMQ 90 + =
T gic BQMC ni tip


BMQ BCQ = .
C

0
CAQ 90 =

0
BCQ BQC 90 + = .
Vy

PCA BQC = .
Do : AAPC ABCQ:
BC . AC BQ . AP
BQ
BC
AC
AP
= =
2
R 3R 3R
. const
2 2 4
= = = .
p dng bt ng thc Csi:
R
2
3
4
R 3
BQ . AP
2
BQ AP
2
= = >
+
du "=" xy ra khi AP = BQ.
CM AB.
Hay ( )
2
min ABQP
R 3 R 3 . R 2 .
2
1
BQ AP AB
2
1
S = = + = khi s

AM = 60
0
.
Bi tp 45: Cho tam gic u ABC, E l mt im trn cnh AC (E A), K l trung im ca on
AE. ng thng EF i qua E v vung gc vi ng thng AB (F e AB) ct ng thng i qua
C v vung gc vi ng thng BC ti D. Xc nh v tr ca E sao cho on KD c di nh
nht.
Gii
AAEF vung ti F,

0
A 60 = , FK l trung tuyn ng vi cnh huyn
AAKF u

0
FKC 120 = .
Vy t gic BCKF ni tip.
T gic BCDF c

0
F C 90 = =
Vy t gic BCDF ni tip hay 5 im B, C, D, K, F cng
thuc mt ng trn ng knh BD.
s

DK = 2

DFK = 60
0

KD =
2
1
DB
1
CB
2
s du "=" xy ra khi E C.
Vy KD
min
=
1
CB
2
khi E C.
Bi tp 46: Cho AABC cn B c

ABC = |, O l trung im ca cnh AC, K l chn ng vung


gc h t O xung cnh AB, () l ng trn tm O bn knh OK. E l mt im thay i trn cnh
BA sao cho gc AOE bng (20
0
< < 90
0
). F l im trn cnh BC sao cho EF tip xc vi ().
Tm AE + CF nh nht.
Gii
Trong AOEF:

0 0
1 1
EOF 180 OEF OFE 180 AEF CFE
2 2
= =
Trong t gic AEFC:

( )
( )
0 0 0 0
AEF AFE 360 A C 360 180 180 + = + = | = +|
Q
P
M
C O A B
K
D
F
E
B
A
C
.:: CHUYN N THI VO LP 10 ::.
Bin son: Trn Trung Chnh 79
Vy:

EOF = 90
0
-
2
1

AABC cn

( )
0 0
1 1
A C 180 90
2 2
= = | = |
Vy

EOF A C = = .
AAEO AOEF v AOEF ACOF.
Vy AAEO ACOF.
AE CO
AE.CF AO.CO const
AO CF
= = =
p dng bt ng thc Csi:
AE CF 2 AE.AF 2 AO.CO const + > = = .
Du "=" xy ra khi AE = CF
AOEF cn ti O
AAEO cn ti A


0 0 0 0
1 1 1 1
AOE 90 A 90 90 45
2 2 2 4
| |
= = | = + |
|
\ .
th AE + CF nh nht
Bi tp 47: Cho hai ng trn(O
1
; r
1
) v (O
2
; r
2
) ct nhau ti hai im A v B. Bit rng r
1
= 1cm;
r
2
= 2cm; AB = 1cm v hai im O
1
, O
2
hai pha ca ng thng AB. Xt ng thng (d) i qua
A, ct (O
1
; r
1
) v (O
2
; r
2
) lm lt ti cc im M v N sao cho A nm trong on MN. Tip tuyn
ca (O
1
; r
1
) ti M v tip tuyn ca (O
2
; r
2
) ti N ct nhau ti im E.
a) Chng minh t gic EMBN l t gic ni tip.
b) Tnh O
1
O
2
b) Tm gi tr ln nht ca 2EM + EN.
Gii
a)

ABN ANE; ABM AME = =


0
MBN EMN ENM 180 MEN = + =
Vy

0
MBN MEN 180 + = nn t gic EMBN ni tip.
b) O
1
O
2
=
1
( 3 15
2
+ )
c)
1 2
O O A A MNB A .
2
AO
AO
BM
BN
1
2
= = (v R
1
= 1cm, R
2
= 2cm)
BN 2BM =
AEMB ANAB
AN . MB EM
AB
AN
MB
EM
= = (v AB = 1cm)
Tng t, ta cng ch ra: EN = NB.AM
Vy
2.EM + EN = 2.MB.AN + NB.AM
= 2. MB.AN + 2.MB.AM
= 2.MB.(AM + AN)
= 2MB.MN
Li c AMBN AO
1
AO
2
ng dng theo t s
1 2 1
MN MB
2
O O r
= s
E
N
M
B
A
O
2
O
1
F
E
K
C
B
A
O
www.VNMATH.com
.:: CHUYN N THI VO LP 10 ::.
Bin son: Trn Trung Chnh 80
Vy
2 1 1
O O 2 . r 2 . 2 MN . MB 2 s = ( ) 15 3 4 ) 15 3 (
2
1
. 8 + = + du "=" xy ra khi MB = 2r
1
hay M
i xng vi B qua O
1
.
Bi tp 48: Cho tam gic ABC vung ti A ni tip ng trn (O). M l im nm trn cung BC
khng cha im A. Gi N, H, K ln lt l hnh chiu ca M trn AB, BC, CA. Tm gi tr nh
nht ca
BC AB AC
MH MN MK
+ +
Gii
Nhn thy: Nu K nm ngoi AC th N nm trong AB.

AB BN AN
MN MN MN
AC AK CK
MK MK MK
= +
=

AMCK AMBN
BN CK
MN MK
=
AMAK AMBH
AK BH
MK MH
=
AMAN AMCH
AN CH
MN HM
=
Vy
BC AB AC BC CH BH BC
2
MH MN MK MH MH MH MH
+ + = + + =
Vy
BC AB AC
MH MN MK
+ + nh nht MH ln nht MH = R M
l im chnh gia ca cung BC.
Bi tp 49: Trn mt phng cho trc tam gic u ABC v im
M bt k. Chng minh rng:
MA + MB MC
Chng minh
S dng bt ng thc Plotemy vo t gic MABC ta c:
MA.BC + MB.AC MC.AB
Nn MA + MB MC (pcm)
Bi tp 50: Chng minh rng im B nm trong ng trn ng knh AC khi v ch khi ta c

0
ABC> 90
Chng minh
Cn v im B nm trong hnh trn ng knh AC l
AC
BM<
2
.
Gi A l im i xng vi B qua trung im M ca cnh AC.
Theo nh l 3, ta c BC > AA khi v ch khi

BAC> ABA' .
Do

BAC> ABA' b nhau nn

BAC> ABA' khi v ch khi

ABC l gc t.
3. Bi tp t luyn:
Bi tp 1: Cho gc vung xOy; im A thuc min trong ca gc. Cc dim M, N theo th t
chuyn ng trn cc tia Ox, Oy sao cho

0
MAB 90 = . Xc nh v tr ca M, N MN c di nh
nht.
K
H
N
M
O
C
B
A
M
C
B
A
A'
M
B
A
C
.:: CHUYN N THI VO LP 10 ::.
Bin son: Trn Trung Chnh 81
Bi tp 2: Cho 2 ng trn ngoi nhau (O; R) v (O'; R'). A nm trn (O), B nm trn (O'). Xc
nh v tr ca im A, B on thng AB c di ln nht.
Bi tp 3: Chng minh rng bn knh ng trn ni tip tam gic vung ABC khng vt qu
( )
AD 2 -1 , vi AD l di ng phn gic ca gc vung A.
Bi tp 4: Trn cnh BC, AC ca tam gic u ABC ly tng ng hai im M v N sao cho BM =
CN. Tm v tr ca M MN c gi tr ln nht.
Bi tp 5: Cho na ng tron (O; R) ng knh AB. M l mt im trn na ng trn, k
MHHB. Xc nh v tr ca M :
a) SAABC ln nht.
b) Chu vi ca AMAB ln nht.
Bi tp 6: Trn cnh BC, AC ca tam gic u ABC ly tng ng hai im M v N sao cho BM =
CN. Tm v tr ca M MN c gi tr ln nht.
Bi tp 7: Cho t gc ABCD ni tip trong ng trn (O; R) cho trc. Tm t gic c tng
AB.CD + AD.BC t gi tr ln nht.
Bi tp 8: Cho hnh vung ABCD c di cnh bng a. Trn hai cnh AB v AD ln lt ly 2
im M, N sao cho chu AAMN = 2a. Tm v tr ca M v N S
AAMN
ln nht.
Bi tp 9: Cho AABC ngoi tip ng trn (O; R). K cc tip tuyn ca ng trn (O; R) song
song vi cc cnh ca tam gic. Cc tip tuyn ny to vi cc cnh ca tam gic thnh 3 tam gic
nh c din tch l S
1
, S
2
, S
3
. Gi S l din tch ca AABC. Tm gi tr nh nht ca t s
1 2 3
S S S
S
+ +

Bi tp 10: Cho tam gic u ABC ni tip ng trn O v D l im nm trn cung BC khng
cha im A. Xc nh v tr ca D sao cho DA + DB + DC ln nht.
Bi tp 11: Cho hai ng trn (O) v (O') ct nhau ti A v B sao cho hai tm O v O' nm v hai
pha khc nhau i vi ng thng AB. ng thng (d) quay quanh B ct cc ng trn (O) v
(O') ln lt ti C v D (C A, B v D A, B). Xc nh v tr ca (d) sao cho an thng CD c
di ln nht.
Bi tp 12: Cho ng trn (O; R) ng knh AB, im M di ng trn ng trn sao cho
MAsMB. Trong tam gic AMB k ng cao MH. Gi r
1
, r
2
, r
3
theo th t l bn knh cc ng
trn ni tip cc tam gic AMB, AMH v BMH. Hy xc nh v tr ca M tng: r
1
+ r
2
+ r
3
t
gi tr ln nht.
Bi tp 13: Cho AABC c

0
A 30 = , AB = c, AC = b, M l trung im ca BC. Mt ng thng (d)
quay xung quanh trng tm G ca tam gic ABC sao cho (d) ct on AB ti P v (d) ct on AC
ti Q.
a) t AP = x, hy tm tp hp cc gi tr ca x.
b) Tnh gi tr ca biu thc
AB AC
AP AQ
+ .
c) Hy tm gi tr nh nht, gi tr ln nht ca din tch tam gic APQ theo b, c.
Bi tp 14: Cho na ng trn tm O ng knh AB = 2R v M l mt im thuc na ng
trn ( khc A v B). Tip tuyn ca (O) ti M ct tip tuyn ti A v B ca ng trn (O) ln lt
ti cc im C v D. Tnh gi tr nh nht ca tng din tch hai AACM v ABDM.
Bi tp 15: Cho ng trn (O) v dy BC c nh. Gi A l im di ng trn cung ln BC ca
ng trn (O), (A khc B v C). Tia phn gic ca

ACB ct ng trn (O) ti D khc C, ly I


thuc on CD sao cho DI = DB. ng thng BI ct ng trn (O) ti im K khc B.
a) Chng minh: AKAC cn.
b) Xc nh v tr ca A di on AI l ln nht.
www.VNMATH.com
.:: CHUYN N THI VO LP 10 ::.
Bin son: Trn Trung Chnh 82
Bi tp 16: Cho AABC c ba gc nhn ni tip ng trn (O; R). im M lu ng trn cung nh
BC. T M k cc ng thng MH, MK ln lt vung gc vi AB, AC (H e AB, e AC). Tm v
tr ca M di on HK ln nht.
Bi tp 17: Cho hai ng trn (O
1
, R
1
) v (O
2
; R
2
) tip xc ngoi vi nhau ti A. ng thng (d)
i qua A ct ng trn (O
1
, R
1
) ti M v ct ng trn (O
2
; R
2
) ti N (cc im M, N khc A).
Xc nh v tr ca ng thng (d) di on thng MN ln nht.
Bi tp 18: ng trn tm O c dy AB c nh v I l im chnh gia ca cung ln AB. Ly
im M bt k trn cung ln AB, dng tia Ax MI ti H v ct BM ti C.
a) Chng minh: AAIB v AAMC cn.
b) Khi M di ng trn cung ln AB. Chng minh rng im C di ng trn mt cung trn c nh.
c) Xc nh v tr ca im M chu vi AAMC t gi tr ln nht.
Bi tp 19: Cho AABC nhn. im D di ng trn cnh BC. Gi O
1
, O
2
ln lt l tm ng trn
ngoi tip cc AABD, AACD tng ng.
a) Chng minh rng ng trn ngoi tip AAO
1
O
2
lun i qua mt im c nh khc A.
b) Gi O l tm ng trn ngoi tip AABC v I l tm ng trn ngoi tip AAO
1
O
2
. Hy xc
nh v tr ca D trn BC sao cho IO nh nht.
Bi tp 20: Cho na ng trn ng knh AB = 2R. Gi C l im ty trn na ng trn, D
l hnh chiu vung gc ca C trn AB. Tia phn gic ca

ACD ct ng trn ng knh AC ti


im th hai l E, ct tia phn gic

ABC ti H.
a) Chng minh: AE // BH.
b) Tia phn gic

CAB ct ng trn ng knh AC ti im th hai l F, ct CE ti I. Tnh din


tch AFID trong trng hp tam gic l u.
c) Trn on BH ly K sao cho HK = HD, gi J l giao ca AF v BH. Xc nh v tr ca C tng
khong cch t cc im I, J, K n ng thng AB t gi tr ln nht.
Bi tp 21: Cho ng trn tm O, bn knh R v dy BC < 2R, cc tip tuyn ca ng trn ti B
v C ct nhau ti A. M l im bt k trn cung nh BC v khng trng vi B, C. Gi H, I, K ln
lt l hnh chiu ca M trn BC, CA, AB. BM ct HK ti P, CM ct HI ti Q.
a) Chng minh: PQ // BC.
b) Xc nh v tr ca M tch MH.MI.MK t gi tr ln nht.

CMI OI 14
QIY TCM - TAP MOP OIIM
1. Kin thc c bn:
1.1. gii bi ton qu tch, ta thc hin cc bc sau:
Phn thun: Phn tch cc yu t c nh v thay i ch ra tp hp m im cn tm qu tch
phi thuc vo (thng l ng trn, ng thng). Ta s s dng cc qu tch c bn (nh cung
cha gc, trung trc, ng trn Appolonius ) xc nh v chng minh qu tch. d on
qu tch, c th phi v mt s v tr (trong c cc v tr c bit) ca cu hnh.

Phn o: Sau khi lm phn thun, tc l xc nh tp hp M nhng im m qu tch thuc vo,
ta cn xem xt xem vi nhng im P no thuc M th tn ti mt cu hnh c v tr im cn tm
qu tch trng vi P. Bc ny s loi b nhng im khng tng ng vi mt cu hnh no.

Gii hn: Sau khi thc hin phn o, ta c th s thy rng ch mt phn ca M thuc v qu tch.
Bc ny m t r phn . V d mc d im P thuc ng trn (C) nhng qu tch c th ch l
mt cung ca (C).
Kt lun: Da trn cc phn trn kt lun qu tch l tp hp nhng im nh th no.
2.2. Mt s qu tch c bn
.:: CHUYN N THI VO LP 10 ::.
Bin son: Trn Trung Chnh 83
(1) Qu tch nhng im cch u hai im A v B l ng trung trc ca on thng , tc l
ng thng qua trung im M ca AB v vung gc vi AB.
(2) Qu tch nhng im A cch mt im I c nh mt on AI = R khng i l ng trn tm I
bn knh R.
(3) Qu tch nhng im cch u hai ng thng ct nhau a v b l hai ng phn gic ca gc
to bi hai ng thng .
(4) Qu tch nhng im cch mt ng thng a cho trc mt on d khng i l hai ng
thng song song vi a v cch a mt khong cch bng d.
(5) Qu tch nhng im nhn on AB c nh mt gc c nh l hai cung cha gc nhn AB
lm dy cung. c bit, nu = 90
0
th qu tch l ng trn ng knh AB.
(6) Cho hai im A, B v s thc k. Qu tch nhng im M sao cho MA
2
MB
2
= k l mt ng
thng vung gc vi AB ti H, trong H xc nh bi h thc:
(HA HB)BA k + =
(7) Cho hai im A, B vi AB = 2a v s thc dng k.
Qu tch nhng im M sao cho MA
2
+ MB
2
= 2k
2
l tp rng nu k
2
< a
2
v l ng trn tm I,
bn knh
2 2
R k a = .
(8) Cho hai im A, B v s thc dng k 1. Qu tch nhng im M sao cho
MA
k
MB
= l ng
trn ng knh EF, trong E v F l cc im thuc ng thng AB sao cho

EA
k
EB
= v
FA
k
FB
=
(ng trn Appolonius)

Lu : Ta phi rn cch gii bi ton qu tch:
- Cc qu tch c bn.
- on qu tch.
- Chng minh qu tch on nhn l ng.
Phng php 1: Chng minh qu tch (tp hp im) da vo tnh cht ca trc i xng, i xng
tm, php tnh tin, php quay, php v t.
Phng php 2: Chng minh qu tch nhanh v chnh xc hc sinh cn phi luyn :
- Xc nh yu t c nh.
- Xc nh yu khng i.
- Xc nh yu t thay i.
Lu : Phi rn phn on qu tch.
Cc dng qu tch thng gp:
(1) Qu tch cc im cch u hai u mt ca mt on thng l ng trung trc ca on thng
.
(2) Qu tch cc im cch u hai cnh ca mt gc l ng phn gic ca gc .
(3) Tp hp cc im cch u mt im O cho trc mt khong khng i R l ng trn (O; R).
(4) Qu tch cc im cch u mt ng thng c nh mt khong bng h l hai ng thng
song song vi ng thng v cch ng thng mt khong bng h.
(5) Qu tch cc im nhn mt cnh di mt gc bng 90
0
l mt ng trn c tm l trung im
ca cnh v ng knh l di ca cnh cho.
(6) Qu tch cc im nhn on AB di mt gc khng i l hai cung cha gc i qua A, B v
i xng vi nhau qua AB.
2. Bi tp p dng:
Bi tp 1: Cho ng trn (O; R) v tam gic cn ABC c AB = AC ni tip ng trn (O; R) K
ng knh AI. Gi M l mt im bt k trn cung nh AC. Mx l tia i ca tia MC. Trn tia i
ca tia MB ly im D sao cho MD = MC.
www.VNMATH.com
.:: CHUYN N THI VO LP 10 ::.
Bin son: Trn Trung Chnh 84
a) Chng minh rng MA l tia phn gic ca ca gc BMx.
b) Gi K l giao th hai ca ng thng DC vi ng trn (O). T gic MIKD l hnh g? v sao?
c) Gi G l trng tm ca tam gic MDK. Chng minh rng khi M di ng trn cung nh AC th G
lun nm trn mt ng trn c nh.
d) Gi N l giao im th hai ca ng thng AD vi ng trn (O). P l giao im th hai ca
phn gic gc IBM vi ng trn (O). Chng minh rng, ng thng DP lun i qua mt im c
nh khi M di ng trn cung nh AC.
Hngdn
a)

1
AMB
2
= s

AB (gc ni tip (O) chn

AB)

0 0
1
AMx 180 AMC 180
2
= = s

ABC =
1
2
s

AC =
1
2
s

AB
Vy:

AMB AMx = hay MA l tia phn gic ca

BMx
b) AMCD cn

MCD MDC = =

1
BMC
2
(gc ngoi
ca tam gic)
Ta li c: AABC cn I l im chnh gia ca cung
BC
Suy ra:

1
IMC IMB BMC
2
= =
Vy

MCD IMC = , suy ra: IM // CD.

MCD MDC BMI = = BI = MK

MIK IMB =
Suy ra: IK // MD.
Vy MIKD l hnh bnh hnh.
c) D thuc ng trn (A; AC)
Gi N l im trn AI sao cho NA =
1
3
AI.
NG =
2
3
AD =
2
3
AC = const
G thuc ng trn
2
N; AC
3
| |
|
\ .
.
Bi tp 2: Cho AABC ni tip ng trn (O;
R). Gi D l im chnh gia ca cung BC khng
cha A. V ng trn qua D v tip xc vi AB
ti B. V ng trn qua D v tip xc vi AC
ti C. Gi E l giao im th hai ca hai ng
trn ny.
a) Chng minh 3 im B, C, E thng hng.
b) Mt ng trn tm K di ng lun i qua A
v D, ct AB, AC theo th t ti M v N. Chng
minh rng BM = CN.
c) Tm qu tch trung im I ca on thng MN.
Hngdn
a)

BED DBx ACB = = ,

CED DCy ABD = =
P
N
G
K
D
x
M
I
C
A
B
O
N
M
y
x
E
D
I
C
B
A
.:: CHUYN N THI VO LP 10 ::.
Bin son: Trn Trung Chnh 85
Suy ra:

0
BEC ABD ACD 180 = + = .
Suy ra: B, E, C thng hng.
b)

BD DC =

BAD CAD =

DN DM = DM = DN.

BD DC = DB = DC

DCN DBM =
ABMD = ACND BM = CN.
c) Tnh c DI = 2KD sin
2
A
2

2
DI A
2sin const
DK 2
= =
K thuc trung trc ca AD I thuc ng thng vung gc vi
AD ct AD ti P sao cho
2
DP A
sin
DA 2
=

Bi tp 3: Cho AABC cn ti A. Cc im M, N theo th t
chuyn ng trn cc cnh AB, AC sao cho AM = CN.
a) Chng minh ng trn ngoi tip AAMN lun i qua mt
im c nh khc A.
b) Tm qu tch tm ng trn ngoi tip AAMN.
Gii
a) ng cao AH ct ng trn ngoi tip tam gic
AMN ti P
AAMP = ACNP
PA = PC
P l tm ng trn ngoi tip AABC
P c nh.
b) Tm I ca ng trn ngoi tip AAMN nm trn
ng trung trc ca AP.

Bi tp 4: Tm qu tch nh C cc AABC c AB c nh,
ng cao BH bng cnh AC.
Hngdn
K ng thng vung gc vi AB ti A, trn
ly E sao cho AE = AB
AACE = ABHA

0
ACE 90 = C thuc cung cha gc 90
dng trn AE.

Bi tp 5: T gic li ABCD c AC c nh,
gc

0
A 45 = ,

0
B D 90 = = .
a) Chng minh rng BD c di khng i.
b) Gi E l giao ca BC v AD, F l giao ca
DC v AB. Chng minh EF c di khng
i?
c) Tm qu tch tm ng trn ngoi tip
AAEF.
Hngdn
a)

0
B D 90 = = B, D thuc ng trn ng
knh AC
I
P
H
N
M
C
B
A
H
E
C
B A
H
I
J
F
E
D
A
C
B
www.VNMATH.com
.:: CHUYN N THI VO LP 10 ::.
Bin son: Trn Trung Chnh 86

0
A 45 = BD = R 2 = const.
b) ACDE vung cn CD = ED.
AADF vung cn DA = DF.
AACD = AFED
EF = AC = const
c) Trung trc ca AF ct trung trc ca AE ti J, ct (O) ti H. Trung trc ca AE ct (O) ti I
H, I l im chnh gia ca hai cung AC H, I c nh.

0
HJI BCD 135 = =
J thuc cung cha gc 135 dng trn HI.
Bi tp 6: Cho hai ng trn (O; R) v (O'; R') ct nhau ti A v D c cc ng knh AOB v
AO'C vung gc vi nhau ti A. Mt ng thng d i qua A v ct cc na ng trn khng cha
im D ca (O), (O') tng ng ti cc im M, N khc A.
a) Chng minh: AABM ACAN.
b) Tm qu tch giao im P ca OM v O'N khi d di ng.
c) Tip tuyn ti M ca (O) ct AD ti I. Chng minh rng: IM
2
= IA. ID.
d) Tm v tr ca ct tuyn d cho tip tuyn ti M ca (O) v tip tuyn ti N ca (O') ct nhau ti
mt im thuc ng thng AD.
e) Xc nh v tr ca d sao cho t gic MNCB c din tch ln nht. Tm gi tr ln nht theo R
v R'.
Hngdn
a) AAMB ACAN.
b)

0
PMA PNA OAM O' AN 90 = = + =

0
OPO' 90 = P thuc ng trn ng knh
OO'.
c) AIMA AIDM IM
2
= IA.ID
d) Tng t cu c gi s tip tuyn ti N ca (O') ct
AD ti I' I'M
2
= I'A.I'D.
Vy I trng I' IM = I'N I thuc trung trc ca
NM.
Vy khi I l giao ca AD v trung trc ca MN th
tip tuyn ti M ca (O) v tip tuyn ti N ca (O')
ct nhau ti mt im thuc ng thng AD.
e) Din tch t gic BMNC ln nht (S
BMA
+ S
ANC
)
min
(S
BMA
)
min
(BM.AM)
min
Ta li c: BM
2
+ AM
2
= R
2
.
Vy: BM.AM
2
R
2
s du bng khi BM = AM d to vi AB mt gc 45
0
.
Khi din tch t gic BMNC l:
( )
2 2
1
R.R' +R +R'
2
.

Bi tp 7: Mt im A i ng trn na ng trn
ng knh BC c nh. ng thng qua C song song
vi BA ct ng phn gic ngoi ca gc BAC ca tam
gic ABC ti D. Tm qu tch D.
Hngdn
AD ct (O) ti E E c nh
Ta li c:

0
CDE 45 = .
Vy D thuc cung cha gc 45
0
dng trn CE.
I'
I
P
N
M
C
B
D
A
O'
O
E
O
D
A
C
B
.:: CHUYN N THI VO LP 10 ::.
Bin son: Trn Trung Chnh 87

Bi tp 8: Cho ng trn (O; R) c nh v ng thng d ct (O; R) ti hai im A, B c nh.
Mt im M di ng trn d v bn ngoi on AB. V cc tip tuyn MP v MN vi (O; R). Gi
N, P l hai tip im.
a) Chng minh rng khi M di ng, ng trn ngoi tip tam gic MNP lun i qua hai im c
nh.
b) Tm qu tch tm I ca ng trn ngoi tip tam gic MNP.
c) Trnh by cch dng im M sao cho tam gic MNP
l tam gic u.
Hngdn
a) Gi s (I) ct AB ti H khc M

0
OHM 90 =
HA = HB hay H c nh.
Vy (I) i qua O v H c nh.
b) IO = IH I thuc trung trc ca OH.
c) Tam gic MNP u

0
OMN 30 =
OM = 2ON = 2R.
Vy M thuc ng trn (O; 2R).
Bi tp 9: Cho hnh vung ABCD c nh. Mt im I di ng trn cnh AB (I khc A v B). Tia
DI ct tia CB ti E. ng thng CI ct ng thng AE ti M. ng thng BM ct ng thng
DE ti F. Tm qu tch im F.
Hngdn
Trn BC ly G sao cho AI = BG AI EG.
p dng nh l Menelauyt trong tam gic AEB vi 3 im thng hng C, I, M, ta c:

CB IA ME
1
CE IB MA
= (1)
Li c
BE
IB
CE
CD
CE
CB
= =
Thay vo (1)
BG
BE
IA
BE
MA
ME
= =
MB // AG hay

0
DFB 90 = .
Vy F thuc ng trn ng knh BD (cung nh AB).

Bi tp 10: Cho ng trn (O; R) v mt im A c nh
trn ng trn. im M lun ng trn tip tuyn xy ti A ca (O; R). Qua M v tip tuyn th hai
vi (O; R). Gi tip im l B.
a) Tm qu tch tm cc ng trn ngoi tip
AAMB.
b) Tm qu tch trc tm H ca AAMB.
Hngdn
a) ng trn ngoi tip AAMB l ng trn tm
E, ng knh OM.
E thuc trung trc ca OA
b) T gic AOBH l hnh thoi
AH = R.
Vy H thuc ng trn (A; R) (thuc na mt
phng b xy cha B).

H
I
N
P
M
B
A
d
O
G
F
M
E
I
C
B
D A
H
E
B
M
y
x
A
O
www.VNMATH.com
.:: CHUYN N THI VO LP 10 ::.
Bin son: Trn Trung Chnh 88
Bi tp 11: Cho tam gic ABC ni tip ng trn tm O. ng
phn gic ca gc A ct ng trn ti im D. Mt ng trn
(L) thay i nhng lun i qua hai im A v D. (L) ct hai ng
thng AB, AC giao im th hai l M, N (c th trng vi A).
a) Chng minh rng: BM = CN.
b) Tm qu tch trung im K ca MN.
Hngdn
a)

BAD DAC =
DB = DC; DM = DN.
Ta li c:

MBD NCD = ;

BMD NCD =


BDM CDN = .
Vy ABDM = ACDN BM = CN.
b) Tng t cu c bi b)
Bi tp 12: Cho gc vung xOy. Mt chic ke ABC trt
trong mt phng ca gc xOy sao cho nh B di chuyn trn
cnh Ox, nh C di chuyn trn cnh Oy v nh gc vung A
di chuyn trong gc xOy. Tm qu tch im A.
Hngdn
T gic OBAC ni tip

yOA CBA = = o.
Vy A thuc tia to vi tia Oy mt gc o (phn nm trong gc
xOy)
Bi tp 13: Cho ng trn tm O bn knh R v mt im P c nh ngoi ng trn. V tip
tuyn PA v ct tuyn PBC bt k (A, B, C trn (O; R)). Gi H l trc tm ca tam gic ABC. Khi
ct tuyn PBC quay quanh P.
a) Tm qu tch im i xng ca O qua BC.
b) Tm qu tch im H.
Hngdn
a) Ta c: PO' = PO = const; P c nh
O' thuc ng trn (P; PO)
b) T gic OO'HA l hnh bnh hnh. V hnh bnh
hnh AOPK.
K c nh HO'PK cng l hnh bnh hnh
HK = O'P = OP = const.
Vy H thuc ng trn (K; OP).
Bi tp 14: Cho tam gic cn ABC ni tip ng trn (O; R) c AB = AC = R 2 .
a) Tnh di BC theo R
b) M l mt im di ng trn cung nh AC, ng
thng AM ct ng thng BC ti D. Chng minh rng:
AM.AD lun lun l hng s
c) Chng minh tm ng trn ngoi tip AMCD di
ng trn mt ng c nh khi M di ng trn cung
nh AC.
Hngdn
a) BC l ng knh ca (O).
b) Tam gic AMC ng dng vi tam gic ACD
AM.AD = AC
2
= R 2 .
N
M
D
C B
A
O
A
C
B
y
x
O
K
O'
H
C
B
A
P
O
O
D
M
C
B
A
.:: CHUYN N THI VO LP 10 ::.
Bin son: Trn Trung Chnh 89
c)

1
ACM MDC
2
= = s

CM AC l tip tuyn ca (I)


IC vung gc vi AC c nh
I thuc ng thng qua C v vung gc vi CA.
3. Bi tp t luyn:
Bi tp 1: Cho tam gic ABC cn ti A. Mt im P di ng trn cnh BC. V PQ song song vi
AC (Q thuc AB), v PR song song vi AB (R thuc AC). Tm qu tch cc im D i xng vi P
qua QR.
Bi tp 2: Cho gc vung xOy. Cc im A v B tng ng thuc tia Ox, Oy sao cho OA = OB.
Mt ng thng d i qua A v ct OB ti M nm gia O v B. T B h ng thng vung gc vi
AM ct AM ti H v ct ng thng OA ti I.
a) Chng minh rng OI = OM v t gic OMHI ni tip.
b) Gi K l hnh chiu ca O ln BI. Chng minh rng OK = HK.
c) Tm qu tch im K khi M di ng trn on OB.
Bi tp 2: Cho tam gic u ABC ni tip ng trn (O) v M di ng trn cung BC.
a) Trn tia i ca tia CM, ly on CE = MB. Tm tp hp cc im E khi M di ng.
b) Trn tia i ca tia MC, ly on MF = MB. Tm tp hp cc im F khi M di ng.
Bi tp 3: Cho hai ng trn bng nhau (O) v (O') ct nhau ti A v B. Mt ct tuyn (d) bt k
qua B ct (O0 ti C v (O') ti C'. Tm tp hp trung im I ca on CC' khi d quay quanh B.
Bi tp 4: Cho hai ng thng xx' v yy' vung gc vi nhau ti O v mt im P c nh. Mt
gc vung nh P quay quanh P. cc cnh ca gc vung ny ct xx' ti A v yy' ti B. Tm tp hp
trung im I ca on AB.
Bi tp 5: Trn mi bn knh OM ca ng trn (O) ly on OI bng khong cch t M n
ng knh c nh AB. Tm tp hp cc im I.
Bi tp 6: Cho ng trn (O) c nh v mt dy AB c nh. Trn cung nh AB, ta ly im C di
ng. Tm tp hp tm I ca ng trn ni tip tam gic ABC.
Bi tp 7: Cho ng trn (O) v mt dy AB c nh. K mt dy AC. Trn ng thng AC ly
hai im M, M' sao cho CM = CM' = CB, M nm ngoi ng trn. Tm tp hp cc im M v M'
khi C vch cung AB.
Bi tp 8: Cho ng trn (O; R), 2 im B, C c nh trn (O) v mt im A di ng trn (O).
Tm tp hp cc trc tm H ca tam gic ABC.
Bi tp 9: Cho tam gic ABC. Tm tp hp nhng im M trong mt phng sao cho hnh chiu ca
M trn ba cnh ca tam gic l ba im thng hng.
Bi tp 10: Cho on thng AB v M l im tu trn on AB. Dng trn cng mt na mt
phng b l ng thng AB cc hnh vung ANCD v BMEF. Cc ng trn ngoi tip chng
tm P v Q ct nhau ti M v N.
a) Chng minh rng: AE, BC i qua N.
b) Chng minh rng: MN i qua mt im c nh khi M di ng.
c) Tm tp hp trung im I ca PQ khi M di ng.
Bi tp 11: Cho ng trn (O; R) v mt im P c nh trong ng trn khng trng vi O. Qua
P dng dy cung APB, cc tip tuyn ca (O) ti A v B ct nhau ti M. Tm tp hp cc im M
khi dy AB quay quanh P.
Bi tp 12: Hai ng trn (O) v (O') giao nhau ti A v B. Mt ct tuyn di ng qua A ct (O)
ti C v (O') ti D. Tm tp hp tm I ca cc ng trn ni tip tam gic BCD.
Bi tp 13: Cho hnh vung ABCD c tm O. V ng thng (d) quay quanh O ct AD, BC ti E,
F. T E, F ln lt v cc ng thng song song vi DB, AC chng ct nhau ti I.
a) Chng minh rng I thuc mt ng thng c nh
b) T I k IH vung gc vi EF ti H. Chng minh H thuc mt ng c nh v IH i qua mt
im c nh.
www.VNMATH.com
.:: CHUYN N THI VO LP 10 ::.
Bin son: Trn Trung Chnh 90
Bi tp 14: Cho tam gic ABC c BC c nh cn A di ng sao cho gc BAC bng 60
0
. Tm qu
tch trng tm G ca tam gic ABC.
Bi tp 15: Cho ng trn (C) tm O. P l mt im c nh nm trong (C) nhng khng trng vi
O. Mt ng thng (d) thay i qua P ct (C) ti A v B. Tm qu tch trung im M ca on BC
khi (d) quay quanh P.
Bi tp 16: Cho hai im A, B c nh. C l mt im thay i trn on AB, C khc A v B. Dng
cc hnh vung ACDE v BCFG nm v cng mt pha i vi ng thng AB. Tm qu tch trung
im I ca EG.
Bi tp 17: Cho mt gc nhn Oxy v mt im M nm trong gc y. T M ta k cc ng vung
gc MH xung cnh Ox v MK xung cnh Oy. Tm tp hp cc im M tha mn iu kin MH +
MK = a, trong a l mt di cho trc.

Bi tp 18: Cho tam gic u ABC v mt im P nm trong tam gic. H PA
1
, PB
1
, PC
1
vung gc
vi BC, CA, AB tng ng. Tm tp hp cc im P sao cho A
1
B
1
C
1
l tam gic cn.
Bi tp 19: Cho tam gic u ABC. P l mt im nm trong tam gic. Gi x, y, z ln lt l
khong cch t P n cnh BC, CA, AB tng ng.
a) Bit rng x = 1, y = 2, z = 3. Hy tnh din tch tam gic ABC.
b) Tm qu tch nhng im P trong tam gic sao cho x + y = z. T suy ra tp hp nhng im P
trong tam gic sao cho x, y, z lp thnh 3 cnh ca mt tam gic.
Bi tp 20: Cho hai ng trn (C
1
) v (C
2
) ct nhau ti A v B. Mt ng thng (d) thay i
nhng lun i qua A ct (C
1
), (C
2
) ti cc im th hai C v D tng ng. Tm qu tch trung im
M ca CD khi (d) quay quanh A.
Bi tp 21: Cho ng trn (C) tm O bn knh R. ng trn (C
1
) c bn knh R/2 tip xc trong
vi (C) ti A. By gi ta c nh v tr im A trn ng trn (C
1
) l cho (C
1
) ln nhng lun tip
xc trong vi (C). Hy tm qu tch im A.
Bi tp 22 (*): Cho hai im A, B c nh, AB = 2a.
Tm qu tch nhng im M sao cho MA + MB = 2c khng i, vi c > a.
Bi tp 23: Cho hnh vung ABCD. M l mt im di ng trn cnh CD. AM v BM ko di ct
BC v AD ko di ti P v Q. DP ct CQ ti N. Tm qu tch im N khi M di ng trn cnh BC.
Bi tp 24: Cho tam gic ABC. Trn AB ko di v pha B ly im M v trn AC ko di v pha
C ly im N sao cho BM = CN. Tm qu tch trung im I ca MN.
Bi tp 25: Cho hai im A, B c nh. C l mt im thay i trn on AB, C khc A v B. Dng
cc hnh vung ACDE v BCFG nm v cng mt pha i vi ng thng AB. Gi I, J l tm cc
hnh vung ACDE v BCFG. Tm qu tch trung im K ca IJ.
Bi tp 26: Tm qu tch nhng im cch u mt im cho v mt ng thng cho.
Hng dn
Bi ton tng nh rt n gin ny khng th gii bng phng php hnh hc thun ty. Ta c th
dng mt s v tr thy rng qu tch khng phi l ng thng. Mt c im ng ch na l
trn 1 ng thng vung gc vi ng thng cho s tm c duy nht 1 im tha mn tnh
cht.
Bi ny c th gii c d dng bng phng php ta . Gi im cho l P v ng thng
cho l P. Xt h trc ta c Ox l ng thng d v Oy l ng thng qua im P v vung gc
vi d. Gi s P c tung l p > 0.
Xt im M(x, y) bt k nm trn qu tch. D thy y > 0. Khi khong cch t M n d l y v t
M n P l
2 2
x (y p) + .
T ta c
2
2 2 2 2 2 2
x p
y x (y p) y x y 2py p y
2p 2
= + = + + = + .
l phng trnh ca mt parabol!

.:: CHUYN N THI VO LP 10 ::.
Bin son: Trn Trung Chnh 91
CMI OI 15
OING MNM
1. Kin thc c bn:
Dng hnh bng thc v com-pa l dng ton kh i hi ngi gii phi nm vng cc kin thc
c bn, k nng cng nh s sng to trong vic k thm cc yu t ph kt ni cc d kin. V
th nm vng k nng dng hnh s c ngha quan trng trong vic gii ton hnh hc ni chung.
Bi ton dng hnh bng thc v compa c ngha ton hc rt su sc v ni dung ca n nhiu
lc vt ra khi lnh vc hnh hc. ng Vua ca cc nh Ton hc Carl Friederich Gauss rt t ho
vi kt qu tm ra cch dng a gic u 17 cnh ca mnh. Kt qu ny c c nh vo lng
gic, c th Gauss tnh c
17
360
cos
0
ch thng qua cc php tnh s hc v php khai cn bc 2.
gii bi ton dng hnh, ta i theo cc bc c bn sau:
Phn tch: Gi s hnh dng c, tm cch kt ni cc i tng bit vi cc i tng cn
dng bng nhng cu ni tm ra quy trnh dng: Bt u t mt thnh phn c th dng c,
tip tc dng ra cc thnh phn khc cho n khi hon thnh yu cu. V d php dng mt tam gic
s hon thnh khi ta dng c 3 nh ca n.

Cch dng: Nu ra cc bc dng c cu hnh tha mn yu cu bi ton. Mi bc dng
phi l nhng ng tc c th thc hin c bng thc v compa (k ng thng ni hai im, v
mt ng trn c tm v bn knh xc nh, tm giao im ca hau ng thng, hai ng trn
).
Chng minh: Chng minh cch dng va nu phn trn s cho ta cu hnh cn dng.
Bin lun: Bin lun s nghim ca bi ton theo cc iu kin ban u cho. Khi no v nghim,
khi no nghim duy nht, khi no c 2 nghim hnh
Kt lun: Tng kt li cc bc trn a ra kt lun.

Ta bit v hnh bng nhiu dng c: thc (thc thng), compa, ke, thc o gc,
Ta xt cc bi ton v hnh m ch s dng hai dng c l thc v compa, chng c gi l
cc bi ton dng hnh.
Vi thc, ta c th:
- V c mt ng thng khi bit hai im ca n.
- V c mt on thng khi bit hai u mt ca n.
- V c mt tia khi bit gc v mt im ca tia.
- Vi compa, ta c th v c mt ng trn khi bit tm v bn knh ca n.
hnh hc lp 6 v hnh hc lp 7, vi thc v compa, ta bit cch gii cc bi ton dng hnh
sau :
(1) Dng trung trc ca mt on thng.
Dng trung im ca mt on thng.
Dng mt ng thng i qua mt im cho v vung gc vi mt im cho.
(2) Dng mt ng thng i qua mt im cho v song song vi mt im cho.
(3) Dng mt on thng bng n ln on thng cho.
Dng mt on thng bng 1/n on thng cho.
(4) Dng mt gc bng gc cho. Chia i mt gc.
Dng tng v hiu ca hai gc.
(5) Cho hai on thng c di a, b, dng on thng c di ab .
(6) Dng tip tuyn k t mt im n mt ng trn.
(7) Dng ng trn ni tip, ngoi tip ca mt tam gic.
(8) Dng tam gic bit ba cnh, hoc bit hai cnh v gc xen gia, hoc bit mt cnh v hai gc
k.
www.VNMATH.com
.:: CHUYN N THI VO LP 10 ::.
Bin son: Trn Trung Chnh 92
Dng hnh bng phng php i s:
Gii mt bi ton dng hnh bng phng php i s thng c quy v dng mt s on thng.
Ta gi cc di cc on thng phi tm l x, y, z. Sau ta lp phng trnh biu th mi tng
quan gia cc on thng bit l a, b, c. Sau gii h phng trnh c cc n x, y, z.
Mt vi on thng dng c biu th bng biu thc n gin l:
x = a b ; x =
a.b.c
e.f

x = na, n e N ; x =
2 2 2 2
a b c d + (a
2
+ d
2
> b
2
+ c
2
)
x =
a
n
, n e N ; x =
2 2
a b
x =
na
m
; m, n e N ; x = ab
x =
ab
c
; x = a n ; n e N
Dng hnh bng phng php bin hnh:
Dng hnh bng phng php bin hnh l p dng php i xng, php tnh tin, php quay, ng
dng. Ta quy vic dng mt hnh v vic dng mt im M. Dng trc tip im M i khi gp kh
khn. Trong trng hp ny ta chn mt php bin hnh l mt song nh f ( f c nh x ngc)
bin im M thnh im M' m im M' ny ta c th ng c mt cch d dng. Sau khi dng
c im M' ta c php bin hnh ngc: f
-1
(M') = M. V d nh tnh tin a

.

2. Bi tp p dng:
Bi tp 1: Dng AABC bit cnh BC = a, ng cao AH = h, trung tuyn AM = m.
Gii
Phn tch
Gi s ta dng c AABC tho mn:
BC = a; AH = h; AM = m.
Ta phi xc nh nh A tho mn 2 iu kin:
- A cch BC mt khong bng h, suy ra A e ng thng d// BC
v cch BC mt khong h.
- A cch im M l trung im ca BC mt khong m.
Cch dng
- Dng BC bng a
- Dng ng thng d // BC v cch BC mt
khong bng h.
- Dng ng trn tm M bn
knh m ct d ti A.
AABC l tam gic cn dng.
Chng minh
AABC c BC = a (cch dng)
ng cao AH = h (cch dng)
Trung tuyn AM = m (cch dng)
AABC l tam gic cn dng.
Bin lun
* m > h bi ton c 4 nghim (4 im A)
* m = h bi ton c 2 nghim (2 im A)
* m < h bi ton v nghim (khng c im A)

m
h
C
M H B
A
d
m
h
C
M H B
A
.:: CHUYN N THI VO LP 10 ::.
Bin son: Trn Trung Chnh 93
Bi tp 2: Cho ng thng m song song vi ng thng n v im A khng thuc 2 ng thng
. Dng im B e m, C e n sao cho AABC l tam gic u.
Gii
Phn tch
Gi s dng c im B e m, im C e n AABC u.
Dng hnh chiu vung gc ca A trn im M l E
Dng tam gic u AEF.
Xt AAEB v AAFC ta c:
AE = AF (AABF u)

( )
0
CAF BAE 60 CAE = = +
AB = AC (AABC u)
AAEB = AAFC (c.g.c)


0
BEA CFA 90 = = (v AE BE)
Cch dng
T A h AE m ti E
- Dng AAEF u
- T F dng ng vung gc vi AF ct n ti C
- Ni A vi C, dng ng trn tm A bn knh AC ct
m ti B.
- Ni A vi B, B vi C ta c AABC cn dng
Chng minh
Xt A vung ABE v A vung ACF c:
AB AC
AE AF
=
`
=
)
(Cch dng) AABF = AACF (c.g.c)
AE = AF


BAE CAF =
M

0
CAF EAF CAE 60 CAE = + = +
V

BAE BAC CAE = +

0
BAC 60 =
AABC c: AB = AC v

0
BAC 60 =
AABC u
d) Bin lun
Bi ton c 2 nghim v ta c th dng c 2 A u
Bi tp 3: Dng AABC bit BC = a; AB + AC = d;

ABC= o.
Gii
a) Phn tch
Gi s ta dng c AABC tho mn cc iu kin ca u bi.
Ko di BA v trn ng ko di ly im D sao cho AD = AC.
Suy ra: BD = AB + AD = AB + AC = d
ADAC cn A = BD ng trung trc ca CD
b) Cch dng
- Dng on BC = a
- Dng tia Bx sao cho

xBC= o .
- Dng im D trn Bx sao cho BD = d
- Ni D vi C.
B
C
F
E
A
n
m
A
D

C B
www.VNMATH.com
.:: CHUYN N THI VO LP 10 ::.
Bin son: Trn Trung Chnh 94
- Dng im A l giao ca BD v ng trung trc ca CD.
- Ni A vi C ta c AABC cn dng.
c) Chng minh
ABC = o (cch dng)
BC = a (cch dng)
A e ng trung trc ca DC AD = AC
A, D e Bx; BD = d (cch dng)
BD = AB + AD = AB + AC = d
AABC l A cn dng.
d) Bin lun
- d < a bi ton v nghim
- d > a Bi ton c mt nghim
Bi tp 4: Dng AABC bit BC = a, trung tuyn AM = m, ng cao CH = h.
Gii
Phn tch:
Gi s dng c AABC tho mn iu kin ca u bi
A e ng trn tm M bn knh m.
H e ng trn ng knh BC
CH = h; B, H, A thng hng
Cch dng:
- Dng BC = a, trung im M ca BC
- Dng ng trn (M, m)
- Dng ng trn ng knh BC
- Dng im H e ng trn ng knh BC sao cho HC = h
- Dng im A l giao im ca BH v (M, m)
Chng minh:
BC = a
CH = h (cch dng)
A e (M, m) AM = m
AABC l tam gic cn dng
Bin lun:
Bi ton c nghim khi
h < BC = a
2m> h


Bi ton c hai nghim do BH ct (M, m) ti hai im l A v A'.
Bi tp 5: Dng AABC bit B = | < 90
0
, ng cao BH v ng cao AD.
Gii
Phn tch:
Gi s AABC dng c.
A vung ABD l dng c
ta ch cn dng im C.
Mun vy ta phi i dng im H: H e giao ca hai ng trn
ng knh AB v ng trn tm B bn knh BH C = AH
BD
Cch dng:
- Dng AABD vung ti D
sao cho ABD < 90
0

v AD cho trc.
- Dng im H l giao im
B'
m
h
A
H
M
C
B
H
C
D
B
A
.:: CHUYN N THI VO LP 10 ::.
Bin son: Trn Trung Chnh 95
ca hai ng trn: (B, BH)
v ng trn ng knh AB (BH cho trc).
- Dng im C l giao ca BD v AH AABC l A ta cn dng.
Chng minh:
ABD = | < 90
0
(cch dng)
AD l ng cao c di cho trc (cch dng)
BH bng on cho trc (cch dng)
AABC tho mn yu cu ca bi
Bin lun:
Bi ton lun c nghim
Bi ton c mt nghim
Bi tp 6: Dng hnh bnh hnh ABCD bit 2 nh i din A v C cn 2 nh B v D thuc mt
ng trn (O, R) cho trc.
Gii
Phn tch:
Gi s dng c hnh bnh hnh tho mn iu kin ca bi l ABCD. Nu I l giao im ca
2 ng cho ca ABCD th: I e AC v IA = IC, I e BD v IB = ID; B, D e (O,R) OI BD
Cch dng:
- Dng I l trung im ca AC
- Dng ng thng qua I
v OI ct (O) ti B v D
ABCD l hnh bnh hnh cn dng.
Chng minh:
OI BD IB = ID
IA = IC (cch dng); B, D e (O, R) (cch dng)
AAIB = ADIC (c.g.c) ABI = IDC AB // CD
ABCD l hnh bnh hnh tho mn u bi.
Bin lun:
Bi ton c nghim khi im I trong ng trn (O) khi bi ton c 1 nghim.
Bi tp 7: Cho ng trn (O, R) v im A e ng thng d.
Dng ng trn tip xc vi C(O,R) v tip xc vi d ti A.
Gii
Phn tch:
Gi s dng c (O',R') tip xc vi (O, R) v tip xc
vi d ti A O' e d' l ng thng qua A v vi d.
Dng im E sao cho O'E = O'O (AE = R).
O' nm trn ng trung trc ca OE
O' l giao ca ng trung trc ca OE & p
Cch dng:
- Dng ng thng d' d ti A
- Dng im E e d' sao cho AE = R
- Dng ng trung trc ca
OE l m, m d' O'
- Dng ng trn (O',O'A)
l ng trn cn dng
Chng minh:
(O', O'A) tip xc vi d ti A (cch dng)
Ni O vi O'. V O' e ng trung trc ca OE
OO' = O'E
C
A
I
B
D
O
E
O
O'
A
d
d'
www.VNMATH.com
.:: CHUYN N THI VO LP 10 ::.
Bin son: Trn Trung Chnh 96
M O'E = O'A + AE OO' = OA + AE = O'A +R
(O, R) & (O', O'A) tip xc vi nhau
(O') l ng trn cn dng
Bin lun:
Trn p c th ly E
1
trong ng trn (O') sao cho AE
1
= R.
Vy bi ton c 2 nghim hnh.
Bi tp 8: Cho hnh thang ABCD, AD // BC. Dng ng thng EF//BC chia i din tch hnh
thang.
Gii
Phn tch:
Gi s dng c EF//BC chia i din tch hnh thang ko di BC, CD ct nhau ti O.
Suy ra:
AOBC AOEF AOAD
t OB = a, OA = b, OE = x
Ta c:
2 2
OBC OAD
2 2
OEF OEF
S S a b
;
S x S x
A A
A A
= =

2 2
OBC OAD
2
OEF
S S a b
S x
A A
A
+ +
=
M: S
AOBC
+ S
AOAD
= S
AOEF
+ S
hnh thang EBCF
+ S
AOAD

= S
AOEF
+ S
hnh thang AEFD
+ S
AOAD
= 2S
AOEF

2 2
2
a b 2
x 1
+
= 2x
2
= a
2
+ b
2

2 2
2
a b
x
2 2
= +
t
2 2
2 2
a b
y ; z x y z
2 2
= = = +
z
y
x
z
b
2
b a
2
a


Cch dng:
- Ko di BA, CD ct nhau O
- Dng on trung bnh nhn ca a,
a
2
ta c y.
- Dng on trung bnh nhn ca
b
2
, b ta c z.
- Dng A vung c y, z l 2 cnh gc vung
di cnh huyn ca A l x.
- Trn OB ly OE = x, dng EF // BC ta s c on EF cn dng.
.:: CHUYN N THI VO LP 10 ::.
Bin son: Trn Trung Chnh 97
Chng minh:
Gi hnh thang ADEF din tch l S
1
v hnh thang EBCF c
din tch l S
2

Ta phi chng minh S
1
= S
2

Ta c AOAD ADEF (v AD//EF)
T s ng dng l:
a
x


2
OAD 0
2
OEF 0 1
S S a
S x S S
A
A
= =
+

AOEF AOBC
2
OBC 0 1 2
2
OEF 0 1
S S S S b
S x S S
A
A
+ +
= =
+


2 2 2 2
0 1 2 0 1 2
2 2 2
0 1 0 1
2S S S 2S S S a b a b
a b x S S S S
2
+ + + + + +
= =
+ + +

0 1 2
0 1 2 0 1 1 2
0 1
2S S S
2 2S S S 2S 2S S S
S S
+ +
= + + = + =
+

S
hnh thang ADEF
= S
hnh thang EBCF
Bin lun:
Bi ton lun c mt nghim hnh.
Bi tp 9: Cho hnh bnh hnh ABCD. Dng hai ng thng i qua nh A v chia hnh bnh hnh
thnh 3 phn c din tch bng nhau.
Gii
Phn tch:
Gi s dng c ng thng qua A ct BC ti E, ct CD ti F tho mn:
S
AABE
= S
'BECF
= S
AAFD
=
3
1
S
'ABCD

Gi di: BE = x, ng cao AH = h S
AABE
=
2
1
h.x
S
'ABCD
= AH.BC = h.BC. M S
'ABCD
= 3 S
AABE
h.BC = 3.
2
1
hx <=> BC =
2
3
x x =
3
2
BC
Tng t ta gi: DF = y y =
3
2
DC
Cch dng:
- Dng on BE =
3
2
BC
- Dng on DF =
3
2
DC
- Ni A vi E, A vi F ta c:
S
AABE
= S
AAFD
= S
'AECF
=
3
1
S
'ABCD
Chng minh:
Ta c: S
AABE
=
2
1
hx =
2
1
h.
3
2
BC =
3
1
h.BC =
3
1
S
'ABCD
F
b
x
E
a
O
D
C
B
A
F
E
B
C
D A
www.VNMATH.com
.:: CHUYN N THI VO LP 10 ::.
Bin son: Trn Trung Chnh 98
Tng t: S
AADF
=
3
1
S
'ABCD
S
'AECF
=
3
1
S
'ABCD
iu phi chng minh
Bin lun:
Bi ton c mt nghim hnh
Bi tp 10: Cho 2 im A, B nm v mt pha ca ng thng d.
Tm im M e d sao cho AM + MB l nh nht.
Gii
Phn tch:
Gi s dng c im M e d (AM + MB) ngn nht.
Ta ly im A' i xng vi A qua d.
IA = IA'; MA = MA' (AM + MB) ngn nht khi: A, M, B
thng hng.
M e giao ca ng thng ni 2 im A', B v ng thng
d.
Cch dng:
- Dng im A' i xng A qua d
- Ni A' vi B
- Dng M = A'B d
l im M cn dng
Chng minh:
- Ly M' e d (M' tu ) v ta chng minh:
M'A + M'B > MA + MB
Theo cch dng th A', M, B thng hng v AM = A'M
Xt AA'BM' ta c: M'A + M'B > A'B (1)
M theo cch dng th A'B = MA' + MB = MA + MB (2)
T (1) v (2), suy ra:
MA' + MB' > MA + MB (MA + MB) min (pcm)
Bin lun:
Bi ton c 1 nghim hnh v im A' dng c l duy nht.
Bi tp 11: Cho 2 ng thng b // c, im A e b, c. Dng AABC u sao cho B e b, Cec.
Gii
Phn tch:
Gi s ta dng c AABC u tho mn iu kin ca bi ton.
B e b, C e c.
Ta thc hin php quay theo chiu kim ng h ta c:
r(A, 60
0
)(B) = C;
r(A, 60
0
)(b) = b'
M B e b C e b'.
Mt khc: C e c
c b' = C
Cch dng:
- Dng ng thng
b' = r(A, 60
0
)(b)
- Dng im C
l giao im ca b' v c
- Dng im B bng cch:
r(A, 60
0
)(C) = B
M' M
B
A'
A
d
b'
C' C
B' B
A
c
b
.:: CHUYN N THI VO LP 10 ::.
Bin son: Trn Trung Chnh 99
Chng minh:
r(A, -60
0
)(C) = B; r(A, -60
0
)(b') = b
M C e b' B e b (pcm).
Bin lun:
Bi ton c 2 nghim hnh
Bi tp 12: Cho AABC. Dng hnh vung MNPQ sao cho M e AB; N,P e BC, Q e AC.
Gii
Phn tch:
Gi s dng c hnh vung MNPQ tho mn iu kin ca bi ton.
Ni B vi Q v thc hin php v t: V(B, k =
BQ'
BQ
) (Q' e BQ): Q Q'; M M'; N N'; P
P'
M' Q' N' M' N' P' P' Q'
MQ NM NP PQ
= = =
M MQ = MN = NP = PQ v NMQ = 90
0

M'Q' = M'N' = N'P' = P'Q'; N'M'Q' = 90
0

'M'N'P'Q' l hnh vung.
Cch dng:
- Ly M' e AB, dng M'N' BC
- Dng hnh vung M'N'P'Q'
- K BQ' ct AC ti Q
- Thc hin php v t:
V(B; k =
BQ'
BQ
) (Q') = Q; p' p; M' M; N' N
ta dng c hnh vung MNPQ cn dng.
Chng minh:
Theo cch dng ta c:
MQ NM NP PQ
M' Q' N' M' N' P' P' Q'
= = = v t gic M'N'P'Q' l hnh vung;

0
N' M' P' 90 = .
MN = NP = PQ = MQ & NMP = 90
0

^MNPQ l hnh vung
Bin lun:
Bi ton c 1 nghim hnh

Bi tp 13: Dng tam gic bit di ba ng trung tuyn.
Gii
Phn tch:
Gi s AABC dng xong v c trung tuyn:
AM = m
a
, BN = m
b
, CP = m
c
.
Nhn vo hnh v ta cha thy c yu t no c th dng c,
tr cc on thng AM, BN, CP mt cch ring l.
V d nhin, nu ta dng, chng hn AM th c th xc nh
thm c G.
Tuy nhin, nu ta gi E l trung im ca AG th do
BG BN AG AM
PE ; EG
2 3 2 3
= = = = v
CP
PG
3
= (tnh cht
ng trung tuyn v tnh cht ng trung bnh) nn cc cnh ca APEG hon ton xc nh.
Khi xc nh c APEG, ta d dng xc nh c cc im C, A, M v cui cng l B.
Q'
Q
P P' M' N'
M'
M
C
B
A
P
E
G
C
N
B
M
A
www.VNMATH.com
.:: CHUYN N THI VO LP 10 ::.
Bin son: Trn Trung Chnh 100
T suy ra cch dng.
Cch dng:
- Dng APEG c:
b c a
m m m
PE ; PG ; EG
3 3 3
= = = .
- Ni di PG v pha G, trn dng C sao cho GC = 2GP;
- Ni di GE v pha E, trn dng A sao cho EA = EG;
- Ni di EG v pha G, trn dng M sao cho GM = GE;
- Ni AP v MC ct nhau ti B.
AABC chnh l tam gic cn dng.
Chng minh:
Theo cch dng trn th AM = m
a
v CP = m
c
.
Cng theo cch dng v tnh cht ng trung tuyn th G chnh l trng AABC.
Do BG l ng trung tuyn.
V PE l ng trung bnh trong tam gic ABG nn BG = 2PE =
b
2m
3
.
Suy ra ng trung tuyn k t B bng m
b
.
Nh vy ta c AABC c ba trung tuyn bng vi m
a
, m
b
, m
c
.
Bin lun:
Bc dng th nht dng c khi
a b c
m m m
; ;
3 3 3
l di 3 cnh ca mt tam gic.
iu ny tng ng vi m
a
, m
b
, m
c
l di 3 cnh ca mt tam gic.
Cc bc dng tip theo u thc hin c mt cch duy nht.
Suy ra nu di 3 on thng cho l di 3 cnh ca mt tam gic th bi ton c 1 nghim
hnh.
Trong trng hp ngc li bi ton v nghim.
Ghi ch: T bi ton dng hnh ni trn, ta suy ra mt kt qu th v sau: Ba ng trung tuyn ca
tam gic ABC l di 3 cnh ca mt tam gic c din tch bng 3/4 din tch tam gic ABC.

Bi tp 14: Cho hai ng trn (C
1
), (C
2
) c bn knh R
1
< R
2
ct nhau ti A v B. Hy dng tip
tuyn chung ca hai ng trn.
Gii
Phn tch:
Gi s tip tuyn chung tip xc (C
1
) ti M v (C
2
) ti N.
Ni di NM ct ng thng O
1
O
2
ti P.
V O
1
M v O
2
N u vung gc vi MN nn chng song song vi
nhau.
Theo nh l Talet ta c
1 1 1
2 2 2
PO O M R
PO O N R
= = nn t y ta dng
c im P.
V PMO
1
= 90
0
nn M nm trn ng trn ng knh PO
1
.
Nh vy M l giao im ca ng trn ng knh PO
1
v (C
1
).
Cch dng:
- Dng im P trn O
2
sao cho
1 1
2 2
PO R
PO R
=
- Dng ng trn ng knh PO
1
;
- ng trn ng knh PO
1
ct (C
1
) ti M;
- Ni PM, l tip tuyn chung cn dng.
Chng minh:
Theo bc 2, 3 th PM vung gc vi MO
1
.
N
M
O
2
O
1
B
A
.:: CHUYN N THI VO LP 10 ::.
Bin son: Trn Trung Chnh 101
Suy ra PM l tip tuyn ca (C
1
).
T O
2
k O
2
N vung gc vi PM th O
2
N//O
1
M.
p dng nh l Talet ta c:
1 1
2 2
PO O M
PO O N
= .
Theo bc 1 th ta c:
1 1
2 2
PO R
PO R
= .
T hai ng thc cui, vi ch O
1
M = R
1
, ta c O
2
N = R
2
, tc l im N nm trn (C
2
).
Suy ra PM tip xc (C
2
) ti N, tc l PM chnh l tip tuyn chung ca hai ng trn.
Bin lun: Bi ton lun c 2 nghim hnh (HS t chng minh).
3. Bi tp t luyn:
Bi tp 1: Cho trc mt on thng c di bng 1, hy dng cc on thng c di bng
a) 2; b)
2
1
; c)
3
1
; d)
5
1
; e) 2 ; f) 5 ; g)
4
2
Bi tp 2: Dng AABC c = 52
0
, AB = 5cm, AC = 7 cm
Bi tp 3: Dng AABC c - 60
0
, AB = 3cm, AC + BC = 7,5 cm.
Bi tp 4: Dng AABC c = 90
0
, phn gic AD = 10 cm, ng cao AH = 6 cm.
Bi tp 5: Dng AABC c = 60
0
, AB = 3cm, ng cao AH = 2cm.
Bi tp 6: Dng tam gic bit b, a + c v C.
Phn tch: Gi s AABC dng c. Ni di CB v pha B ti im D sao cho BD = BA. Khi
tam gic ACD c gc C cho, AC = b v CD = a + c nn hon ton xc nh. nh B l nh ca
tam gic cn BDA, do l giao im ca trung trc on AD vi CD.
Bi tp 7: Cho hai ng thng a // b v mt im C. Hy dng tam gic u ABC c A nm trn a
v B nm trn b.
Gi : Hy chn mt s im A ty trn A ri dng tam gic u ABC. Ch xem B s vch ra
ng g?
Bi tp 8: Dng tam gic ABC bit di ng trung tuyn, ng phn gic v ng cao k t
nh A.
Cu hi gi : ng phn gic gc A v ng trung trc cnh BC ct nhau u?
Bi tp 9: Cho t gic ABCD. T A hy k mt ng thng chia i din tch tam gic.
Cu hi gi : Nu t gic ABCD suy bin thnh tam gic ABC th v nh th no?
Bi tp 10: Dng tam gic bit a, b v m
a
.
Bi tp 11: Dng tam gic c chu vi 2p, gc A v ng cao h
a
.
Bi tp 12: Dng t gic bit di 4 cnh lin tip v on ni trung im hai ng cho.
Bi tp 13: Cho bit
4
5 1
) 72 cos(
0
+
= . Hy nu cch dng ng gic u cnh bng a cho trc.
Bi tp 14: Cho ng thng (d) v hai im A, B nm cng mt pha i vi d. Hy dng ng
trn i qua A, B v tip xc vi (d).
Bi tp 15: Nu cch dng trc ng phng ca hai ng trn trong cc trng hp sau
a) Hai ng trn ct nhau
b) Hai ng trn ngoi nhau
c) Hai ng trn cha nhau
Bi tp 16: Cho tam gic ABC. Hy nu cch dng ng thng chia tam gic thnh 2 phn c din
tch v chu vi bng nhau.
Bi tp 17: Cho hai ng trn (O
1
, R
1
) v (O
2
, R
2
) v phng A. Dng on AB = a song song vi
A sao cho A e (O
1
, R
1
), B e (O
2
, R
2
).
Bi tp 18: Cho hai ng trn (O
1
, R
1
) v (O
2
, R
2
) cng ng thng d. Dng hnh vung ABCD
sao cho A e (O
1
, R
1
), C e (O
2
, R
2
); B, D e d.
Bi tp 19: Dng mt A u sao cho din tch ca n bng din tch mt A cho trc
www.VNMATH.com
.:: CHUYN N THI VO LP 10 ::.
Bin son: Trn Trung Chnh 102
Bi tp 20: Cho hai im A, B nm cng pha vi ng thng d. Dng ng trn i qua A, B v
tip xc vi d.
Bi tp 21: Cho hai im A, B e ng thng d cho trc. Dng ng trn i qua hai im A, B
v tip xc vi ng thng d.
Bi tp 22: Dng hai ng thng i qua A chia hnh bnh hnh thnh 3 phn bng nhau v din
tch.
Bi tp 23: Cho AABC, dng ng thng song song vi BC chia AABC thnh hai phn c din
tch bng nhau.
Bi tp 24: Cho ng trn (O, R) v hai im A, B e (O, R) cng mt on thng bit l. Dng
hai dy cung song song i qua A v B sao cho tng ca chng bng l.
Bi tp 25: Cho im A ngoi (O, R).
Dng ct tuyn i qua A ct (O, R) ti B v C sao cho AB = BC.
Bi tp 26: Cho ng trn (O) v mt dy cung AB c nh. Dng A u MNP tho mn: M & P
e (O); N e AB v MN AB.
Bi tp 27: Cho hnh vung ABCD c giao im hai ng cho l 0. hy dng nh ca cc im
A, B, C, D trong php quay tm O mt gc 45
0
ngc chiu kim ng h.
Bi tp 28: Dng mt hnh vung ni tip mt ng trn bn knh R, dng mt lc gic v mt
tam gic u ni tip ng trn bn knh R.

BAI TAP TONG MOP HIIN TMIC
Bi tp 1: Cho AABC c cc ng cao BD v CE.ng thng DE ct ng trn ngoi tip tam
gic ti hai im M v N.
a) Chng minh: T gic BEDC ni tip.
b) Chng minh:

DEA ACB = .
c) Chng minh: DE // vi tip tuyn tai A ca ng trn ngoi tip tam gic.
d) Gi O l tm ng trn ngoi tip tam gic ABC. Chng minh: OA l phn gic ca gc MAN
e) Chng t: AM
2
= AE.AB.
Bi tp 2: Cho ng trn (O), ng knh AC. Trn on OC ly im B v v ng trn (O),
ng knh BC. Gi M l trung im ca on AB. T M v dy cung DE AB; DC ct ng trn
(O) ti I.
a) T gic ADBE l hnh g?
b) Chng minh: T gic DMBI ni tip.
c) Chng minh: Ba im B; I; C thng hng v MI = MD.
d) Chng minh: MC.DB = MI.DC.
e) Chng minh: MI l tip tuyn ca ng trn (O).
Bi tp 3: Cho AABC c gc A = 90
0
. Trn AC ly im M sao cho AM < MC. V ng trn (O),
ng knh CM. ng thng BM ct (O) ti D. Ko di AD ct (O) ti S.
a) Chng minh: T gic BADC ni tip.
b) K BC ct (O) ti E. Chng minh rng: MR l phn gic ca

AED.
c) Chng minh: CA l phn gic ca gc

BCS.
Bi tp 4: Cho AABC c gc A = 90
0
. Trn cnh AC ly im M sao cho AM>MC. Dng ng
trn (O) ng knh MC. ng trn ny ct BC ti E. ng thng BM ct (O) ti D v ng
thng AD ct (O) ti S.
a) Chng minh: T gic ADCB ni tip.
b) Chng minh: ME l phn gic ca

AED.
c) Chng minh: Gc

ASM ACD = .
d) Chng t ME l phn gic ca

AED.
e) Chng minh: Ba ng thng BA; EM; CD ng quy.
.:: CHUYN N THI VO LP 10 ::.
Bin son: Trn Trung Chnh 103
Bi tp 5: Cho tam gic ABC c 3 gc nhn v AB < AC ni tip trong ng trn tm O. K
ng cao AD v ng knh AA. Gi E; F theo th t l chn ng vung gc k t B v C
xung ng knh AA.
a) Chng minh: T gic AEDB ni tip.
b) Chng minh: DB.AA = AD.AC.
c) Chng minh: DE AC.
d) Gi M l trung im BC. Chng minh: MD = ME = MF.
Bi tp 6: Cho AABC c ba gc nhn ni tip trong ng trn (O). Gi M l mt im bt k trn
cung nh AC. Gi E v F ln lt l chn cc ng vung gc k t M n BC v AC. Gi P l
trung im AB; Q l trung im FE.
a) Chng minh: T gic MFEC ni tip.
b) Chng minh: BM.EF = BA.EM.
c) Chng minh: AAMP AFMQ.
d) Chng minh:

0
PQM 90 = .
Bi tp 7: Cho (O) ng knh BC. Ly im A bt k nm trn cung BC. Trn tia AC ly im D
sao cho AB = AD. Dng hnh vung ABED; AE ct (O) ti im th hai F. Tip tuyn ti B ct
ng thng DE ti G.
a) Chng minh: T gic BGDC ni tip. Xc nh tm I ca ng trn ny.
b) Chng minh: ABFC vung cn v F l tm ng trn ngoi tip ABCD.
c) Chng minh: T gic GEFB ni tip.
d) Chng t: C; F; G thng hng v G cng nm trn ng trn ngoi tip ABCD. C nhn xt g
v I v F?
Bi tp 8: Cho AABC c 3 gc nhn ni tip trong ng trn (O). Tip tuyn ti B v C ca ng
trn ct nhau ti D. T D k ng thng song song vi AB, ng ny ct ng trn E v F, ct
AC I (E nm trn cung nh BC).
a) Chng minh: T gic BDCO ni tip.
b) Chng minh: DC
2
= DE.DF.
c) Chng minh: T gic DOIC ni tip.
d) Chng t I l trung im EF.
Bi tp 9: Cho ng trn (O), c dy cung AB. T im M bt k trn cung AB (M = A v M =
B). K dy cung MN AB ti H. Gi MQ l ng cao ca tam gic MAN.
a) Chng minh: 4 im A; M; H; Q cng nm trn mt ng trn.
b) Chng minh: NQ.NA = NH.NM.
c) Chng minh: MN l phn gic ca gc BMQ.
d) H on thng MP vung gc vi BN. Xc nh v tr ca M trn cung AB MQ.AN+MP.BN
c gi tr ln nht.
Hng dn
d) Xc nh v tr ca M trn cung AB MQ.AN + MP.BN c gi tr ln nht:
Ta c:
MAN
MBN
MAN MBN
2S MQ.AN
2S MP.BN
2S 2S MQ.AN MP.BN
A
A
A A
=
=
+ = +

Ta li c:
2S
AMAN
+ 2S
AMBN
= 2(S
AMAN
+ S
AMBN
) = 2S
AMBN
=
AB.MN
2. AB.MN
2
= .
Vy: MQ.AN + MP.BN = AB.MN
M AB khng i nn tch AB.MN ln nht MN ln nht MN l ng knh M l im
chnh gia cung AB.
www.VNMATH.com
.:: CHUYN N THI VO LP 10 ::.
Bin son: Trn Trung Chnh 104
Bi tp 10: Cho ng trn (O; R) v (I; r) tip xc ngoi ti A (R > r). Dng tip tuyn chung
ngoi BC (B nm trn ng trn (O) v C nm trn ng trn (I)). Tip tuyn BC ct tip tuyn
ti A ca hai ng trn E.
a) Chng minh tam gic ABC vung A.
b) K OE ct AB N; IE ct AC ti F.
Chng minh: N; E; F; A cng nm trn mt ng trn.
c) Chng t rng: BC
2
= 4Rr.
d) Tnh din tch t gic BCIO theo R; r.
Hng dn
c) Chng minh: BC
2
= 4Rr.
Ta c t gic FANE c 3 gc vung (cmt)
FANE l hnh vung
AOEI vung E v EA OI (tnh cht tip tuyn).
p dng h thc lng trong tam gic vung c:
AH
2
= OA.AI (bnh phng ng cao bng tch hai hnh chiu)
M AH =
BC
2
v OA = R; AI = r
2
BC
4
= Rr BC
2
= Rr.
d) S
BCIO
= ?
Ta c BCIO l hnh thang vung
S
BCIO
=
OB IC
.BC
2
+

S =
(r R) rR
2
+
.
Bi tp 11: Trn hai cnh gc vung xOy ly hai im A v B sao cho OA = OB. Mt ng thng
qua A ct OB ti M (M nm trn on OB). T B h ng vung gc vi AM ti H, ct AO ko
di ti I.
a) Chng minh: T gic OMHI ni tip.
b) Tnh

OMI .
c) T O v ng vung gc vi BI ti K. Chng minh: OK = KH.
d) Tm tp hp cc im K khi M thay i trn OB.
Hng dn
d) Tp hp cc im K:
Do OK KB
Suy ra:

OKB = 90
0
.
OB khng i khi M di ng K nm trn ng trn ng knh OB.
Khi M O th K O.
Khi M B th K l im chnh gia cung AB.
Vy qu tch im K l
1
4
ng trn ng knh OB.
Bi tp 12: Cho ng trn (O) ng knh AB v dy CD vung gc vi AB ti F. Trn cung BC
ly im M. Ni A vi M ct CD ti E.
a) Chng minh: AM l phn gic ca gc CMD.
b) Chng minh: T gic EFBM ni tip.
c) Chng t: AC
2
= AE.AM.
d) Gi giao im CB vi AM l N; MD vi AB l I. Chng minh: NI // CD.
e) Chng minh: N l tm ng trn ni tip ACIM
Hng dn
e) Chng t N l tm ng trn ni tip AICM:
.:: CHUYN N THI VO LP 10 ::.
Bin son: Trn Trung Chnh 105
Ta phi chng minh N l giao im 3 ng phn gic ca ACIM.
Theo chng minh, ta c MN l phn gic ca

CMI .
Do MNIB ni tip (cmt)

NIM NBM = (cng chn cung MN)
Gc

MBC MAC = (cng chn cung CM)
Ta li c:

0
CAN 90 = (gc ni tip

0
ACB 90 = );

0
NIA 90 = (v

0
NIB 90 = )
Suy ra: ACNI ni tip


CAN CIN = (cng chn cung CN)


CIN NIM =
IN l phn gic

CIM.
Vy N l tm ng trn ni tip AICM.
Bi tp 13: Cho ng trn (O) v im A nm ngoi ng trn. V cc tip tuyn AB;AC v ct
tuyn ADE. Gi H l trung im DE.
a) Chng minh: A; B; H; O; C cng nm trn 1 ng trn.
b) Chng minh: HA l phn gic ca gc

BHC.
c) Gi I l giao im ca BC v DE. Chng minh: AB
2
= AI.AH.
d) K BH ct (O) K. Chng minh: AE//CK.
Bi tp 14: Cho ng trn (O) ng knh AB = 2R; xy l tip tuyn vi (O) ti B. CD l 1 ng
knh bt k. Gi giao im ca AC; AD vi xy theo th t l M; N.
a) Chng minh: T gic MCDN ni tip.
b) Chng t: AC.AM = AD.AN
c) Gi I l tm ng trn ngoi tip t gic MCDN v H l trung im MN. Chng minh: AOIH l
hnh bnh hnh.
d) Khi ng knh CD quay xung quanh im O th I di ng trn ng no?
Hng dn
d) Qu tch im I:
Do AOIH l hnh bnh hnh.
Suy ra: IH = AO = R khng i
CD quay xung quanh O th I nm trn ng thng song song vi xy v cch xy mt khong
bng R.
Bi tp 15: Cho tam gic ABC ni tip trong ng trn tm O. Gi D l 1 im trn cung nh BC.
K DE; DF; DG ln lt vung gc vi cc cnh AB; BC; AC. Gi H l hnh chiu ca D ln tip
tuyn Ax ca (O).
a) Chng minh: T gic AHED ni tip.
b) Gi giao im ca AH vi HB v vi (O) l P v Q; ED ct (O) ti M.
Chng minh: HA.DP = PA.DE.
c) Chng minh: QM = AB.
d) Chng minh: DE.DG = DF.DH.
e) Chng minh: E; F; G thng hng. (ng thng Sim sn)
Hng dn
e) Chng minh: E; F; G thng hng:
Ta c:

BFE BDE = (cmt) v

GFC CDG = (cmt)
Do ABCD ni tip.
Suy ra:

0
BAC BMC 180 = =
Do GDEA ni tip
Suy ra:

0
EDG EAG 180 + = .
www.VNMATH.com
.:: CHUYN N THI VO LP 10 ::.
Bin son: Trn Trung Chnh 106


EDG BDC =
M

EDG EDB BDG = + v

BCD BDG CDG = +


EDB CDG =


GFC BEF =
Vy E; F; G thng hng.
Bi tp 16: Cho tam gic ABC c A = 90
0
; AB < AC. Gi I l trung im BC. Qua I k IKBC (K
nm trn BC). Trn tia i ca tia AC ly im M sao cho MA = AK.
a) Chng minh: T gic ABIK ni tip c trong ng trn (O).
b) Chng minh:

BMC 2ACB = .
c) Chng t rng: BC
2
= 2AC.KC.
d) Ko di AI ct ng thng BM ti N. Chng minh AC = BN.
e) Chng minh: T gic NMIC ni tip.
Bi tp 17: Cho (O) ng knh AB c nh. im C di ng trn na ng trn. Tia phn gic
ca

ACB ct (O) tai M. Gi H; K l hnh chiu ca M ln AC v AB.


a) Chng minh: T gic MOBK ni tip.
b) Chng minh: T gic CKMH l hnh vung.
c) Chng minh: Ba im H; O; K thng hng.
d) Gi giao im HK v CM l I. Khi C di ng trn na ng trn th I chy trn ng no?
Hng dn
c) Chng minh: Ba im H, O, K thng hng.
Gi I l giao im HK v MC.
Do MHCK l hnh vung
HK MC ti trung im I ca MC.
Do I l trung im MC
OI MC (t/c ng knh v dy cung)
Vy HI MC; OI MC v KI MC
Suy ra: H; O;I thng hng.
d) Do

0
OIM 90 = ; OM c nh
Suy ra: I nm trn ng trn ng knh OM.
Gii hn:
Khi C B th I Q;
Khi C A th I P.
Vy khi C di ng trn na ng trn (O) th I chy trn cung trn PHQ ca ng trn ng
knh OM.
Bi tp 18: Cho hnh ch nht ABCD c chiu di AB = 2a, chiu rng BC = a. K tia phn gic
ca

ACD. T A h AH vung gc vi ng phn gic ni trn.


a) Chng minh: T gic AHDC ni tip trong ng trn (O). Khi xc nh tm v bn knh ca
ng trn theo a.
b) K HB ct AD ti I v ct AC ti M; HC ct DB ti N.
Chng t rng: HB = HC v AB.AC = BH.BI.
c) Chng t MN song song vi tip tuyn ti H ca (O)
d) T D k ng thng song song vi BH; ng ny ct HC K v ct (O) J.
Chng minh: T gic HOKD ni tip.
Bi tp 19: Cho na ng trn (O) ng knh AB, bn knh OC AB. Gi M l 1 im trn cung
BC. K ng cao CH ca AACM.
a) Chng minh: T gic AOHC ni tip.
b) Chng t ACHM vung cn v OH l phn gic ca

COM.
.:: CHUYN N THI VO LP 10 ::.
Bin son: Trn Trung Chnh 107
c) Gi giao im ca OH vi BC l I. MI ct (O) ti D. Chng minh rng: T gic CDBM l hnh
thang cn.
d) K BM ct OH ti N. Chng minh: ABNI AAMC. T suy ra: BN.MC = IN.MA.
Bi tp 20: Cho AABC u ni tip trong (O; R). Trn cnh AB v AC ly hai im M; N sao cho
BM = AN.
a) Chng t rng: AOMN cn.
b) Chng minh: T gic OMAN ni tip.
c) Ko di BO ct AC ti D v ct (O) E. Chng minh: BC
2
+ DC
2
= 3R
2
.
d) ng thng CE v AB ct nhau F. Tip tuyn ti A ca (O) ct FC ti I; AO ko di ct BC ti
J. Chng minh: BI i qua trung im ca AJ.
Hng dn
c) Chng minh: BC
2
+ DC
2
= 3R
2
.
Do BO l phn gic ca A u
BO AC hay ABOD vung D.
p dng nh l Pi-ta-go, ta c:
BC
2
= DB
2
+ CD
2
= (BO + OD)
2
+ CD
2
= BO
2
+ 2.OB.OD + OD
2
+ CD
2
. (1)
M OB = R.
AAOC cn O c

0
OAC 30 = .

0
AOC 120 =

0
AOE 60 =
AAOE l tam gic u, c AD OE OD = ED =
R
2

p dng nh l Pi-ta-go, ta c: OD
2
= OC
2
- CD
2
= R
2
- CD
2
. (2)
T (1) v (2), suy ra: BC
2
= R
2
+ 2.R.
R
2
+ CD
2
- CD
2
= 3R
2
.
Bi tp 21: Cho AABC, (A = 90
0
) ni tip trong ng trn (O). Gi M l trung im cnh AC.
ng trn (I) ng knh MC ct cnh BC N v ct (O) ti D.
a) Chng minh: T gic ABNM ni tip v CN.AB = AC.MN.
b) Chng t rng: B, M, D thng hng v OM l tip tuyn ca (I).
c) Tia IO ct ng thng AB ti E. Chng minh: T gic BMOE l hnh bnh hnh.
d) Chng minh: NM l phn gic ca

AND.
Bi tp 22: Cho hnh vung ABCD c cnh bng a. Gi I l im bt k trn ng cho AC. Qua I
k cc ng thng song song vi AB; BC. Cc ng ny ct AB; BC; CD; DA ln lt P; Q; N;
M.
a) Chng minh: T gic INCQ l hnh vung.
b) Chng t rng: NQ // DB.
c) Ko di BI ct MN ti E; MP ct AC ti F. Chng minh: T gic MFIN ni tip c trong
ng trn. Xc nh tm ca ng trn .
d) Chng t t gic MPQN ni tip. Tnh din tch ca n theo a.
e) Chng minh: T gic MFIE ni tip.
Bi tp 23: Cho hnh vung ABCD. Gi N l trung im DC; K BN ct AC ti F. V ng trn
(O) ng knh BN. (O) ct AC ti E. Ko di BE ct AD M; MN ct (O) ti I.
a) Chng minh: T gic MDNE ni tip.
b) Chng t rng: ABEN vung cn.
c) Chng minh: MF i qua trc tm H ca ABMN.
d) Chng minh: BI = BC v AIEF vung.
e) Chng minh: AFIE l tam gic vung.
Bi tp 24: Cho AABC c 3 gc nhn(AB < AC). V ng cao AH. T H k HK; HM ln lt
vung gc vi AB; AC. Gi J l giao im ca AH v MK.
www.VNMATH.com
.:: CHUYN N THI VO LP 10 ::.
Bin son: Trn Trung Chnh 108
a) Chng minh: T gic AMHK ni tip.
b) Chng minh: JA.JH = JK.JM
c) T C k tia Cx AC v Cx ct AH ko di D. V HI DB v HN DC. Chng minh rng:

HKM HCN = .
d) Chng minh: M; N; I; K cng nm trn mt ng trn.
Bi tp 25: Cho AABC (A = 90
0
). ng cao AH. ng trn tm H, bn knh HA ct ng thng
AB ti D v ct AC ti E; Trung tuyn AM ca AABC ct DE ti I.
a) Chng minh: D; H; E thng hng.
b) Chng minh: T gic BDCE ni tip. Xc nh tm O ca ng trn ny.
c) Chng minh: AM DE.
d) Chng minh: T gic AHOM l hnh bnh hnh.
Bi tp 26: Cho AABC c 2 gc nhn. ng cao AH. Gi K l im i xng ca H qua AB; I l
im i xng ca H qua AC. Gi E; F l giao im ca KI vi AB v AC.
a) Chng minh: T gic AICH ni tip.
b) Chng minh: AI = AK.
c) Chng minh: Cc im A; E; H; C; I cng nm trn mt ng trn.
d) Chng minh: CE; BF l cc ng cao ca AABC.
e) Chng t giao im 3 ng phn gic ca AHFE chnh l trc tm ca AABC.
Bi tp 27: Cho AABC, (AB = AC) ni tip trong (O). Gi M l mt im bt k trn cung nh AC.
Trn tia BM ly MK = MC v trn tia BA ly AD = AC.
a) Chng minh:

BAC 2BKC = .
b) Chng minh: T gic BCKD ni tip. Xc nh tm ca ng trn ny.
c) Gi giao im ca DC vi (O) l I. Chng minh: B; O; I thng hng.
d) Chng minh: DI = BI.
Bi tp 28: Cho t gic ABCD ni tip trong(O). Gi I l im chnh gia cung AB (cung AB
khng cha im C; D). IC v ID ct AB M; N.
a) Chng minh: D; M; N; C cng nm trn mt ng trn.
b) Chng minh: NA.NB = NI.NC.
c) Ko di DI ct ng thng BC F; ng thng IC ct ng thng AD E.
Chng minh: EF // AB.
d) Chng minh: IA
2
= IM.ID.
Bi tp 29: Cho hnh vung ABCD, trn ch BC l E. Dng tia Ax AE, Ax ct cnh CD ko
di ti F. K trung tuyn AI ca AAEF. Ko di AIct CD ti K. Qua E dng ng thng song song
vi AB, ct AI ti G.
a) Chng minh: T gic AECF ni tip.
b) Chng minh: AF
2
= KF.CF.
c) Chng minh: T gic EGFK l hnh thoi.
d) Chng minh rng: Khi E di ng trn BC th EK = BE + DK v chu vi ACKE c gi tr khng
i.
e) Gi giao im ca EF vi AD l J. Chng minh: GJ JK.
Hng dn
d) Chng minh: EK = BE + DK.
Xt AADF v AABE c:
AD = AB;
AF = AE (AAEF vung cn)
AADF = AABE
BE = DF
M FD + DK = FK v FK = KE (t/c hnh thoi)
KE = BE + DK.
.:: CHUYN N THI VO LP 10 ::.
Bin son: Trn Trung Chnh 109
Chng minh chu vi ACKE khng i:
Gi chu vi l C = KC + EC + KE = KC + EC + BE + DK = (KC + DK) + (BE + EC) = 2BC khng
i.
e) Chng minh: IJ JK.
Do

0
JIK JDK 90 = =
T gic IJDK ni tip


JIK IDK = (cng chn cung IK),

0
IDK 45 = (t/c hnh vung)

0
JIK 45 = AJIK vung cn I
JI = IK, m IK = GI
JI = IK = GI =
2
1
GK
AGJK vung J hay GJ JK.
Bi tp 30: Cho AABC. Gi H l trc tm ca tam gic. Dng hnh bnh hnh BHCD. Gi I l giao
im ca HD v BC.
a) Chng minh: T gic ABDC ni tip trong ng trn tm O, nu cch dng (O).
b) So snh

BAH v

OAC.
c) K CH ct OD ti E. Chng minh: AB.AE = AH.AC.
d) Gi giao im ca AI v OH l G. Chng minh: G l trng tm ca AABC.
Bi tp 31: Cho ng trn (O) v

0
AB 90 = . C l mt tu trn cung ln AB. Cc ng cao
AI; BK; CJ ca AABC ct nhau H. K BK ct (O) N; AH ct (O) ti M. BM v AN gp nhau
D.
a) Chng minh: B; K; C; J cng nm trn mt ng trn.
b) Chng minh: BI.KC = HI.KB.
c) Chng minh: MN l ng knh ca ng trn (O).
d) Chng minh: T gic ACBD l hnh bnh hnh.
e) Chng minh: OC // DH.
Bi tp 32: Cho hnh vung ABCD. Gi N l mt bt k trn CD sao cho CN < ND; V ng
trn tm O ng knh BN. ng trn (O) ct AC ti F; BF ct AD ti M; BN ct AC ti E.
a) Chng minh: ABFN vung cn.
b) Chng minh: MEBA ni tip.
c) Gi giao im ca ME v NF l Q. K MN ct (O) P.
Chng minh: B; Q; P thng hng.
d) Chng t: ME // PC v BP = BC.
e) Chng minh: AFPE l tam gic vung.
Bi tp 33: Trn ng trn tm O ln lt ly bn A; B; C; D sao cho AB = DB.AB v CD ct
nhau c E. K BC ct tip tuyn ti A ca ng trn (O) Q; DB ct AC ti K.
a) Chng minh: CB l phn gic ca

ACE.
b) Chng minh: T gic AQEC ni tip.
c) Chng minh: KA.KC = KB.KD.
d) Chng minh: QE // AD.
Bi tp 34: Cho (O) v tip tuyn Ax. Trn Ax ly hai B v C sao cho AB = BC. K ct tuyn
BEF vi ng trn. K CE v CF ct (O) ln lt M v N. Dng hnh bnh hnh AECD.
a) Chng minh: D nm trn ng thg BF.
b) Chng minh: T gic ADCF ni tip.
c) Chng minh: CF.CN = CE.CM.
d) Chng minh: MN // AC.
www.VNMATH.com
.:: CHUYN N THI VO LP 10 ::.
Bin son: Trn Trung Chnh 110
e) Gi giao im ca AF vi MN l I. Chng minh rng: DF i qua trung im ca NI.
Bi tp 35: Cho (O; R) v ng knh AB; CD vung gc vi nhau. Gi M l mt im trn cung
nh CB.
a) Chng minh: T gic ACBD l hnh vung.
b) K AM ct CD; CB ln lt P v I. Gi J l giao im ca DM v AB.
Chng minh: IB.IC = IA.IM.
c) Chng t rng: IJ // PD v IJ l phn gic ca

CJM.
d) Tnh din tch AAID theo R.
Hng dn
d) Tnh din tch AAID theo R:
S
IAD
= S
CAD
.
M S
ACD
=
2
1
SABCD
.
S
IAD
=
2
1
SABCD
.S
ABCD
=
2
1
AB.CD (din tch c 2 ng cho vung gc)
S
ABCD
=
2
1
2R.2R = 2R
2
S
IAD
= R
b)

Bi tp 36: Cho (O; R). Mt ct tuyn xy ct (O) E v F. Trn xy ly im A nm ngoi on EF.
V 2 tip tuyn AB v AC vi (O). Gi H l trung EF.
a) Chng t 5 im: A; B; C; O; H cng nm trn mt ng trn.
b) ng thng BC ct OA I v ct ng thng OH K. Chng minh: OI.OA = OH.OK = R
2
.
c) Khi A di ng trn xy th I di ng trn ng no?
d) Chng minh: KE v KF l hai tip tuyn ca (O).
Bi tp 37: Cho AABC (A = 90
0
); AB = 15; AC = 20 (cng n v o di). Dng ng trn tm
O ng knh AB v ng trn (O) ng knh AC. Hai ng trn (O) v (O) ct nhau ti im
th hai D.
a) Chng t D nm trn BC.
b) Gi M l chnh gia cung nh DC. AM ct DC E v ct (O) N.
Chng minh: DE.AC = AE.MC
c) Chng minh: AN = NE v O; N; O thng hng.
d) Gi I l trung MN. Chng minh:

0
OIO' 90 = .
e) Tnh din tch AAMC.
Hng dn
c) Chng minh: AN = NE.
Do BA AO(AABC vung A)
BA l tip tuyn ca (O)
s

AE =
2
1
s

AM
S

ED = s
2
1


( )
MC AD +
M

MC DM =

MC AD AM + =


AED BAC =
ABAE cn B, m BM AE
NA = NE.
Chng minh: O; N; O thng hng:
Ta c: ON l ng trung bnh ca AABE
.:: CHUYN N THI VO LP 10 ::.
Bin son: Trn Trung Chnh 111
ON // BE v OO // BE
O, N, O thng hng.
Bi tp 38: Cho AABC u, c cnh bng a. Gi D l giao im hai ng phn gic gc A v gc
B ca AABC. T D dng tia Dx DB. Trn Dx ly im E sao cho ED = DB (D v E nm hai pha
ca ng thng AB). T E k EF BC. Gi O l trung im ca EB.
a) Chng minh: T gic AEBC v EDFB ni tip. Xc nh tm v bn knh ca cc ng trn
ngoi tip cc t gic trn theo a.
b) Ko di FE v pha F, ct (D) ti M. K EC ct (O) N.
Chng minh: T gic EBMC l thang cn. Tnh din tch.
c) Chng minh: EC l phn gic ca

DAC.
d) Chng minh: FD l ng trung trc ca MB.
e) Chng t A; D; N thng hng.
f) Tnh din tch phn mt trng c to bi cung nh EB ca hai ng trn.
Hng dn
e) Chng minh: A; N; D thng hng:
Ta c:

0
BND BED 45 = = (cng chn

DB) v

ENB = 90
o
(cmt);

ENA l gc ngoi AANC




0
ENA NAC CAN 45 = + =


0
ENA ENB BND 180 + + =
A, N, D thng hng.
f) Gi din tch mt trng cn tnh l S.
Ta c: S = S
na (O)
- S
vin phn EDB

S
(O)
= t.OE
2
= t.
2
a 6
6
| |
|
|
\ .

=
2
a
6
t


( )
2
1
O
2
a
S
12
t
=

S
qut EBD
=
2 o
o
.BD .90
360
t

=
2
2
a 6 a
.
4 6 12
| |
t t
=
|
|
\ .

S
AEBD
=
2
1
DB
2
=
2
a
6

S
vin phn
= S
qut EBD
- S
AEDB
=
2
a
12
t
-
2
a
6
=
2
a ( 2)
12
t

S =
2
a
12
t
-
2
a ( 2)
12
t
=
2
a
6
.
Bi tp 39: Cho hnh vung ABCD, E l mt im thuc cnh BC. Qua B k ng thng vung
gc vi DE, ng ny ct cc ng thng DE v DC theo th t H v K.
a) Chng minh: T gic BHCD ni tip.
b) Tnh

CHK.
c) Chng minh: KC.KD = KH.KB.
d) Khi E di ng trn BC th H di ng trn ng no?
Hng dn
d) Do

0
BHD 90 = khng i
Suy ra: E di chuyn trn BC th H di ng trn ng trn ng knh DB.
www.VNMATH.com
.:: CHUYN N THI VO LP 10 ::.
Bin son: Trn Trung Chnh 112
Bi tp 40: Cho ng trn (O;R) ng knh AB. Gi C l im bt k thuc ng trn (C =
A v B). Hai im M, N ln lt l im chnh gia ca cc cung nh AC v BC. Cc ng thng
BN v AC ct nhau ti I, cc dy cung AN v BC ct nhau P.
a) Chng minh: T gic ICPN ni tip. Xc nh tm K ca ng trn ngoi tip t gic .
b) Chng minh: KN l tip tuyn ca ng trn (O; R).
c) Chng minh rng khi C di ng trn ng trn (O; R) th ng thng MN lun tip xc vi mt
ng trn c nh.
Hng dn
c) Chng minh rng khi C di ng trn ng trn (O) th ng thng MN lun tip xc vi mt
ng trn c nh:
Ta c

AM= MC (gt) nn

AOM= MOC.
Vy OM l phn gic ca

AOC.
Tng t ON l phn gic ca

COB, m

AOC v

COB k b nn

0
MON= 90 .
Vy tam gic MON vung cn O.
K OH MN, ta c OH = OM.sinM = R.
2
2
=
R 2
2
khng i.
Vy khi C di ng trn ng trn (O) th ng thng MN lun tip xc vi mt ng trn c
nh
R 2
O;
2
| |
|
|
\ .
.
Bi tp 41: Cho ng trn (O; R) c ng knh AB. Trn ng trn (O; R) ly im M sao cho

0
MAB= 60 . V ng trn (B; BM) ct ng trn (O; R) ti im th hai l N.
a) Chng minh AM v AN l cc tip tuyn ca ng trn (B; BM).
b) K cc ng knh MI ca ng trn (O; R) v MJ ca ng trn (B; BM). Chng minh N, I
v J thng hng v JI.JN = 6R
2

c) Tnh phn din tch ca hnh trn (B; BM) nm bn ngoi ng trn (O; R) theo R.
Hng dn
b) Chng minh: N; I; J thng hng v JI.JN = 6R
2
.

0
MNI = MNJ = 90 (cc gc ni tip chn na ng trn tm O v tm B).
Nn IN MN v JN MN .
Vy ba im N; I v J thng hng.
AMJI c BO l ng trung bnh nn IJ = 2BO = 2R.
AAMO cn O (v OM = OA),

0
MAO= 60 nn AMAO u.
AB MN ti H (tnh cht dy chung ca hai ng trn (O) v (B) ct nhau).
Nn OH =
1 1
OA = R
2 2
.
Vy HB = HO + OB =
R 3R
+R =
2 2

3R
NJ = 2. = 3R
2
.
Vy JI.JN = 2R.3R = 6R
2
.
c) Tnh din tch phn hnh trn (B; BM) nm ngoi ng trn (O; R) theo R:
Gi S l din tch phn hnh trn nm (B; BM) nm bn ngoi hnh trn (O; R).
S
1
l din tch hnh trn tm (B; BM).
S
2
l din tch hnh qut MBN.
S
3
, S
4
l din tch hai vin phn cung MB v NB ca ng trn (O; R).
Ta c : S = S
1
(S
2
+ S
3
+ S
4
).
.:: CHUYN N THI VO LP 10 ::.
Bin son: Trn Trung Chnh 113
Tnh S
1
:

0 0
MAB 60 MB 120 = =
MB= R 3 .
Vy: S
1
=
( )
2
2
R 3 = 3R .
Tnh S
2
:

0
MBN= 60 S
2
=
( )
2
0
0
R 3 60
360
=
2
R
2

Tnh S
3
: S
3
= S
qut MOB
S
MOB
.

0
MOB=120 S
qut MOB
=
2 0 2
0
R .120 R
=
360 3
.
OA = OB S
MOB
=
1
2
S
AMB
=
1 1
. .AM.MB
2 2
=
1
R.R 3
4
=
2
R 3
4

Vy S
3
=
2
R
3
2
R 3
-
4
= S
4
(do tnh cht i xng).
T S = S
1
- (S
2
+ 2S
3
) =
2
3R
2 2 2
R 2R R 3
+ -
2 3 2
| |
|
|
\ .
=
2 2
11R +3R 3
6
(vdt).
Bi tp 42: Cho ba im A, B, C nm trn ng thng xy theo th t . V ng trn (O) i qua
B v C. T A v hai tip tuyn AM v AN. Gi E v F ln lt l trung im ca BC v MN.
a) Chng minh AM
2
= AN
2
= AB. AC
b) ng thng ME ct ng trn (O) ti I. Chng minh IN // AB
c) Chng minh rng tm ng trn ngoi tip tam gic OEF nm trn mt ng thng c nh khi
ng trn (O) thay i.
Bi tp 43: Cho na ng trn ng knh AB = 2R v dy MN c di bng bn knh (M thuc
cung AN). Cc tia AM v BN ct nhau I. Cc dy AN v BM ct nhau K.
a) Tnh

MIN v

AKB.
b) Tm qu tch im I v qu tch im K khi dy MN thay i v tr .
c) Chng minh I l trc tm ca tam gic KAB .
d) AB v IK ct nhau ti H . Chng minh HA.HB = HI.HK .
e) Vi v tr no ca dy MN th tam gic IAB c din tch ln nht? Tnh gi tr din tch ln nht
theo R.
Bi tp 44: Cho na ng trn ng knh AB = 2R. T A v B k hai tip tuyn Ax, By. Qua
im M thuc na ng trn k tip tuyn th ba ct cc tip tuyn Ax , By ln lt C v D. Cc
ng thng AD v BC ct nhau ti N.
a) Chng minh AC + BD = CD.
b) Chng minh:

0
COD 90 = .
c) Chng minh: AC.BD =
2
AB
4
.
d) Chng minh: OC // BM
e) Chng minh: AB l tip tuyn ca ng trn ng knh CD.
f) Chng minh: MN AB.
g) Xc nh v tr ca M chu vi t gic ACDB t gi tr nh nht.
Hng dn
g) Ta c:
Chu vi t gic: ACDB = AB + AC + CD + BD.
M AC + BD = CD.
Suy ra chu vi t gic ACDB = AB + 2CD.
www.VNMATH.com
.:: CHUYN N THI VO LP 10 ::.
Bin son: Trn Trung Chnh 114
M AB khng i nn chu vi t gic ACDB nh nht khi CD nh nht.
V CD nh nht khi CD l khong cch gi Ax v By tc l CD vung gc vi Ax v By.
Khi CD // AB.
Suy ra: M phi l trung im ca cung AB.
Bi tp 45: Cho ng trn (O; R), t mt im A trn (O) k tip tuyn d vi (O). Trn ng
thng d ly im M bt k (M khc A) k ct tuyn MNP v gi K l trung im ca NP. K tip
tuyn MB (B l tip im). K AC MB, BD MA. Gi H l giao im ca AC v BD, I l giao
im ca OM v AB.
a) Chng minh: T gic AMBO ni tip.
b) Chng minh: Nm im O, K, A, M, B cng nm trn mt ng trn .
c) Chng minh: OI.OM = R
2
; OI. IM = IA
2
.
d) Chng minh: T gic OAHB l hnh thoi.
e) Chng minh: Ba im O, H, M thng hng.
f) Tm qu tch ca im H khi M di chuyn trn ng thng d.
Hng dn
e) Theo trn OAHB l hnh thoi.
Suy ra: OH AB; cng theo trn OM AB
Suy ra: O, H, M thng hng (v qua O ch c mt ng thng vung gc vi AB).
f) Theo trn OAHB l hnh thoi.
Suy ra: AH = AO = R.
Vy khi M di ng trn d th H cng di ng nhng lun cch A c nh mt khong bng R. Do
qu tch ca im H khi M di chuyn trn ng thng d l na ng trn tm A bn knh AH =
R.
Bi tp 46: Cho na ng trn ng knh AB = 2R. T A v B k hai tip tuyn Ax, By. Qua
im M thuc na ng trn k tip tuyn th ba ct cc tip tuyn Ax , By ln lt C v D. Cc
ng thng AD v BC ct nhau ti N.
a) Chng minh: AC + BD = CD.
b) Chng minh:

0
COD 90 = .
c) Chng minh: AC. BD =
2
AB
4
.
d) Chng minh: OC // BM
e) Chng minh AB l tip tuyn ca ng trn ng knh CD.
e) Chng minh: MN AB.
f) Xc nh v tr ca M chu vi t gic ACDB t gi tr nh nht.
Hng dn
f) Ta c chu vi t gic ACDB = AB + AC + CD + BD m AC + BD = CD.
Suy ra chu vi t gic ACDB = AB + 2CD m AB khng i.
Chu vi t gic ACDB nh nht khi CD nh nht.
M CD nh nht khi CD l khong cch gi Ax v By, tc l CD vung gc vi Ax v By.
Khi CD // AB M phi l trung im ca cung AB.
Bi tp 47: Cho ng trn (O; R), t mt im A trn (O) k tip tuyn d vi (O). Trn ng
thng d ly im M bt k (M khc A) k ct tuyn MNP. Gi K l trung im ca NP, k tip tuyn
MB (B l tip im). K AC MB, BD MA. Gi H l giao im ca AC v BD, I l giao im
ca OM v AB.
a) Chng minh t gic AMBO ni tip.
b) Chng minh nm im O, K, A, M, B cng nm trn mt ng trn .
c) Chng minh: OI.OM = R
2
; OI. IM = IA
2
.
d) Chng minh OAHB l hnh thoi.
e) Chng minh ba im O, H, M thng hng.
.:: CHUYN N THI VO LP 10 ::.
Bin son: Trn Trung Chnh 115
f) Tm qu tch ca im H khi M di chuyn trn ng thng d
Hng dn
e) Theo trn OAHB l hnh thoi. Suy ra: OH AB; cng theo trn OM AB.
Suy ra: O, H, M thng hng (v qua O ch c mt ng thng vung gc vi AB).
f) Theo trn OAHB l hnh thoi. Suy ra: AH = AO = R.
Vy khi M di ng trn d th H cng di ng nhng lun cch A c nh mt khong bng R. Do
qu tch ca im H khi M di chuyn trn ng thng d l na ng trn tm A bn knh AH =
R.
Bi tp 48: Cho ng trn (O; R) ng knh AB. K tip tuyn Ax v ly trn tip tuyn mt
im P sao cho AP > R. T P k tip tuyn tip xc vi (O) ti M.
a) Chng minh rng t gic APMO ni tip c mt ng trn.
b) Chng minh BM // OP.
c) ng thng vung gc vi AB O ct tia BM ti N. Chng minh t gic OBNP l hnh bnh
hnh.
d) Bit AN ct OP ti K, PM ct ON ti I; PN v OM ko di ct nhau ti J. Chng minh I, J, K
thng hng.
Hng dn
d) T gic OBNP l hnh bnh hnh.
Suy ra: PN // OB hay PJ // AB.
M ON AB ON PJ.
Ta cng c PM OJ (PM l tip tuyn ).
M ON v PM ct nhau ti I nn I l trc tm tam gic POJ.
D thy t gic AONP l hnh ch nht.
V c

0
PAO AON ONP 90 = = = .
Suy ra: K l trung im ca PO (tnh cht ng cho hnh ch nht). (6)
Ta c: AONP l hnh ch nht

APO NOP = (so le) (7)
Theo tnh cht hai tip tuyn ct nhau th:
PO l tia phn gic ca gc

APM APO MPO = (8)
T (7) v (8) AIPO cn ti I c IK l trung tuyn ng thi l ng cao.
Suy ra: IK PO. (9)
T (6) v (9) I, J, K thng hng.
Bi tp 49: Cho ng trn (O) bn knh R c hai ng knh AB v CD vung gc vi nhau. Trn
on thng AB ly im M (M khc O). CM ct (O) ti N. ng thng vung gc vi AB ti M ct
tip tuyn ti N ca ng trn P. Chng minh :
a) T gic OMNP ni tip.
b) T gic CMPO l hnh bnh hnh.
c) CM. CN khng ph thuc vo v tr ca im M.
d) Khi M di chuyn trn on thng AB th P chy trn on thng c nh no.
Hng dn
d) D thy AOMC = ADPO (c.g.c).
Suy ra:

0
ODP 90 = .
Suy ra: P chy trn ng thng c nh vung gc vi CD ti D.
V M ch chy trn on thng AB nn P ch chy trn don thng AB song song v bng AB.
Bi tp 50: Cho AABC vung A.v mt im D nm gia A v B. ng trn ng knh BD
ct BC ti E. Cc ng thng CD, AE ln lt ct ng trn ti F, G.
a) Chng minh: AABC AEBD.
b) Chng minh: T gic ADEC v AFBC ni tip .
c) Chng minh: AC // FG.
d) Chng minh: Cc ng thng AC, DE, FB ng quy.
www.VNMATH.com
.:: CHUYN N THI VO LP 10 ::.
Bin son: Trn Trung Chnh 116
Hng dn
d) D thy CA, DE, BF l ba ng cao ca ADBC nn CA, DE, BF ng quy ti S.
Bi tp 51: Cho ng trn (O) ng knh AB. Trn on thng OB ly im H bt k (H khng
trng O, B); trn ng thng vung gc vi OB ti H, ly mt im M ngoi ng trn; MA v
MB th t ct ng trn (O) ti C v D. Gi I l giao im ca AD v BC.
a) Chng minh MCID l t gic ni tip .
b) Chng minh cc ng thng AD, BC, MH ng quy ti I.
c) Gi K l tm ng trn ngoi tip t gic MCID. Chng minh KCOH l t gic ni tip.
Bi tp 52: Cho hnh vung ABCD. Ly B lm tm, bn knh AB, v 1/4 ng trn pha trong
hnh vung. Ly AB lm ng knh, v 1/2 ng trn pha trong hnh vung. Gi P l im tu
trn cung AC (khng trng vi A v C). H v K ln lt l hnh chiu ca P trn AB v AD, PA v
PB ct na ng trn ln lt I v M.
a) Chng minh I l trung im ca AP.
b) Chng minh PH, BI, AM ng qui.
c) Chng minh PM = PK = AH
d) Chng minh t gic APMH l hnh thang cn.
e) Tm v tr im P trn cung AC tam gic APB l u.
Bi tp 53: Cho ng trn (O) v mt dy AB. Gi M l im chnh gia ca cung nh AB. V ng
knh MN Ct AB ti I. Gi D l mt im thuc dy AB. Tia MD ct ng trn (O) ti C.
a) Chng minh rng t gic CDIN ni tip c
b) Chng minh rng tch MC. MD c gi tr khng i khi D di ng trn dy AB.
c) Gi O' l tm ca ng trn ngoi tip tam gic ACD. Chng minh rng:

1
MAB AO' D
2
= .
d) Chng minh rng ba im A, O', N thng hng v MA l tip tuyn ca ng trn ngoi tip tam
gic ACD.
Bi tp 54: Cho tam gic vung cn ABC (

A = 90
0
), trung im I ca cnh BC. Xt mt im D
trn tia AC. V ng trn (O) tip xc vi cc cnh AB, BD, DA ti cc im tng ng M, N, P.
a) Chng minh rng 5 im B, M, O, I, N nm trn mt ng trn.
b) Chng minh rng ba im N, I, P thng hng.
c) Gi giao im ca tia BO vi MN, NP ln lt l H, K. Tam gic HNK l tam gic g, ti sao?
d) Tm tp hp im K khi im D thay i v tr trn tia AC.
Bi tp 55: Cho hai ng trn (O) v (O') ct nhau ti hai im A v B. ng thng AO ct
ng trn (O) v (O') ln lt ti C v C'. ng thng AO' ct ng trn (O) v (O') ln lt ti
D v D'.
a) Chng minh C, B, D' thng hng
b) Chng minh t gic ODC'O' ni tip
c) ng thng CD v ng thng D'C' ct nhau ti M. Chng minh t gic MCBC' ni tip.
Bi tp 56: T mt im C ngoi ng trn ( O) k ct tuyn CBA. Gi IJ l ng knh vung
gc vi AB. Cc ng thng CI, CJ theo th t ct ng trn (O) ti M, N.
a) Chng minh rng IN, JM v AB ng quy ti mt im D.
b) Chng minh rng cc tip tuyn ca ng trn (O) ti M, N i qua trung im E ca CD.
Bi tp 57: Cho hai ng trn ( O; R) v ( O'; R' ) tip xc ngoi ti A ( R > R' ). ng ni tm OO'
ct ng trn (O) v (O') theo th t ti B v C ( B v C khc A). EF l dy cung ca ng trn (O)
vung gc vi BC ti trung im I ca BC, EC ct ng trn (O') ti D.
a) T gic BEFC l hnh gi?
b) Chng minh ba im A, D, F thng hng.
c) CF ct ng trn (O) ti G. Chng minh ba ng EG, DF v CI ng quy.
d) Chng minh ID tip xc vi ng trn (O).
Bi tp 58:Cho ng trn (O) v (O) tip xc ngoi ti C. AC v BC l ng knh ca (O) v
(O), DE l tip tuyn chung ngoi (D e (O), E e (O)). AD ct BE ti M.
.:: CHUYN N THI VO LP 10 ::.
Bin son: Trn Trung Chnh 117
a) AMAB l tam gic g?
b) Chng minh: MC l tip tuyn chung ca (O) v (O).
c) K Ex, By vung gc vi AE, AB. Ex ct By ti N. Chng minh: D, N, C thng hng.
d) V cng pha ca na mt phng b AB, v na ng trn ng knh AB v OO. ng
thng qua C ct hai na ng tn trn ti I, K. Chng minh OI // AK.
Bi tp 59: Cho ng trn (O ; R). ng thng d ct (O) ti A, B. C thuc d ngoi (O). T
im chnh gia P ca cung ln AB k ng knh PQ ct AB ti D. CP ct (O) ti im th hai I,
AB ct IQ ti K.
a) Chng minh t gic PDKI ni tip.
b) Chng minh: CI.CP = CK.CD.
c) Chng minh IC l phn gic ngoi ca tam gic AIB.
d) A, B, C c nh, (O) thay i nhng vn lun qua A, B. Chng minh rng IQ lun i qua im c
nh.
Bi tp 60:Cho tam gic u ABC ni tip (O ; R). M di ng trn AB. N di ng trn tia i ca tia
CA sao cho BM = CN.
a) ng trn ngoi tip tam gic AMN ct (O) ti A v D. Chng minh rng D c nh.
b) Tnh gc MDN.
c) MN ct BC ti K. Chng minh DK vung gc vi MN.
d) t AM = x. Tnh x din tch tam gic AMN l ln nht.
Bi tp 61: Cho (O; R). im M c nh ngoi (O). Ct tuyn qua M ct (O) ti A v B. Tip
tuyn ca (O) ti A v B ct nhau ti C.
a) Chng minh t gic OACB ni tip ng trn tm K.
b) Chng minh: (K) qua hai im c nh l O v H khi ct tuyn quay quanh M.
c) CH ct AB ti N, I l trung im AB. Chng minh: MA.MB = MI.MN.
d) Chng minh: IM.IN = IA
2

Bi tp 62: Cho na ng trn ng knh AB tm O. C l im chnh gia cung AB. M di ng
trn cung nh AC. Ly N thuc BM sao cho AM = BN.
a) So snh AAMC v ABCN.
b) ACMN l tam gic g?
c) K dy AE//MC. Chng minh t gic BECN l hnh bnh hnh.
d) ng thng d i qua N v vung gc vi BM. Chng minh d lun i qua im c nh.
Bi tp 63: Cho ng trn (O ; R), ng thng d ct (O) ti hai im C v D. im M tu trn
d, k tip tuyn MA, MB. I l trung im ca CD.
a) Chng minh 5 im M, A, I, O, B cng thuc mt ng trn.
b) Gi H l trc tm ca AMAB, t gic OAHB l hnh g?
c) Khi M di ng trn d. Chng minh rng AB lun qua im c nh.
d) ng thng qua C vung gc vi OA ct AB, AD ln lt ti E v K. Chng minh: EC = EK.
Bi tp 64: Cho AABC cn (AB = AC) ni tip trong ng trn (O) v M l im di ng trn ng
trn . Gi D l hnh chiu ca B trn AM v P l giao im ca BD vi CM.
a) Chng minh ABPM cn.
b) Tm qu tch ca im D khi M di chuyn trn ng trn (O).
Bi tp 65: ng trn (O ; R) ct mt ng thng d ti hai im A, B. T mt im M trn d v
ngoi ng trn (O) k cc tip tuyn MP, MQ.
a) Chng minh rng:

QMO QPO = v ng trn ngoi tip AMPQ i qua hai im c nh khi M
di ng trn d.
b) Xc nh v tr ca M MQOP l hnh vung?
c) Tm qu tch tm cc ng trn ni tip AMPQ khi M di ng trn d.
Bi tp 66: Hai ng trn tm O v tm I ct nhau ti hai im A v B. ng thng d i qua A
ct cc ng trn (O) v (I) ln lt ti P, Q. Gi C l giao im ca hai ng thng PO v QI.
www.VNMATH.com
.:: CHUYN N THI VO LP 10 ::.
Bin son: Trn Trung Chnh 118
a) Chng minh rng cc t gic BCQP, OBCI ni tip.
b) Gi E, F ln lt l trung im ca AP, AQ, K l trung im ca EF. Khi ng thng d quay
quanh A th K chuyn ng trn ng no?
c) Tm v tr ca d APQB c chu vi ln nht.
Bi tp 67: Cho hnh hp ch nht ABCDABCD. Bit AB = 4 cm; AC = 5 cm v AC = 13 cm.
Tnh th tch v din tch xung quanh ca hnh hp ch nht .
Bi tp 68: Cho hnh lp phng ABCDABCD c din tch mt cho ACCA bng 25 2 cm
2
.


Tnh th tch v din tch ton phn ca hnh lp phng .
Bi tp 69: Cho hnh hp ch nht ABCDABCD. Bit AB = 15 cm, AC = 20 cm v

0
A' AC' 60 = . Tnh th tch v din tch ton phn ca hnh hp ch nht .
Bi tp 70: Cho lng tr ng tam gic u ABCABC. Tnh din tch xung quanh v th tch ca
n bit cnh y di 6 cm v gc AAB bng 30
0
.
Bi tp 71: Cho AABC u cnh a. ng thng d vung gc vi mt phng (ABC) ti trng tm G
ca AABC. Trn ng thng d ly mt im S. Ni SA, SB, SC.
a) Chng minh rng: SA = SB = SC.
b) Tnh din tch ton phn v th tch ca hnh chp S.ABC, cho bit SG = 2a.
Bi tp 72: Cho hnh chp t gic u S.ABCD c cnh y l a v ng cao l
a 2
2
.
a) Chng minh cc mt bn ca hnh chp l cc tam gic u.
b) Tnh th tch v din tch xung quanh ca hnh chp.
Bi tp 73: Cho hnh chp tam gic u S.ABC c cnh y v cnh bn u bng a.
a) Tnh din tch ton phn ca hnh chp.
b) Tnh th tch ca hnh chp.
Bi tp 74: Cho hnh chp t gic u S.ABCD c chiu cao 15cm v th tch l 1280cm
3
.
a) Tnh di cnh y.
b) Tnh din tch xung quanh ca hnh chp.
Bi tp 75: Mt hnh chp ct din tch y nh l 75cm
2
, din tch y ln gp 4 ln din tch y
nh v chiu cao l 6 cm. Tnh th tch ca hnh chp ct .
Bi tp 76: Cho hnh chp t gic S.ABCD c y ABCD l hnh vung cnh a, SA = a v SA
vung gc vi mt phng y (ABCD).
a) Tnh th tch hnh chp.
b) Chng minh rng bn mt bn l nhng tam gic vung.
c) Tnh din tch xung quanh ca hnh chp.
Bi tp 77: Mt hnh tr c ng cao bng ng knh y. Bit th tch hnh tr l 128tcm
3
, tnh
din tch xung quanh ca n.
Bi tp 78: Mt hnh nn c bn knh y bng 5 cm v din tch xung quanh bng 65tcm
2
. Tnh
th tch ca hnh nn .
Bi tp 79: Cho hnh nn ct, bn knh y ln bng 8 cm, ng cao bng 12cm v ng sinh
bng 13 cm.
a) Tnh bn knh y nh.
b) Tnh din tch xung quanh v th tch ca hnh nn ct .
Bi tp 80: Mt hnh cu c din tch b mt l 36t cm
2
. Tnh th tch ca hnh cu .

You might also like